Текст
                    МЕТОДИКА
ФАКУЛЬТАТИВНЫХ
ЗАНЯТИЙ
е 78 шсш
ИЗБРАННЫЕ
ВОПРОСЫ
МАТЕМАТИКИ
•
ПОСОБИЕ
ДЛЯ УЧИТЕЛЕЙ
Составители:
И. Л. Никольская,
В. В, Фирсов
Москва «Просвещение» 1981


ББК 74.262 M54 А. М. Абрамов, И. Н. Антипов, JI. Ю. Березина, С. С. Минаева, И. JI. Никольская Рекомендовано к изданию Главным управлением школ Министерства просвещения СССР Методика факультативных занятий в 7—8 классах: Избр. M54 вопросы математики. Пособие для учителей /Сост. И. JI. Ни¬ кольская, В. В. Фирсов.'—М.: Просвещение, 1981.—160 с. Данное пособие содержит методические рекомендации к проведению факуль* тативных ааиятий. В книге приводятся также содержание эачетов по каждой те¬ ме. ответы и указания к наиболее трудным задачам и упражнениям OT01-7g <30M10100 H*7<.2<B 103 (03) — 81 * 51 @ Издательство «Просвещение», 1981 г.
ПРЕДИСЛОВИЕ Назначение этой книги — помочь учителю в проведении факультативных занятий по математике в VII—VIII классах. Пособие содержит методические рекомендации ко всем темам, которые вошли в книгу «Факультативный курс. Избранные вопросы математики, 7—8 классы» (М., Просвещение, 1978), а также к темам «Приложения теории графов» и «Алгоритмы и программи¬ рование». Разделы книги носят те же названия, что и соответствующие темы факультативного курса. ’ Методические рекомендации по теме «Приложения теории графов» ориентированы на книгу JI. Ю. Бе¬ резиной «Графы и их применение» (М., Просвещение, 1979) и на статью «Транспортные сети», помещенную в этой книге. Учебный материал по теме «Алгоритмы и программирование» содержится в книге И. Н. Антипова «Программирование» (М., Просвещение, 1976). Для каждой темы дается ее общая характеристика, выясняются научное и прикладное значение, связь с обязательным курсом; ука¬ зываются основные и вспомогательные понятия, рассматриваются с методической точки зрения важнейшие факты и методы; предлага¬ ется примерное содержание зачета или контрольной работы. В конце каждой темы даются ответы и указания, а к наиболее трудным задачам — решения. Ответы, данные в учебном пособии, здесь не дублируются; однако для некоторых из них даются комментарии (указания либо полные решения). Приводятся списки дополни¬ тельной литературы. Учитывая разнохарактерность содержания и структуры учебных материалов по различным темам, мы не стремились к полной унифи¬ кации структуры и стиля изложения разделов данной книги; каждый из них в той или иной степени отражает своеобразие и осо¬ бенности содержания соответствующей темы в учебном пособии. В конце книги дается ряд задач, связанных с основнымкурсом математики VII—VIII классов, а также задач на соображение, не требующих никаких специальных знаний, но развивающих наблю¬ дательность, воображение, смекалку, логическое мышление. Их мож¬ но предлагать как на занятиях, специально отведенных программой на решение задач повышенной трудности, так и на занятиях по другим темам, для разминки, или отдыха, путем переключения вни¬ мания. К задачам даны ответы и решения. з
СИСТЕМЫ СЧИСЛЕНИЯ И АРИФМЕТИЧЕСКИЕ ОСНОВЫ ЭЛЕКТРОННЫХ ВЫЧИСЛИТЕЛЬНЫХ МАШИН Электронные вычислительные машины, широ¬ ко используемые в настоящее время, служат для преобразования числовой информации или информации другой природы, но записан¬ ной в числовом виде. Поэтому на начальном этапе знакомства уча¬ щихся с ЭВМ полезно рассмотреть системы счисления и арифмети¬ ческие основы электронно-вычислительной техники. Изучение прин¬ ципов подготовки информации для ЭВМ предусмотрено программой факультативного курса в VIII—X классах. Содержание главы «Сйстемы счисления и арифметические осно¬ вы электронных вычислительных машин» учебного пособия вклю¬ чает вопросы, связанные с историей возникновения систем счисле¬ ния, с выполнением арифметических действий в различных систе¬ мах счисления и с переводом чисел из одной системы счисления в дру¬ гую. Показаны преимущества двоичной системы счисления перед другими в практике разработки электронной вычислительной тех¬ ника. Изучение этого материала имеет общеобразовательное значе¬ ние. Рассматривая различные системы счисления, учащимся можно показать, что развитие счета — исторически сложившийся процесс» возникший и развивающийся в зависимости от потребностей об¬ щества* Материалы учебного пособия написаны в соответствии с про¬ граммой факультативного курса и рассчитаны на12ч. Для выра¬ ботки у учащихся необходимых умений в учебном пособии после каждого' пункта приводятся упражнения. Примерное распределение учебного времени. Непозиционные и позивдонные системы счисления. Арифмети¬ ческие действия в позиционных системах счисления — 3 ч. Перевод чисел из одной системы счисления в другую — 3 ч. Двоичная система счисления. Двоичная арифметика — 2 ч. Смешанные системы счисления. Арифметические действия в электронной вычислительной машине — 3 ч. Контрольная работа — 1 ч. 4
НЕПОЗИЦИОННЫЕ И ПОЗИЦИОННЫЕ СИСТЕМЫ СЧИСЛЕНИЯ. АРИФМЕТИЧЕСКИЕ ДЕЙСТВИЯ В ПОЗИЦИОННЫХ СИСТЕМАХ СЧИСЛЕНИЯ О системах счисления, отличных от десятичной, учащимся в ознакомительном плане сообщалось на уроках математики в IV—V классах. В целях более глубокого понимания сущности систем счис¬ ления целесообразно рассмотреть некоторые системы счисления, используемые народами в разные исторические периоды. Ведь да- леко не сразу привычная нам десятичная система счисления полу¬ чила повсеместное распространение. В учебных материалах приводятся примеры римской системы счисления и славянской нумерации — это нелозиционные системы счисления, вавилонской системы, имеющей черты позиционных сис¬ тем, двенадцатеричной позиционной системы счисления. Особый интерес своей необычностью представляет двадцатеричная систе¬ ма счисления, которой пользовались древние индейские племена майя; учащимся можно сообщить о том, что древние рукописи этих племен, никем ранее не прочитанные, удалось расшифровать со¬ ветским ученым благодаря использованию ЭВМ. Происхождение этих систем счисления связано с тем или иным способом счета по пальцам рук (или и рук и ног). Многочисленные следы систем счис¬ ления сохранились до наших дней в языках народов мира, в систе¬ мах мер, в принятых денежных единицах. Принцип построения позиционных систем счисления хорошо ил¬ люстрируется известной десятичной системой счисления. Следует обратить внимание учащихся на то, что: а) один и тот же знак (цифра) в позиционной системе может выражать различные количества в зависимости от его места в за¬ писи по отношению к запятой (или к правому концу записи числа, если оно — целое); б) позиционная система записи чисел очень удобна и эконо¬ мична для выполнения над ними арифметическихдействий; в) позиционная система счисления удобна не только для за¬ писи чисел на бумаге, но и для механического представления чисел. -Необходимо добиваться от учащихся понимания сущности за¬ писи числа: запись числа в десятичной системе счисления озна-1 чает его представление в виде суммы целых степеней десяти, взя¬ тых с некоторыми коэффидиентами. С этой целью полезно пред- ложить учащимся выполнить следующие упражнения. 1. Найти пятизначное число, каждая цифра которого па еди¬ ницу больше предыдущей, а сумма его цифр равна 30. Р е ш е н и е. Обозначив крайнюю слева цифру искомого числа черёз х, число можно представить в виде: х. 104 + (х + 1) • 103 + (x^+ 2) -10* + (х + 3) ■ 101 + (х + 4) -10°, а тогда из уравнения х + (х + 1) +/(x + 2) + (.х + 3) + 5
4- ($ + 4) = 30 следует, что х = 4 и искомое число имеет вид 45 678. Заметим, что эту задачу можно было бы решить способом пе¬ ребора.. 2. Доказать, что разность любого четырехзначного числа и четырехзначного числа, записанного теми же цифрами, но в обрат¬ ном порядке, делится на 9. Д о к а з а т е л ь с т в о. Можно записать: (а . 103 + b • 102 + с ■ 101 + d * 10°) — (d • 103 + с ■ 102 + + b . 101 + а ■ 10°) = 999с + 90fo — 90c-999d = 9 - (llla + + 10fo — 10с — llld), а отсюда следует, что рассматриваемая раз¬ ность делится на 9. 3. Найти двузначное число, если известно, что сумма его цифр равна 16, а при перестановке цифр число увеличивается на 18. Р е ш е н и е. Поскольку двузначное число можно записать в виде а • 10 + fc, то, решая систему r a + b= 16 1(6. 10 +fl)-(a. 10 + fc) = 18, получим искомое число 79. Выполнение подобных упражнений полезно еще и потому, что здесь находят применение знания, получаемые учащимися на уро¬ ках алгебры. Формированию у учащихся понятия числа как общего свойства всех эквивалентных конечных множеств способствует практический счет определенного числа предметов в системах счисления с различ¬ ными основаниями. Упражнения в счете показывают учащимся недопустимость чтения чисел, записанных не в десятичной системе счисления, как десятичных чисел. Так, число 32 в пятеричной систе¬ ме счисления нельзя читать как «тридцать два», или, что то же са¬ мое, как «три десятка и две единицы», а следует читать: «три пятер¬ ки и две единицы», или,.что проще, «три, два в пятеричной». Число 51728 читают как «пять, один, семь, два в восьмеричной». Полезно выполнить следующее упражнение. Пусть дано число 26,15, записанное в позиционной системе счи- сления с основанием d. Задавая различные значения d, надо найтн величину этого числа, вычисляя его эквивалент в привычной деся¬ тичной системе счисления: а) если число 26,15 десятичное (d = 10), то 26,15io = 2 - 101 + 6 . 10° + 1 * 10-1 + 5 . 10-2 = 26,1510; б) если число 26,15 записано в восьмеричной системе (d = 8), то 26,15fl = 2 ■ 81 + 6 - 8° + 1 - 8-1 + 5 - 8~2 » 22,20310; ч в) если число 26,15 записано в семеричной системе (d — 7), то 26,15, = 2 • 71 + 6 - 7° + 1 . 7-1 + 5 • 7~2 » 20,24510. 6
Следуетааметить, что нельзя предположить, что число 26,15 записано в шестеричной, пятеричной, четверичной и т. д. систе¬ мах счисления, так как среди цифр этого числа есть цифра 6. Учащиеся должны уметь определить, к каким системам счисле¬ ния могут, а к каким не могут относиться, например, следующие числа: 623, 420, 120, 101, 971, 301, 221, 183. Теоретические сведения о непозиционных и позиционных сис¬ темах счисления изложены в пунктах 1 и 2 учебного пособия. На уроках можно выполнить упражнения 1, 2, 3, 5, 8 и решить несколь¬ ко задач в системах счисления с основаниями 3, 4, 5. Для самостоя¬ тельной работы дома учащимся можно предложить упражнения 4, 6, 7, 9. В пункте 3 учебного пособия учащимся сообщается материал, в котором рассматриваются арифметические действия в позицион¬ ных системах счисления. Учащиеся должны усвоить, что арифме¬ тические действия над числами- в любой позиционной системе счис¬ ления производятся по тем же правилам, что и в десятичной системе, поскольку все они построены по общему принципу. При выполне¬ нии арифметических операций в d-ной системе счисления необхо¬ димо всегда помнить, что одна единица каждого старшего разряда количественно равна d единицам соседнего младшего разряда. Ре¬ зультат соответствующей операции над каждым разрядом оценива¬ ют в привычной десятичной системе счисления, затем его переводят в d-ную систему счисления. Основное внимание надо уделить восьмеричной системе счисле¬ ния. В упражнениях достаточно ограничиться четырехзначными восьмеричными числами, добиваясь прочного усвоения сложения и вычитания. Для облегчения выполнения арифметическихдействий можно рекомендовать составить таблицы сложения и умножения одноразрядных чисел в восьмеричной системе счисления, что пред¬ лагается сделать,в упражнениях 13 и 14. На уроке можно выпол¬ нить упражнения 13, 14, 16, а дома — упражнения 12, 15./ ПЕРЕВОД ЧИСЕЛ ИЗ ОДНОЙ СИСТЕМЫ СЧИСЛЕНИЯ B ДРУГУЮ Перевод целых чисел из одной позиционной системы счисления в другую методически целесообразно рассмотреть в следующей по¬ следовательности . 1. Обращается внимание учащихся на то, что запись числа — это представление числа в виде суммы степеней основания с коэффи¬ циентами, меньшими основания. Такое представление использует¬ ся при переводе чисел из недесятичной системы в десятичную. Воз¬ можен и обратный перевод, но при условии выполнения действий в недесятичной системе счисления. 2. Отмечается, что к представлению числа в виде суммы степе¬ ней основания%новой системы счисления с соответствующими ко¬ эффициентами можно прийти с помощью приема, основанного на подборе — сравнении заданного числа с различными степенями 7
нового основания.Приэтом'используется таблица степеней основа¬ ния, а поэтому способ перевода называют табличным. 3. Для алгоритмизации процесса перевода чисел из одной сис¬ темы счисления в другую рассматривается правило перевода целых чисел, основанное практически на той же идее, но позволяющее обойтись без подбора. Например, для записи числа 1691 ю в восьмеричной системе счис¬ ления нужно выполнить последовательно деление: 1691| 8 9 211 I 8 И JH 261_8 3 3 2 3 направление чтения Отсюда следует, что 169110 = 3 • 83 + 2 • 8г 4^ 3 • 81 + Зг- 8° = = 3233g. Правило перевода чисел из одной позиционной системы счис¬ ления в другую можно распространить на любые две системы счис¬ ления при соблюдении условия, что деление выполняется в той си¬ стеме счисления, в которой записано подлежащее преобразованию число. Например, требуется число 111010002 перевести в восьмерич¬ ную систему счисления. Для этого надо данное число последова¬ тельно делить на 8, записанное в двоичной системе, т. е. на 10002. 11101000 | 1000 1101 1110ll 1000 1010 1101 11 1000 101 0 Полученные остатки записываются в новой (восьмеричной) систе¬ ме счисления. Таким образом, 111010002 = 3508. Теоретические сведения 0 переводе целых чисел в различные позиционные системы счисления составляют вторую часть пункта 3 учебного пособия. Они изучаются на одном уроке. Рекомендует¬ ся выполнить упражнения 10, 11, 19, а дома — упражнения 17, 18 и типа 19. При рассмотрении дробей в различных системах счисления сле¬ дует отметить, что обычная запись дроби есть ее представление в виде суммы степеней числа, обратного основанию, с соответствую¬ щими коэффициентами. Но это означает перенесение общего прин¬ ципа записи числа в позиционной системе счисления на дробные числа, а отсюда появляется возможность при переводе их из одной системы в другую провести аналогию с целыми числами. Вывод правила перевода дробных чисел основывается на отыс¬ кании способа выявления долей нового основания в заданной дро¬ би, Пусть, например, требуется десятичную дробь 0,JJ75 обратить e
в восьмеричную. Сначала надо выявить, сколько в этой дроби вссь- мых.Для этого ее делят на —, т. е. выполняютумноженне: 0,175 х 8 x8 = 1,4- Записывают 1 и таким же образом отыскивают, сколько восЬмых в оставшихся 0,4, 0,4 : — = 0,4 • 8 = 3,2, т. е. в данном о (1 2 —j . Записывают 3, а с 0,2 поступают аналогич¬ ным образом. 0,2 : — = 0,2 • 8= 1,6, 8 т. е. данная дробь содержит одну ^j3. Этот процесс можно продол¬ жить и в итоге записать: )' + 3 ' (?)" + * ^ (a"J*+ 4 ’ (^s)* + 6 х X (I)6 + ... = 0,13146...в. Процесс перевода дробей из одной позиционной системы в дру¬ гую может быть как конечным, так и бесконечным. Первоначально надо рассмотреть конечные дроби, например, такие0,62510 = 0,5в; 0,06251О = б,048; 0,12510.= 0,18. При переводе бесконечной дроби из одной системы в другую надо обратить внимание учащихся на встречающуюся иногда возможность выделения периода периодиче¬ ской дроби. р1едует остановиться на округлении чисел. Перед тем как от¬ бросить лишние разряды, к старшему из них добавляется половина единицы последнего сохраняемого разряда. Например, при округле¬ нии восьмеричных чисел к старшему из отбрасываемых разрядов добавляется 4. Полученная сумма после отбрасывания лишних раз¬ рядов дает округленное число. Ниже приводятся примеры перевода правильных десятичных-дробей в восьмеричную систему счисления с округлением до трех знаков после запятой. 0,0210 = 0,012I7...s fn 0,0128; —0,135ю = '—0,10507...e да —0,1058; 0,99910 = 0,7773...в »0,777«; 0|910 =» ,0,7 (1463)8 «0,715*; 0,31О = 0,2(3146)8 ж 0,2328; 0,810 = 0,(6314)g « 0,6328. Рекомендуется перевести в восьмеричную систему счисления несколько смешанных чисел и выполнить с ними арифметические действия. Например, 2456,15ю да4630,1146в; 135,2710 да207,21228. Выполним сложение и вычитание восьмеричных чисел: 4630.11468 + 207,2122g = 5036,3270* да5036,3278; 4630.11468 — 207,2122* = 4420,7024в да4420,703в. 9
Переводом результатов в десятичную систему счисления можно убедиться в их правильности: 5036,327g ^2591,42io; 4420f7038 »2320,881О. Теоретические сведения о переводе дробей из одной системы счисления в другую составляют пункт 4 учебного пособия. Haypo- ках можно выполнить упражнения 21, 22 (частично) и 24, а для ра¬ боты дома рекомендовать упражнения 20, 22, 25. ДВОИЧНАЯ СИСТЕМА СЧИСЛЕНИЯ. ДВОИЧНАЯ АРИФМЕТИКА Следующая система счисления, которая требуетболеедетального расдмотрения, — двоичная. С учащимися полезно выяснить, какое число символов достаточ- но для передачи любого сообщения. Можно привести пример: в аз¬ буке Морзе точкой и тире записывается какой угодно текст. Точку и тире можно заменить другими символами, например нулем и единицей — знаками двоичной системы счисления. Возможностями двоичной системы счисления интересуются давно. Особенно много занимались двоичным счислением с XVI по XVII век. Но тогда ее считали забавой, использовали лишь в составлении и решении го¬ ловоломок. ir Значительное распространение двоичный способ записи чисел получил с появлением электронных вычислительных машин. По¬ ясним почему. Для изображения цифр в ЭВМ применяются элемен¬ ты, способные находиться в одном из нескольких резко разграни¬ ченных состояний. Число таких состояний должно равняться числу цифр системы счисления, применяемой для ввода информации. В этом отношении двоичная система счисления имеет преимущества перед остальными. Другое преимущество двоичной системы состо¬ ит в простоте выполнения арифметических действий, так как она дает возможность упростить конструкцию арифметических ус¬ тройств. Выяснив преимущества двоичной системы счисления для прак¬ тического применения в электронной вычислительной технике, на¬ до предложить учащимся несколько десятичных чисел для перевода их в двоичные. При выполнении арифметических действий в двоич¬ ной системе надо обратить внимание учащихся на сложение, явля¬ ющееся основой выполнения арифметических действий в электрон¬ ной вычислительной машине. Операция сложения двух чисел, представленных в двоичной системе счисления, ведется поразрядно, начинаяснизшего разряда; при сложении,цифр может иметь место один из трех случаев: а) одна цифра 0, другая 1, тогда в разряде суммы будет 1; б) в рассматриваемом разряде обоих чисел стоит 1, тогда в разряде суммы будет 0, а 1 (в уме) переносится в следующий разряд; в) как и в предыдущем случае, в рассматриваемом разряде обоих чисел стоит 1, но, помимо этого, имеется 1 «в уме»; в этом ю
случае в данном разряде суммы появится единица, но и «в уме:> остается 1. Умножение в двоичной системе состоит из сдвигов и сложений. Вычитание и деление в двоичной арифметике выполняется по обыч¬ ным правилам, но в целях упрощения построения электронных схем для выполнения этих действий вычитание заменяют сложе¬ нием: к уменьшаемому прибавляют число, являющееся дополне¬ нием к вычитаемому. Надо показать учащимся правило получения дополнения числа до следующей'высшей разрядноиединицы и применение этого пра¬ вила к выполнению вычитания двух чисел. Например, учащиеся должны понимать, что для вычитания числа 101110 из 10011101 нуж¬ но найти дополнение к вычитаемому: 010001 + 1 = 10010, а затем сумму: 10011101 + 10010 = 10101111, но поскольку дополнение производилось до седьмого разряда, то из седьмого разряда резуль¬ тата надо вычесть единицу и записать ответ: 1101111. Действия в двоичной системе должны быть усвоены учащимися достаточно твердо. В упражнениях рекомендуется рассматривать 12—15-разрядные целые двоичные числа и дроби с 4—6 зиаками после запятой. Теоретические сведения, составляющие содержание пункта 5 учебного пособия, можно рассмотреть на двух уроках. На одном уроке учащимся должен быть сообщен теоретический материал до введения понятия двоичного дополнения. Можно выполнить упраж¬ нения 26, 27, 28 и рекомендовать учащимся для выполнения дома упражнения такого же типа. А на другом уроке надо показать уча¬ щимся способ замены вычитания сложением и выполнить упраж¬ нения типа 29. СМЕШАННЫЕ СИСТЕМЫ СЧИСЛЕНИЯ. АРИФМЕТИЧЕСКИЕ ДЕЙСТВИЯ В ЭЛЕКТРОННОЙ ВЫЧИСЛИТЕЛЬНОЙ МАШИНЕ В ряде случаев числа, заданные в системе счисления с некото¬ рым основанием d, приходится изображать с помощью цифр другой системы счисления с основанием q, где q < d. Тогда используются смешанные системы счисления, в которых каждый коэффициент d-ичного разложения числа записывается в <?-ичной системе; систе¬ ма называется q — d-ичная. Чтобы запись числа в смешанной систе¬ ме была однозначной, необходимо, чтобы для представления любой #-ичной цифры было отведено одно и то же достаточное число раз¬ рядов. Так, в смешанной двоично-десятичной' системе счисления для изображения каждойдесятичной цифры отводится четыре двоич¬ ных разряда (тетрада), а в двоично-восьмеричной системе счисле¬ ния — три двоичных разряда (триада). Особого внимания заслуживает случай, когда d = qfc. Здесь за¬ пись числа в смешанной q — d системе счисления совпадает с изо¬ бражением этого числа в системе счисления с основанием q. if
Смешанные системы счисления описаны в пункте 6 учебного пособия. Их следует рассмотреть на двух уроках. На одном уроке можно познакомить учащихся с двоично-десятичной и с двоично¬ восьмеричной системами счисления и выполнить упражнения типа 30. Полезно принести в класс перфокарты и перфоленты и поупраж¬ нять учащихся в их чтении, т. е. в переводе двоичной информации в восьмеричную, используя двоично-восьмеричную систему счис¬ ления. Для самостоятельной работы дома учащимся можно порекомен¬ довать рассмотреть, например» тро'ично-девятеричную систему счисления. На другом уроке надо рассмотреть двоично-шестнадцатеричную систему счисления и выполнить упражнения 31, 33 (а, б), 34, а упражнения 32, 33 (в, г) рекомендовать для самостоятельной ра¬ боты дома. К занятию на тему «Арифметические действия в электронной вычислительной машине» надо повторить материал из пункта 5, обратив внимание учащихся нато, каквыполняются арифметические операции в двоичной системе счисления. Теоретические сведения пункта 7 учебного пособия могут со¬ ставить содержание одного урока. Учитель должен рассказать о двоичном сумматоре. Здесь следует подробно разобрать все возмож¬ ные значения входов и соответствующие выходы, не касаясь вопро¬ са а техническом осуществлении требуемой зависимости между ними. Последний (двенадцатый) час можно посвятить подведению итогов — контрольной работе или беседе с учащимися о перспек¬ тивах развития вычислительной техники и общения человека с машиной, рассказать учащимся о возможностях дальнейшего знакомства с вычислительными средствами на факультативных занятиях. Для контроля за усвоением изучаемого материала полезно проводить текущие самостоятельные работы, примерное содержа¬ ние которых приводится ниже. Особенность этих работ состоит в том, что учащийся в ходе их выполнения сам может убедиться в правильности результатов. Выбор данных чиеел учитель может предоставить самим учащимся. С А М О С Т О Я Т Е Л ь Н Ы Е Р А Б О Т Ы 1. Сложить два целых четырехзначных восьмеричных числа. Числа и их сумму представить в десятичной системе счисления. 2. Записать два целых четырехзначных десятичных числа. Пе-. ревести их в восьмеричную систему счисления. Выполнить сложе¬ ние и вычитание восьмеричных чисел; полученные результаты пред¬ ставить в десятичной системе счисления. 3. Найти представление двух десятичных дробей с тремя зна¬ ками после запятой в восьмеричной системе счисления. Вычислить 12
сумму и разность полученных восьмеричных дробей и результаты представить Б десятичной системе счисления. ,4- ЦеЛое трехзначное десятичное число пере^сти в восьмерич- нук) систему счисления. Это же число перевести в двоичную систе¬ му счисления, а полученное двоичное число представить в восьме¬ ричной системе счисления. П р и м е ч а н и е. Используется только правило перевода лисел из одной системы счисления в другую. 5.*Записать число в десятичной#системе счисления, имеющее в целой части три и в дробной две одинаковые цифры. По правилам перевода чисел из одной системы счисления в другую представить это число сначала в двоичной, а затеи в восьмеричной системах счисления. Результаты проверить, используя двоично-восьмерич¬ ную систему счисления. 6. СлоЖить два двоичных 15-разрядных числа. Результат сложе¬ ния и данные числа представить в десятичной системе счисления, предварительно переведя их в восьмеричную систему. 7. Число, записанное в десятичной системе счисления и имею* щее в целой части четыре, а в дробной три цифры, представить в двоичной системе счисления, пользуясь предварительным переводом в восьмеричную систему счисления. А затем для проверки пользуясь предварительным переводом в шестнадцатеричную систему счис¬ ления. Ответы к упражнениям 1. 5, 25, 125, 625. 2, 10, 100, 1000. 3. В любой; в восьмеричной. 4. Девятеричная; не существует, одиннадцатеричная. 5. В пяте¬ ричной круглые 5, 10, 15, 20, в семеричной круглые 7, 14, 21, 28, 35, 42. 6. Круглое число должно делиться на 6. 8. Количество единиц в четном числе должно быть четным. 10. 8, 64, 512, 4096. U. 10, 100, 1000, 10000. 12., Таблица сложения в пятеричной системе: 1 ° 1 2 3 4 0 | 0 I 2 3 4 1 1 2 3 4 10 2 2 3 4 10 | 11 3 3 » 4 10 11 | 12 4 4 10 11 12 | 13 13
13, Таблица сложения в восьмеричной системе: 0 1 - 2 3 4 5 6 7 0 0 I zr ^~ 4 5 6 7 1 1 2 з 4 5 6 7 10 2 2 3 4 5 6 7 10 11 3 3 4 5 6 7 10 11 12 4 4 5 6 7. 10 11 12 13 5 5 6 7 10 11 12 13 14 6 6 7 10 II 12 13 14 15 7 7 10 1] 12 13 14 15 16 14. Таблица умножения в восьмеричной системе! 1 2 3 4 5 6 7 1 1 2 3 4 5 6 7 2 2 4 6 10 12 1 I 14 16 3 3 6 11 14 17 22 25 4 4 10 14 20 24 30 34 5 5 12 17 24 31 36 43 6 6 14 22 30 36 44 52 7 7 16 25 34 43 52 61 ДОПОЛНИТЕЛЬНАЯ ЛИТЕРАТУРА Б е н д у к и д з е А. Д. О системах счисления. — Квант, 1975, № 8 с. 59—61. Г у т е р Р. С. Вычислительные машины и системы счисления. — Квант 1971, Кя 9, с. W6. Г утер Р. С и Полунов Ю. JI. Математические машины. М., 1975 Детская энциклопедия, 2-e, 3-е изд., т. 2. Мон а х ов В. М. Программирование. Пособие для учителей. Главы II и IV. М., 1974. Ф о м и и С. В. Системы счисления. Популярные лекции по математике Вып. 40. 3-е изд. М., 1975. 14
СИММЕТРИЯ В современной научной литературе понятие симметрии .встречается довольно часто. Открыв, например, соот¬ ветствующий том БСЭ, мы обнаружим даже не одну, а пять статей «Симметрия» (в математике, физике, химии, кристаллографии, био¬ логии). Это и неудивительно. Симметрию мы наблюдаем в природе — среди растений и живых организмов, молекул и кристаллов. Кро¬ ме того, симметрия используется человеком при строительстве зда¬ ний, различных механизмов, при создании картин и скульптур, стихотворных и музыкальных форм. Соображения симметрии при¬ меняются и как средство классификации объектов, изучаемых той или иной отраслью знания, и как аппарат науки. Таким образом, факультативный курс имеет важное прикладное значение: здесь приводятся многочисленные примеры проявления симметрии в природе, технике и искусстве, изучаются симметрии плоских фигур и на этой основе дается классификация бордюров, орнаментов, решеток, многоугольников и розеток. Изучение курса способствует развитию мировоззрения учащихся: на многочислен¬ ных примерах показано, что абстрактные математические понятия и методы находят практическое применение. В ходе изучения темы учащиеся существенно расширяют свои познания в области математики. С одной стороны, здесь повторяются и углубляются сведения о геометрических преобразованиях; ре¬ шаются задачи, развивающие уменияи навыки, полученные при изучении геометрии. С другой стороны, делается значительный шаг вперед — рассматривается классификация фигур по их симметри¬ ям, вводится понятие группы преобразований, изучаются свой¬ ства группы перемещений плоскости. Получен такой серьезный ре¬ зультат, как теорема о конечных группах перемещений (иногда эту теорему называют теоремой Леонардо да Винчи). Основой темы «Симметрия» являются сведения о перемещениях и симметриях фигур, известные из курса геометрии для VI—VIII классов. Требуется знать определения и основные свойства осевон симметрии, поворота (и, в частности, центральной симметрии), параллельного переноса (см. «Геометрия-6—8», п. 19—21, 36), стро¬ ить образы фигур при этих перемещениях. По-видимому, специально 15
отводить время на повторение соответствующего материала не следует. Более естественно повторить это в ходе изучения теории, а главное — при решении задач, — именно задачи являются осно¬ вой курса. Особо следует остановиться на понятии «композиция перемеще¬ ний». Как известно, композиции рассматриваются в начале второго полугодия VII класса. Поскольку с композицией перемещений часто приходится иметь дело в курсе «Симметрия», лучше начать изучение этого курса во втором полугодии VII класса (ниже изло¬ жение построено таким образом, что понятие композиции подробно рассматривается в самом конце) или в VIII классе. Для удобства изложения в настоящей работе принята рубрика¬ ция, отличная от рубрикации, принятой в пособии «Избранные во¬ просы математики» (разделы, выделенные ниже, охватывают содер¬ жание пунктов пособия, объединенных общей идеей). На изучение факультативного курса отводится 12 ч. Приведем примерный тематический план (в скобках указаны номера пунктов пособия, содержание которых комментируется далее в соответ¬ ствующей рубрике). Симметрия плоских фигур; простейшие задачи (раздел I, п. 1, 2) — 2 ч. Метод геометрических преобразований (раздел I, п. 3) — 3 ч. Симметрия и классификация фигур (раздел I, п. 4—9; раздел И, п. 5) — 4 ч. Композиция перемещений (раздел II, п. 1, 2; а также заключи¬ тельная часть п. I раздела 1*) — 2 ч. Группы геометрических преобразований (раздел II, п. 3—4)**. Зачет — 1 ч. СИММЕТРИЯ ПЛОСКИХ ФИГУР (ПРОСТЕЙШИЕ ЗАДАЧИ) Прежде чем решать задачи методом геометрических преобра¬ зований и давать полную классификацию некоторых видов геомет¬ рических фигур, необходимо сначала научиться решать задачи типа: «Найти все перемещения, которые отображают данную фигуру L на себя». В пособии перемещения, отображающиефигуру L на себя, называют симметриями фигуры L. Поэтому указанную выше задачу можно сформулировать так: «Найти все симметрии фигуры L». Задачи такого типа по существу уже знакомы учащимся. 3 а д а ч а 1. Найдите все симметрии: а) отрезка; б) луча; в) полуплоскости; г) угла; д) окружности; е) треугольника; * Заключительную часть пункта I, содержащую новый для учащихся ма¬ териал об ориентации и классификации перемещений по числу неподвижных то¬ чек (а также задачи 3—5), естественно рассматривать при иэучении темы «Компо¬ зиция перемещений», где эти сведения активно применяются. ** Изучение этой темы всеми учащимися не предполагается. 16
Рис. I ж; квадрата; з) ромба; и) прямой; к) тра¬ пеции; л) полосы (рис. 1). 3 а д а ч а 2. Приведите примеры дру¬ гих фигур, симметриями которых являют¬ ся: а) поворот; б) перенос; в) осевая сим¬ метрия. Важно также привести примеры фигур, обладающих симметрия¬ ми и встречающихся в природе, искусстве, быту и технике (см. п. 2, а также [1], [3]). Для доказательства того, что указанные в ответах к этим зада¬ чам перемещения действительно являются симметриями той или иной фигуры, достаточно, как правило, сослаться на соответствую¬ щее предложение школьного курса. Решение задачи на нахождение всех симметрий полосы приво¬ дит к выводу: существуют перемещения плоскости, не являющиеся осевой симметрией, поворотом или переносом. Действительно, ком¬ позиция симметрии Sa и переноса а, как легко видеть, не относится ни к одному из известных видов перемещений. Естественно такое определение: композиция переноса вдоль прямой а и симметрии Sa называется скользящей симметрией. Оказывается, что других видов перемещений плоскости нет. Без доказательства* формулируется следующая теорема. Т е о р е м а 1 (Ш а л я). Каждое перемещение плоскости есть перенос, поворот, осевая или скользящая симметрия. Приведем задачи на понятие скользящей симметрии. 3 а д а ч а 3. а) Постройте образ данной точки (отрезка, ок¬ ружности, угла) при скользящей симметрии S*. б) Докажите, что скользящая симметрия не зависит от порядка ~% ^* ^> выполнения переноса и симметрии, т. е. Sa = а о Sa = Sa о а. (Для решения задачи достаточно проверить, что образы любой точки —► —► плоскости при перемещениях а о Sa и Sa о а совпадают.) 3 а д а ч а 4. Укажите осевые и скользящие симметрии, при которых квадрат ABCD отображается на квадрат CEFG (рис. 2). [Существуют три скользящие симметрии и одна осевая — sl si si 5,.] Как уже говорилось выше, задача на¬ хождения симметрий фигур знакома уча¬ щимся. Нокеерешениютеперь предъявля¬ ются большие требования: нужно найти все симметрии. В этом отношении полезнатео- рема Шаля. Действительно, если указать отдельно все осевые и скользящие симмет- ч7 £ 4 t ? \ b N^ G 7 Z/ а/А Г ^ D С Доказательство см., например, в [4]. Рис. 2 17
• а An-7 Ап а рии, все повороты и переносы, отображающие фигуру на себя, то получим полный список ее симметрий, —других перемеще¬ ний плоскости нет. Решение зна- А A', А2 А'з А+ д чительно упростится, если при- а менить следующую теорему. Рис. 3 б) Т е о р е м а 2. Каждая симметрия непустой ограни¬ ченной фигуры есть либо осевая симметрия, либо поворот. Д о к а з а т е л ь с т в о. Напомним сначала определение: фигура называется ограниченной, если существует круг, ее содер¬ жащий. Пусть фигура L ограничена. Из теоремы Шаля следует, .что для доказательства теоремы достаточно показать: нетождественный перенос и скользящая симметрия не могут быть симметриями L. Так как L Ф 0, существует точка А { L. Допустим, что неко¬ торый перенос Т отображает L на себя. Тогда точки At = Т (А), Аг — Т (Ai), ... (рис. 3, а) также принадлежат L. Заметим теперь, что Это означает, что фигура L не может быть ограниченной. Дей¬ ствительно, если существует круг (О, R), такой, что L cz Kp(O, R), то расстояние между любыми двумя точками фигуры L должно быть меньше 2R. Из равенства (1) следует, что это невозможно: найдется такое п, что | АА„\ > 2R, т. е. существует такая точка Ап фигуры L, что Л, или А„ i Кр (О, R). Аналогично доказывается, что симметрия ограниченной фигуры не может быть и скользящей симметрией S2 (рис. 3, б): \ААгп\ = —> = 2п ]а|, где п — любое натуральное число. Из теоремы 2, которую полезно доказать в классе, вытекает, что для ограниченных фигур, указывая их симметрии, достаточно перечислить все повороты и осевые симметрии, отображающие их на себя. В то же время/находя симметрии неограниченных фигур, требуется выяснить также, существуют ли переносы и скользя¬ щие симметрии. Так, симметриями прямой р являются не только осевыеицент- ральные симметрии (ось — любая прямая, перпендикулярная р, а также сама прямая р\ центр — любая точка р), но и переносы АВ (в частности, тождественное преобразование), а также скользящие c*AB симметрии Sp , где А и В — произвольные точки р. В связи с тем что примерами фигур, симметриями которых яв¬ ляются переносы и скользящие симметрии, могут быть.только не¬ \АА„| = п \AAi\ >0. (1) 18
ограниченные фигуры, полезно уже в самом начале решать неслож¬ ные задачи на нахождение симметрий бордюров и решеток (см. п. 7 и 8), не требуя пока перечисления всех симметрий. На изучение темы отводится 2 ч. На первом уроке следует вве¬ сти понятие симметрии фигуры и скользящей симметрии, а также повторить основные свойства перемещений. На втором уроке ре¬ шаются задачи из п. 1 и 2. МЕТОД ГЕОМЕТРИЧЕСКИХ ПРЕОБРАЗОВАНИЙ Как уже говорилось выше, повторение основных свойств пере¬ мещений в курсе «Симметрия» осуществляется в ходе решения за¬ дач. В этом отношении важное место занимает пункт «Симметрии и геометрические задачи». При разборе примеров, приведенных в этом пункте, необходимо напомнить свойства перемещений, а глав¬ ное — выделить основные приемы применения метода геометриче¬ ских преобразований. Так, разобрав примеры 4 и 5, полезно сформулировать следую¬ щие предложений, помогающие при доказательстве конгруэнтности фигур (равенства длин отрезков, величин углов), перпендикуляр¬ ности прямых, принадлежности трех точек одной прямой, а также доказательства того, что три прямые проходят через одну точку. Т е о р е м а 1. Если пережщение отображает фигуру Lt на Lz, mo L2 ^ Lj. Т е о р е м а 2. Если две точки Р и Q симметричны относи¬ тельно точки 0, то три точки Р, Q и 0 лежат на одной прямой. Т е о р е м а 3. Если две точкиР и Q симметричны относитель¬ но прямой 1, то (PQ) X 1• Т е о р е м а 4. Если прямые /* и 12 центрально симметричны, то они параллельны. Эти предложения доказываются в курсе геометрии. Рассмотрим еще одну теорему. Т е о р е м а 5. Если две прямые lt и 12 симметричны относи¬ тельно прямой 1, то они либо параллельны 1, либо пересекаются в точке прямой 1. Действительно, пусть две симметричные относительно 1 прямые U и /2 пересекаются в точке 0. Так как S, (lt) = h и 0 6 /i, то S, (0) £ 6 1%. Аналогично показывается, что S( (0) € h~ Значит, S( (0) — точка пересечения 1и /2 и 5t (0) — 0. Так как при осевой симметрии неподвижны только точкц оси, то 0 £ 1, что и требовалось доказать. Решение задач, основанных на применении этих утверждений, полезно начать с рисунка. Как правило, рисунок подсказывает, какое перемещение следует рассмотреть, 8атем, пользуясь условием и извеСтными свойствами переме¬ щений, надо доказать, что «подозреваемое» перёмещение действи¬ тельно отображает интересующие нас точки и фигуры на те, о ко¬ торых в задаче требуется что-то узнать. Окончательный вывод часто ю
удается получить на основании сформулированных только что утверждений. На изучение темы отводится 3 ч. Первые два можно посвятить подробному разбору примеров и упражнений, имеющихся в п. 3. Важно,' чтобы идеи метода геометрических преобразований сразу же подкреплялись разбором упражнений, в ходе решения которых эти идеи применяются, причем желательно, чтобьГ эти решения на¬ ходили сами учащиеся. На третьем уроке решаются задачи из до¬ машнего задания.' СИММЕТРИЯ И КЛАССИФИКАЦИЯ ФИГУР Идея классификации фигур в зависимости от количества их симметрий (п. 4) поясняется сначала на примерах известных фигур. Так, при рассмотрении разбиения треугольников, при котором в один класс попадают треугольники, обладающие одинаковой сим¬ метрией, выясняется, что эта классификация совпадает с уже из¬ вестной классификацией (разносторонние, равнобедренные, не являющиеся равносторонними, и равносторонние треугольники). Естественность выделения классов симметрий подтверждается и для четырехугольников. Проведем исследование классов симмет¬ рии четырехугольников по несколько упрощенной по сравнению с пособием схеме. Заметим, что при любом перемещении, отображающем на себя четырехугольник, его диагонали могут отображаться только на диагонали (это следует из предложения а), с. 44). Рассмотрим два случая. 1) Длины диагоналей -АС и BD четырехугольника ABCD раз¬ личны. В этом случае каждая диагональ при симметрии четырехуголь¬ ника может отображаться только на себя. Как известно, существуют лишь четыре симметрии отрезка АС — Е, 1м (М — середина [ЛС]), Sp (р — серединный перпендикуляр отрезка АС) и 5(ЛС). В зависи¬ мости от того, какие из этих перемещений являются симметриями четырехугольника ABCD, выделим классы симметрий. а) Один из классов образуют произвольные четырехугольники (выпуклые и невыпуклые), единственной симметрией которых явля¬ ется тождественное преобразование Е. * б) Из. свойств центральной симметрии сразу получаем, что че¬ тырехугольники, симметриями которых являются только Е и ZM, параллелограммы, не являющиеся прямоугольниками или ром¬ бами. в) К классу {E, S4C) относятся дельтоиды. г) К классу {E, ZM, SAC, Sp) относятся ромбы. Других классов в случае 1 мы не получим: если, наприМер, ZM и SAC — симметрии четырехугольника ABCD, то и Sp = — ZM° SAC также является симметрией ABCD, т. е. этот четырех- 20
угольник относится к уже рассмотренному классу симметрии (см. случай r)). 2) Диагонали АС и BD имеют равные длины. Случай, когда при симметрии четырех¬ угольника ABCD каждая из его диагона¬ лей отображается только на себя, уже рас¬ сматривался. Для того чтобы получить но¬ вые классы симметрии, необходимо рас¬ смотреть четырехугольники, для которых существуют симмет¬ рии, отображающие диагонали АС и BD друг на друга. Это воз¬ можно лишь в том случае, когда отрезки, отсекаемые на любой диа¬ гонали другой его диагональю, конгруэнтны (при любой симметрии четырехугольника точка пересечения диагоналей отображается на себя). а) Если точка М не является общей серединой отрезков АС и BD, то тройка точек М, A,D должнаотображаться на себя и, зна¬ чит, существуют лишь две такие симметрии (предложение б) — с. 44), которые легко найти: это Sp и Е (рис. 4). Из симметричности точек А и D, С и В сразу получаем, что к этому классу относятся равнобедренные трапеции. б) Если |ЛЛ1| — \ВМ\ = |C7W| = |DAf|, а прямые АС и BD не перпендикулярны, получаем четырехугольники, классы симмет¬ рии которых таковы: {£, ZM, SQf Sp}. Это прямоугольники. в) Если, наконец, диагонали АС и BD в точке пересечения де¬ лятся пополам и (ЛС) _L (BD), получаем квадраты, множество сим¬ метрий которых состоит из 8 известных элементов. Таким образом, рассматривая классы симметрии четырехуголь¬ ников, мы получаем известную ранее классификацию (отметим толь¬ ко, что прямоугольники и ромбы, дельтоиды и равнобедренныетра- пеции попадают в один класс). Это подсказывает нам, что и длядру- гих типов фигур удобно проводить классификацию в соответствии с их классами симметрии. Далее это делается для правильных мно¬ гоугольников, розеток, бордюров, решеток и орнаментов. В четы¬ рех случаях целесообразность такой классификации особенно по¬ нятна: иного способа найти общность между бордюрами, орнамен¬ тами и розетками,гимеющими самые разнообразные изображения, нет. Точное определение фигур, относящихся к одному классу сим¬ метрии, в пособии не приводится (см. пояснительный текст на с 42). Можнодать такоеопределение: фигуры Ф5 и Ф2 относятся к одному классу симметрии, если существует такое преобразование подобия (для ограниченных фигур достаточно рассматривать только переме¬ щения), что оси симметрии и центры поворотов фигуры Ф2 и обра¬ за Ф{ фигуры Ф4 при этом преобразовании совпадают, а каждое перемещение, отображающее одну из этих фигур на себя, отображает на себя и другую. Это определение не предназначено для учеников. 21
<9 6) i, 1 h а \ ^zY й\ LLLL а) rn_r б) NNNN б) DODD г) vvvv д) VAVA е) нннн т) Рис. 6 От учащихся требуется толь¬ ко в каждом конкретном слу¬ чае уметь ответить навопрос, относятся ли данные фигуры к одному классу симметрии, относятся ли они к классу Zn и Dn (эти обозначения вводят¬ ся ради сокращения записи). Полезно показать естест¬ венность введения понятия «фундаментальная область фи¬ гуры относительно Zn и Dn»: фундаментальная область фи¬ гуры Ф — это такая «наимень¬ шая» часть фигуры Ф, зная которую можно восстановить фигуру Ф по заданным осям симметрии и центрам поворо¬ тов, отображающихФнасебя. Основные требования, предъявляемые к учащимся при изучении п. 4—9, таковы: требуется знать определения правильных многоугольников и уметь доказывать, что пра¬ вильные многоугольники от¬ носятся к классу Dn\ уметь выделить фундаментальные области относительно Zn и Dn для простейших фигур; знать, что существует лишь семь ти¬ пов бордюров* и пять типов решеток (см. с. 59 и 62 посо¬ бия); уметь указывать на ри¬ сунках все эти типы и пере¬ числить все их симметрии. При изучении пунктов 4—9 необходимо постоянно * На рисунке 5 изображены бордюры всех этих типов и пока¬ заны оси и центры симметрии. На рисунке 6 даны символические изображения этих же типов бордю¬ ров. Указывая все симметрии, следует напомнить, что в случаях б), е) и ж) имеются скользящие симметрии. 22
обращаться к примерам фигур, встречающихся в природе, искус¬ стве, технике, описывать их симметрии. Полезно проводить прак¬ тические работы — вырезая или изображая розетки заданной сим¬ метрии и т. д. На изучение темы отводится 4 ч. На первых двух уроках разби¬ рается материал п. 4 и 5, на третьем и четвертом — п. 7—9. КОМПОЗИЦИЯ ПЕРЕМЕЩЕНИЙ ронятие «композиция» рассматривается в заключительной части пункта 1, а также В пунктах «Композиция геометрических преобра¬ зований» и «Алгебра преобразований». Соответствующие понятия и доказательства являются новыми для учащихся, поэтому возмож¬ ную последовательность изложения материала и основные методы решения задач следует рассмотреть подробнее. Задачи на нахождение композиции заданных перемещений в кур¬ се «Симметрия» имеютдвоякое назначение. С одной стороны, они подготавливают учащихся к введению понятия «группа преобразо¬ ваний» и доказательству теоремы Леонардо да Винчи (с. 70—74 пособия), С другой стороны, эти задачи имеют и самостоятельное значение: их решение требует основательного знания свойств пере¬ мещений ri привлечения новых идей. Рассмотрение композиции перемещений существенно расширяет список задач, решаемых мето¬ дом геометрических преобразований. Сначала следует напомнить, что такое композиция преобразо¬ ваний (см. с, 63 пособия), а также следующие предложения, из¬ вестные учащимся из курса геометрии. Т е о р е м а 1. а) Композиция перемещений есть перемещение. б) Отображение, обратное перемещению, есть перемещение. Контрольными вопросами могут служить задача 78, а также сле¬ дующая задача. 3 а д а ч а 1. Докажите, что если Fi и F2 — симметрии фигуры Ф, то F2 ° Fi и F~~\ также являются симметриями Ф. (Этот результат применяется при решении упражнений 90—96.) Т е о р е м а 2. Композиция переносов АВ и ВС — перенос AC, причем АВ ° ВС = ВС о АВ = АС. Т е о р е м а 3. Композиция симметрий Sa и Sbt оси KomopbLx перпендикулярны, есть центральная симметрия ZOJ где 0 — тонка пересечения прямых а и b; композиция Sp о Sp есть тождественное преобразование. Т е о р е м а 4. Любое перемещение, отображающее каждый луч плоскости на сонаправленный ему луч, является параллельным пере¬ носом. Большинство задач на «композицию перемещений», приведенных в статье «Симметрия», относится к задачам типа: «Найти вид переме¬ щения, являющегося композицией двух данных перемещений Ft и F2, и указать элементы, задающие перемещение F2 0 Fх (если, 23
Рие. 7 например, известно, что F2 0 Ft — пово¬ рот, надо указать центр, угол и направ¬ ление этого поворота). Итогом решения таких задач является «таблица умножения перемещений», т. е. таблица, в которой указаны виды компо¬ зиций двух перемещений Л и Fz в зависи¬ мости от вида Fj и F2- Так как по теореме Шаля существуют всего четыре вида пере¬ мещений плоскости, для заполнения этой таблицы следуетрешить 16 задач (сущест¬ венно разных задач меньше, так как за¬ дачи на определение вида симметрии Fi о Fz и Fz o Fi отличаются незначитель¬ но). Ниже приведены решения некоторых из них и указаны методы решения, которые помогут в случае, ес¬ ли учитель сочтет возможным решить все эти задачи, найти реше¬ ния остальных (см. также п. 8 пособия). При этом существенную помощь окажет таблица, приведенная на странице 35 пособия. Полезно, чтобы эту таблицу заполнили самн учащиеся. Естественно, что перед этим необходимо дать определе¬ ние неподвижной точки (точка X называется неподвижной точкой отображения F, если F (X) = X). Для того чтобы ознакомить уча- щихсяс понятием ориентации, следует рассмотреть рисунки 2—5 по¬ собия, а также рисунок 7, где показано, что скользящая симмет¬ рия меняет ориентацию. Необходимо также выделить следующие интуитивно ясные пред¬ ложения: 1) композиция двух перемещений, каждое из которых со¬ храняет {меняет) ориентацию, есть перемеьцение, сохраняющее ориентацию; 2) композиция двух перемещений, одно из которых со¬ храняет ориентацию, а другое ее меняет, есть перемещение, ме¬ няющее ориентацию. Полезно дополнить таблицу (с. 35 пособия) указанием об уртах между лучами плоскости и их образами: угол между любым лучом плоскости и его образом постоянен только при поворотах R% и равен а. Соображения, связанные с углами, часто применяются далее. Отметим, наконец, следующее предложение, которое также по¬ лезно сформулировать и доказать (см. с. 65 пособия). Т е о р е м а 5. Композиция перемещений ассоциативна: для любых перемеирний Fu F2 и F3 Fi о (F2 -=> Fs) = (Fi ° Fz) ° Fa- Эта теорема позволяет оперировать с перемещениями почти так¬ же, как с произведениями чисел. 3 а д а ч а 2. Докажите, что композиция двух центральных симметрий Z0j и Z0> есть перенос. 24
Рис. 8 Рис. 9 Р е ш е н и е. Любая центральная симметрия отображает каж¬ дый луч плоскости на противоположно направленный ему луч. Поэтому композиция симметрий Z0 и ZQ отображает каждый луч на сонаправленный ему луч. По теореме 4 имеем: Z0 о Z0 и Z0 ° о 10 — переносы. Для того чтобы задать перенос, достаточно ука¬ зать точку и ее образ. Проще всего найти образ точки Oj. В итоге получаем: h, ° Zo, = 2<5xOj, Z0< о Z0^ = 2020i. Отметим, что Z0 о ZQ ф ZQ о ZQ . 3 а д а ч а 3. Найдите композицию двух осевых симметрий 5в и S*. Р е ш е н и е. Так как Sb о Sa сохраняет ориентацию, то Sb о о Sa есть поворот или перенос. (Вообще соображения, связанные с ориентацией, сразу позволяют определИть, что данное перемещение является осевой или скользящей симметрией, поворотом или пере¬ носом. Но для того чтобы выяснить, какая из двух возможностей реализуется, в каждом конкретном случае требуются дополнитель¬ ные соображения.) а) Ёсли а || b, то Sb о Sa — перенос. Действительно, проведем прямую 1, перпендикулярную а и b. Пусть 1 f| я = Ои 1 f| b = = 02 (рис. 8). Воспользуемся ассоциативностью композиции и тео¬ ремой 2. ^b 0 ^a = *^ft ° & 0 ^a ~ $b ° (^1 ° Sj)^ Sa = (Sb о Sf) а о (S( о Sa) = Z0>° Z0t = 20j0j. Аналогично доказывается, что Sa о Sb = 20*0j. б) Если а П b = О, то Sb о Sa — поворот с центром О. Действи¬ тельно, точка 0 — неподвижная точка Sb о 5а; угол между про¬ извольным лучом 0X и его образом OY равен 2a, где a — меньший из четырех углов, образованных прямыми а и b (рис. 9). Заметим, что Sb о Sa и Sa о Sb — повороты с общим центром 0 на один и тот же угол a, но в противоположных направлениях. 3 а м е ч а н и е. Из результата этой задачи сразу получаем, что всякий поворот Ro представим в виде композиции двух симметрий, осикоторых проходят через 0 (см. № 74 пособия) и одна из них — 25
произвольная прямая /1# адругая — прямая /2, проходящая через 0 и образующая угол ^- с прямой 1У (существуют две такие прямые, но направление Rg задает прямую / однозначно). Аналогично каж¬ дый перенос представим в виде композиции двух осевых симметрий (см. № 75 пособия), оси которых перпендикулярны направлению переноса, а расстояние между ними равно удвоенному расстоянию между прямыми /х и 12. 3 а д а ч а 4. Найдите композицию поворотов Ro[ и Ro\. Р е ш е н и е. Композиция Я“2 о ^g;: 1) сохраняет ориента¬ цию; 2) отображает каждый луч ОХ плоскости на луч OF, образую¬ щий угол ах + а2 с лучом ОХ. Следовательно, если ах + а2 Ф Ф 360°, то Rol о Ro[ — поворот на угол аг + а2;если ах + а2 = = 360°, то Ro\ о Roi — перенос. Чтобы задать этот поворот (или перенос), можно ,воспользоваться решением, данным в пособии (с. 63). Дадим иное решение. Повороты Rfy и R%* можно представить в виде: R^t = 5/ о 5Д, R%* = Sb о 5/ (если Ох Ф 02, то 1 — (OiOg), если 0 = Ох = 02, то 1 — произволь¬ ная прямая, проходящая через О). Тогда ^oJ ° Ro\ = (Sb ° $l) 0 (^/ ° SJ = $b ° (S/ ° S[) ° Sa = Sb О Е о ° $a — $b ° $a' Построив прямые а и b, легко найти точку их пересечения при ai + а2 Ф 360° или задать результирующий перенос, если щ + + а2 = 360° (см. задачу 3). 3 а д а ч а 5 (см. № 63 пособия). Найдите композицию пере- —> носа а и осевой симметрии Sf, Р е ш е н и е. Композиция переноса а и симметрии S, меняет ориентацию и, следовательно, является осевой или скользящей симметрией. Рассмотрим несколько случаев. ^> —> 1) Если а II /, то 5; о а — скользящая симметрия. При а = 0 имеехм S, о а — осевая симметрия. —► 2) Пусть а J_ 1. Рассмотрим прямые 1г и /2, цараллельные /, отстоящие на расстояние ^ от этой прямой и лежащие по разные стороны от 1 (рис* 10). Нетрудно понять, что а о S/ (X) = X для любой точки X € /i, а так как а о S, меняет ориентацию, то полу¬ чаем, что а о S[ — симметрия с осью /х. Аналогично St о а — сим¬ метрия с осью /2. 26
X а Рис. 10 Рис. 11 3) Пусть теперь направление переноса а не перпендикулярно и ^. ^ ^- ^ —*■ не параллельно прямой 1. Тогда а = at + ап, где at || 1, ап L 1 (рис. II), имеем: — V —> ^> —^ —► —>- S[ о Cl — S[ о (йд ° й|) = (S^ о fl^) о Clf = 5/а о Cl[* (St о ап = S/2, так как ад i. /, см. случай 2.) Но /2 II fy> поэтому S/ о а — скользящая симметрия. Аналогично —>* ^ ^> * > —^ —^ а о S, = (ая о а() о S[ = а, о (а„ о S{) = at о Sh. »>■ ^> Отметим, что а о Sz ф St о а. ГРУППЫ ГЕОМЕТРИЧЕСКИХ ПРЕОБРАЗОВАНИЙ Понятию «группа преобразований» и доказательству теоремы Леонардо да Винчи посвящены пункты 3 и 4 второго раздела. Это необязательный материал, который можно рекомендовать для само- стоятельного чтения. В случае, если учитель сочтет возможным вы¬ делить дополнительное время на изучение этой темы, для доказа¬ тельства теоремы о конечных группах преобразований можно по¬ ручить нескольким учащимся подготовить доклад. Знакомство с теоремами о композиции различных перемещений (см. предыдущий пункт) существенно упрощает эту задачу. Перед докладом необходимо дать определение группы и разобрать соот¬ ветствующие упражнения. ПРИМЕРНОЕ СОДЕРЖАНИЕ ЗАЧЕТА По выбору учителя зачетная работа может проводиться письмен* но или устно по заранее подготовленным вопросам или билетам. Желательно, чтобы в ходе проведения зачета каждому учащемуся были заданы вопросы следующих трех основных типов. 1. Решить задачу с применением метода геометрических пре¬ образований. Упражнение такого рода может быть выбрано из задач 13—30 (труднее задачи 68—73) или задач,, содержащихся в разделах «До¬ полнительные упражнения к главе» учебника «Геометрия-6—8». 2. Найти все симметрии ограниченной фигуры; указать, к ка¬ кому классу симметрий (Zn или DJ относится этафигура; выделить фундаментальную область этой фигуры (относительно Zn или Dn). 27
Примерами фигур, заданных в каждом конкретном случае, мо¬ гут служить фигуры, указанные в упражнениях 7, 8 а), б), 53—57. Для более сильных учащихся можно предложить задачи на указа¬ ние всех симметрий бордюров, заданных в пункте 8, решеток (п. 9). 3. Каким перемещением является композиция данных переме¬ щений Ft и F2? В качестве Fx и F2 могут быть заданы осевая симметрия и пере- нос, две осевые симметрии с параллельными (пересекающимися) осями и т. д. Ответы, решения и указания к упражнениям 7. При нетождественной симметрии, отображающей изображение двузначного числа на себя, каждая его цифра может отображаться либо на себя, либо на другую цифру этого числа. Но единствен¬ ными цифрами, обладающими нетождественными симметриями, являются 0 и 8, а едйнственная пара- конгруэнтных цифр — это 6 и 9. Так как цифры 6 и 9 могут входить в искомое число лишь одно¬ временно, получаем два числа, обладающих центральной симмет¬ рией, — это 69 и 96. Из цифр 0 и 8 также можно составить два сим¬ метричных числа — 88 и 80. Число 80 обладает горизонтальной осью симметрии, 88 — горизонтальной и вертикальной осью, а также центром симметрии, 08 не является записью двузначного числа. 8. в) Решетка отображается на себя при параллельных пере¬ носах al± + blZl где 1г и /2 — единичные векторы, заданные на осях координат, а и b — произвольные целые числа. Середины сторон единичных квадратов (ячеек решетки) являются центрами симмет¬ рии второго порядка, а узлы решетки — центрами симметрии чет¬ вертого порядка (см. с. 145 учебного пособия «Геометрия-6—8»). Осями симметрии решетки являются прямые вида х = а и у = fe, а также прямые вида у = dox + а (а и b — произвольные целые числа). Наконец, любая такая прямая и прямые, проходящие через середины смежных сторон единичных квадратов решетки, являют¬ ся осями скользящих симметрий, при которых решетка отобража¬ ется на себя. 9. Задача ^становится интереснее, если попросить учащихся назвать возможно больше симметричных' «слов». Предварительно требуется выделить буквы, обладающие той или иной симметрией, а также пары конгруэнтных букв (начертание алфавита, при кото¬ ром буквы обладают возможно большей симметрией, оставаясь при этом похожими иа знакомые буквы, приведено в «Геометрии-6—8», с. 51), Горизонтальной осью симметрии обладают буквы В, Е, Ж» 3, К, Н, X, О, С, Ю, Э; вертикальной осью — буквы А, Д, Ж* М, Н, 0, П, Т, Ф, X, Ш; центральносимметричныбуквы Ж! И, Н, О, X. Наконец, единственной парой конгруэнтных различных букв яв¬ ляется пара Р, Ь. Отметим также, что слово, обладающее горизон¬ тальной осью симметрии, должно состоять из симметричных относи¬ тельно этой оси букв. Подбирая из букв первой группы слова (ос¬ 28
мысленные!), можно получить, например, такие: а) СОК, ЗВОН, КОН, ОКНО, НЕЖЕН. Аналогичные соображения по¬ зволяют в случаях б) и в) придумать, например, такие слова: б) ТОПОТ, ПОТОП, АННА, в) ОНО. 11. Прирешениизадач на нахождение всех симметрий фигуры L полезно вначале обратить внимание на некоторые особен¬ ности (это помогает «не потерять» симмет¬ рий L). а) При любой симметрии отрез¬ ка концы его должны отображаться на се¬ бя. Поэтому осевые симметрии находятся однозначно: либо концы А и В отобража¬ ются друг на друга (осью симметрии в этом случае может быть только середин¬ ный перпендикуляр к этому отрезку), либо каждый на себя (получаем ось {AB)). При этом середина М отрезкадолжна отображаться на себя, поэтому центром поворота, отображающего отрезок на себя, может быть лишь точка М. Угол поворота легко определить — это либо 0°, либо 180°. Из теоремы 2 следует, что дру¬ гих симметрий нет. г) Окружности, ограничивающие данное коль¬ цо, при любой его симметрии должны отображаться на себя — по¬ этому на себя должен отображаться общий центр 0 окружностей. Следовательно, искомыми симметриями могут быть только поворо¬ ты с центром О или симметрии, оси которых содержат точку О. Из курса геометрии известно, что каждое из этих перемещений дей¬ ствительно является симметрией этих окружностей, а значит, и симметрией кольца. 12. НаПример, фигура, изображенная на рисунке 12, имеет три симметрии — повороты с центром О на углы 0°, 120°, 240°. 13. Нетрудно заметить (рис. 13), что интересующие нас отрезки 'MB и НА симметричны относительно прямой 1 — серединного пер¬ пендикуляра к отрезку AB. Действительно, 1 — ось симметрии рав- si нобедренной трапеции, поэтому [AD] ~>-[5C], и равносторонний треугольник с основанием AD при симметрии St отображается на равносторонний треугольник с основанием ВС, лежащий вне трапе¬ ции. Отсюда получаем, что St (Ж) = Н. Итак, точки М и Н, А и.В, С и D симмет- ричны относительно 1. Вследствиетеоремы 1 \МВ\ — \ НА I, |МС |= \HD\. Вследствие яеоремы 3 {MH) _L 1, {AB) _L 1 и, значит, {MH) || {AB). 14—15. Для решения задач 14 и 15 то¬ же следует рассмотреть симметрию S, Рис. 20
Рис. 15 Рис. 16 16. Точки М и Н пересечения окружностей (0и гх) и (02, г2) симметричны относительно линии центров этих окружностей, 5(^)(71) — Т (рис. 14). Вследствие теоремы 1 JLMTO^_ ^ L,HTO^. 'l7. Отрезки МН и РТ симметричны относительно прямой 1, содержащей биссектрису угла АВС (рис. 15). Действительно, при симметрии S, окружность (0, r) отображается на себя (0€ 1), а стороны угла АВС отображаются друг на друга. Следовательно, точки пересечения окружности (0, r) со стороной угла АВС отоб¬ ражаются на точки пересечения этой окружности с другой сто¬ роной, т. е. точки М и Р, Н и Т симметричны относительно 1. Поэтому [iWtf] ga [P7"] fl {MP) _L/, {HT)Ll. Значит, {MP) || {HT). 18. Сначала сделаем общее замечание. Если 1 — касательная к окружности (О, r), а /х и Окр (0', r) — образы этой прямой и окружности при перемещении, то h и Окр (0\ г) также будут иметь одну общую точку, т. е. 1г — касательная к Окр (0', r). Так как при симметрии St точка Т, а также окружности (0i, гх) и (02, г2) отображаются на себя, то получаем, что точки А и В, С и D (рис. 16) симметричны относительно 1. По теореме 5 прямые АС и BD, AD и ВС пересекаются (если rt Ф г2) в точках М и N прямой 1. Приведенные в пункте 3 примеры 1—3 иллюстрируют и другую идею применения метода геометрических преобразований: рассмот¬ рение перемещения часто позволяет получить дополнительные све¬ дения для решения задачи илисуществен- но упростить ситуацию, как это удалось сделать при рассмотрении примера 1. 19. Допустим, что' задача решена (рис. 17). Так как прямая 1 — ось сим¬ метрии квадрата PQRS, то точки Р и R сим¬ метричны относительно 1 и, значит, образ угла АВС при этой симметрии содержит точки Р и R. Заметим теперь, что угол A,B,C, является образом угла АВС при симметрии S,, поэтому его легко построить. Следовательно, можно построить и вер- Рис- 17 шины Р = [BA) П [SA), R = [BC) П 80
П [Bj^i) искомого квадрата. Дальнейшее решение очевидно. 20. Допустим, чтоточка Т — искомая, т. e. Z.1 ^ / 2. Рассмот¬ рим окружность {0\, Гх), симметричную Окр (0lt rt) относительно 1 (рис. 18). Тогда £А ^ Z.3 и, значит, /_2 ^ Z.3. Это означает, что точки A' и В касания 1 с Окр (Ol, гх) и Окр (02, г2) лежат на одной прямой с точкой Т. Следовательно, для решения задачи достаточно построить общую касательную окружностей (0\, fj) и (02, г2) и най¬ ти точку пересечения этой касательной с 1. Точка пересечения об¬ щей касательной Окр (0Х, г±) и Окр (02, r2) с прямой 1 тоже является решением. 21. Если а = d и b = с, решений бесконечное множество. Если а = d и b Ф с (или а ф d, но b = с), решений нет. Рассмотрим слу¬ чай, когда а Ф d и b Ф с. Пусть точка Е симметрична точке В от¬ носительно прямой АС (рис. 19). Тогда \АВ\= \АЕ\ =a, \ВС\ = = ICEI = 6, причем Е 6[AD], так как по условию Z_I ^Z_2. Треугольник ECD легко построить по трем сторонам (| CD\ = с, | СЕ | = b, | DE | = | d — а |). Затем строится точка А луча DE (| DA |= = d) и точка В — точка пересечения Окр (Л, о) и Окр (С, b). 22. (См. также № 97.) Напомним сначала, что после удара о борт шар двигается по закону: угол отражения равен углу паде¬ ния. Это замечание позволяет строить траекторию движения шара, зная его начальные положение и направление. Пусть шар из точки М попал в точку Ог борта /х и Мг = St, (M). Тогда 2л ^ Z_2, Z_3 ^ Z_4 и, значит, дальнейшее движение происходит по прямой МгО, (рис. 20, а). Аналогично получаем, что затем шар двигается по прямымМ202, М303, М404 ... (здесь Мг= S!t(M), М2 = 5^(Mx), М3 = 5/з Ш2)). По условию после удара о борт /4 шар должен попасть в точку N. Для того чтобы это произошло, надо, чтобы удар о борт /4 произошел в точке К={М4М) f) /4. Этотребова- ние легко восстанавливает интересующую нас траекторию (рис. 20, б), так как точки М1г М2, М3 и Mk легко строятся. С .D с Рис. 18 Рис. 19 31
23. Рассмотримточки P'.= S(Ba (P) и Р* = 5{^В) (P) (рис. 21). Пусть L н К — произвольные точки на сторонах угла АВС. Тогда |Р"К\ — \РК\, \Р'Ц — \PL\ и, значит, периметр треуголь¬ ника PLK равен длине ломаной PLKP". По теореме о длине лома¬ ной периметрискомоготреугольника не меньшедлины отрезка P'P". Поэтому решением задачи служит треугольник PL0Ko, где L0 и /Со — точки пересечения (P'P") со сторонами угла АВС. 24. Возьмем произвольную точку Р на стороне ЛС, и пусть P' = S{AB)(P), P"=S(BC5 (P). В силу результата предыдущей задачи периметр треугольника PL0Ko (рис. 22) меньше периметра любого 32 Рис. 20
Рис. 21 Рис. 22 другого треугольника, имеющего вершину Р и вписанного в тре¬ угольник АВС, причем периметр треугольника PL0K0 равен \Р'Р"\. Поэтому, найдя минимальное возможное значение длины \Р'Р\, мы найдем минимальное значение периметра искомоготреугольника. Заметив, что отрезки Р'В и PB, P"B и РВ симметричны относитель¬ но прямых АВ и ВС соответственно, получим: \Р'В\ = \ВР\ = \ВР"\, P*BP" = 2^. Следовательно, \Р'Р"\ — 2\РВ \ stn В] Так как Z^— величина постоянная, то отсюда вытекает, что минимальное значение \Р'Р"\ достигается при минимуме значения |РВ|. Этот минимум — длина высоты треугольника АВС, проведенной из вершины В (Р € [ЛС]). Итак, одна из вершин искомого треугольника — основание высо¬ ты. Аналогично доказывается, что два других основания высот — другие вершины искомого треугольника. 25. Указание. Пользуясь теоремой о центральной симметрично¬ сти параллелограмма, легко доказать, что точки Ai и D.lf Bt и Сг симметричны относительно центра параллелограмма ABCD. 26. Условие «Параллелограмм AjB^Di вписан в параллело- грамм ABCD» означает, что точки Ai, Blt Сг и Dt — внутренние точ¬ ки сторон параллелограмма ABCD. В этом случае нетрудно показать, что существует лишь один отрезок с концами на сторонах ABCD, конгруэнтный и параллельный данному отрезку, концы которого также лежат на сторонах параллелограмма ABCD. Остается за¬ метить, что эти отрезки симметричны относительно центра парал¬ лелограмма ABCD. 27. Указание. Центр окружности, описанной около треуголь¬ ника, отображается при перемещении F на центр окружности, опи¬ санной около образа этого треугольника прй перемещении F. Воспользовавшись этим фактом, следует рассмотреть симметрию Z0, где 0 — центр симметрии параллелограмма. 28. Допустим, что (2k + 1) - угольник, ограниченный простой замкнутой ломаной Р, имеет центр симметрии. При этой симметрии каждая вершина многоугольника отображается на его вершину. Так как число вершин многоугольника нечетно, то отсюда следует, 2 Заказ 43 33
что одна вершина (пусть это Лх) отображается на себя, т. е. много¬ угольник симметричен относительно A^ Рассмотрим отрезки АгА 2 и Л{Л2А | j. При симметрии ZA они должны отображаться друг на друга (существуют лишь две стороны Р, имеющие общую вершину Л^. Но это невозможно: Р — простая замкнутая ломаная, и, зна¬ чит, соседние звенья не могут лежать на одной прямой. 29. Рассматривая рисунок 23, можно заметить, что интересую¬ щие нас отрезки АВ и CD симметричны относительно середины М отрезка ОгОs. Действительно, при симметрии ZAf окружности (0lt rJ и (02, га), имеющие равные радиусы, отображаются друг на друга, а прямая /, проходящая через центр М, отображается на себя. Сле¬ довательно, точки А и В пересечения t с одной из окружностей сим¬ метричны точкам С и D пересечения 1 с другой окружностью, т. е. \АВ\ - |CD\. 30. Допустим, что \АМ\ = |ЛЛГ| (рис. 24). Тогда точки М и N симметричны относительно точки Л, и, значит, образ окружности (Oi, г±) при симметрии ZA — окружность, проходящая через точку N. Но N € Окр (02, г2). Поэтому для решения задачи остается по¬ строить окружность (O', fi), симметричную Окр (0lt г±) относитель- ноточки Л, найти точку N ее пересечения с Окр (02, г2) и соединить точки N и Л. (AN) — искомая прямая. 31. Буквы алфавитов распределяютсяпо классам симметрии так: к классу {£} относятся буквы Б, Г, Ё, Й, Р, У, Ц, Ч, Щ, Ь, Ъ, Ы, Я и буквы F, G, I, J, L, Р, Q, R, Y латинского алфавита; к классу {E, Z} относится буква И (N, S, Z). В класс {£, S} входят буквы А, В, Д, Е, 3, К,Л, М, П, С, Т, Ф, Ш, Э, Ю, а также А, В, С, D, Е, К, М, Т, U, V, W; класс {£, Si, 5а, Z} состоит из букв Ж, Н, О, X и латинских Н, О, X. 34. Фигуры А, В, Г относятся к тому же классу симметрий, что и квадрат; множество симметрий фигур Б, Д состоит из четырех по¬ воротов с общим центром на углы 0°, 180°, 90° в обоих направлениях; фйгура Е принадлежит классу симметрий прямоугольника. 35. См. рис. 25. 36. Например, шестиугольник AxB^D^^Fi (рис. 26). 37. Фиксируем одно из перемещений F0, отображающих Ф иа Oi. Заметим, что если F — произвольное перемещение, перевсдя- 84 Рис. 23 Рис. 24
О Д О А Ач 1 1 1 о v о v Рис. 25 Рис. 26 щее Ф в Ф{% то F$ 1 о F — перемещение G, отоб¬ ражающее Ф на себя: FjT1 о F = G, откуда F= — F0oC ... (1). Таким образом, каждое ис¬ комое перемещение представимо в виде (1). Так как существует 6 симметрий правильного тре- угольника,имеетсяровно6 перемещений, ото¬ бражающих Ф на Ф1# (Если Gi Ф G2, то, до¬ пуская, что F0 о Gx = Fo о G2, и умножая на F~о, получаем: Gt = G2. Это противоречит до¬ пущению Gx Ф Gg, Значит, при различных G* и G2 перемещения F0 о Gi и F0 о G2 различны.) 3 а м е ч а н и е. В ходе проведенного рассуж¬ дения по существу мы показали, что если фигу¬ ра Ф обладает конечным числом N симметрий и Ф1 ^ Ф. то имеется ровно N перемещений, отображающих Ф на Ф1в Этот вывод полезно сформулировать и выделить простейшие следст¬ вия: существуют 8 перемещений, отображающих квадрат на конгруэнтный ему квадрат; 4 переме¬ щения, отображающих прямоугольник общего вида на конгруэнт¬ ный ему прямоугольник, и т. д. 47, 48. См. рис. 79 пособия. 49* Если покрытие плоскости правильными пятиугольниками возможно, то существует пятиугольник, вершина которого принад¬ лежит стороне фиксированного пятиугольника ABCDE. Внутрен¬ ний угол правильного пятиугольника равен 144°, Ясно поэтому, что как бы ни расположить этот соседний пятиугольник, свободную часть плоскости между ними нельзя заполнить еще одним пяти¬ угольником. Рис. 27 2* 35
53. Для случая п — 8 фундаментальная область розетки отно¬ сительно Z„ показанана рисунке 27. 59. Решениеудобно записать в виде следующей таблицы: L L L L I, 2, 11, 13, 22, 24 25 L Г L Г i 5, 15, 26, 30 N N N N 3, 8, 12, 17, 23 D D D D 4. 6, 14» 28, V V V V 7, 9, 10, 16, 27 V А < > 18,20, 31 н Н н н 19, 21. 29 Здесь в первом столбце символически показан класс симметрии бордюра (см. рис. 6), во втором столбце указан номер бордюра, изображенного на рисунке 66 пособия. 64. Доказательство очевидно: композиция поворота Ro и осе¬ вой симметрии Sa меняет ориентацию. Представим Ro в виде компо¬ зиции двух осевых симметрий: Ro = S„ о S,, где b || а. Так как а || b, то Sa о Sb — перенос Т и, следовательно, Sa о Ro = Sa о (Sb о S,j = (Se-o Sb) о S( = Т о S,. Задача свелась к.рассмотренному выше случаю (см. задачу 5). Если 0 € а, решение упрощается, Ro = Sa о Sp, поэтому 5Д о R% = Sa о (Sa о Sp) = (Sa о Sa) с Sp = Е о S,= Sp, т. е. в этом случае Sa о R% — осевая симметрия, ось которой про¬ ходит через точку 0 и образует угол ^- с прямой а. 65. Композиция поворота и переноса: а) сохраняет ориента¬ цию; б) отображает каждый луч Ох плоскости на луч, образующий, —>■ с [0X) угол а. Следовательно, Ro ° а — поворот на угол а. Чтобы а ^^*" найти центр Oj этого поворота, заметим, что Ro ° а (0j) = Oi, и, значйт, Ro (0j) = —а (0j). Пользуясь этим свойством точки Ох, лег¬ ко ее построить (рис. 28). Из точки 0 опускаем пе'рпендикуляр на прямую 1, параллельную направленйю а, откладываем уГлы АОС и СОВ величиной ^-. Затем строим отрезок длины |о|, концы кото¬ рого лежат на лучах 0A и 0B. 36
н Рис. 31 Рис. 28 66. (См. также № 79 пособия.) Так как S, о Ro ° S, сохраняет ориентацию, эта композиция есть перенос или поворот. Заметив, что угол между любым лучом 0A и его образом OA' есть —а (рис. 29), а также что точка 0 неподвижная точка композиции 5, о R% о S/, получаем S, о flg о S, = iTg. —► 67. При перемещении St о а ° S, каждый луч отображается на co- направленный ему луч. 68. Вершина Ах (рис. 30, п = 5) при композиции симметрий Zn о Z о о Zn отображается на'себя. Композиция нечет- U П ^n- i 1 ного числа центральных симметрий — центральная симметрия. Следовательно, остается найти центр этой симметрии. Заметив, что Z0 о z0 == 20x0a, Z0t ° z0 = 20304, решение этой задачи сводимк нахождениюцентра симметрии, являющейся композицией переноса2 (ОхОа + 0304 + ...)исимметрии Z0 . После того как точка Аг построена, дальнейшее решение очевидно. 70. Композиция пяти поворотов на 90° — поворот на 90°. Од¬ на из вершин пятиугольника — центр этого поворота. 71. Указание. Пусть Аг = ЕН (А) (рис. 31). Так какдлина | ЕН\ постоянна и \AiA\ = \ЕМ\, надо найти минимум длины ломаной А1НВ. Эта задача уже решена (см. пример 1, с. 38 пособия). 72. Указание. Заметьте, что если /2 — серединный перпендику¬ ляр к стороне АгА2, /2 — серединный перпендикуляр к стороне А2А3 и т. д., то точка Ах — неподвижная точка композиции 5 1„ ° $ ln-l ° ••• 0 ^1- 76. Представив' параллельный перенос а в виде композиции двух симметрий с параллельными осями 1Х и L (см. задачу 3), полу¬ чим: Sf = Sh о Si, о Sa. 78. а) Параллельный перенос — а; б) поворот Я~а; в) симмет¬ рия S(', г) скользящая симметрия S~a. 37
80. Пользуясь ассоциативностью композиции и свойствами f о f-1 = Е и Е о f=f, получаем: tfogof-l)c(fohof-') = (fog)o(f-lof)c(h*f-1) = tf°g)° О Е О (ft О f-1) = / о te О h) О /-1. si- (f*g*f-')o(f<>g-'*r') = (f*g)<>U-'<>f)*te-1 °/_1) = = /° fe ° g-1) ° /-1 = f ° г1 = £• 82. Нет, композиция двух осевых симметрий не является осе¬ вой симметрией. 83. Нет, композиция двух центральных симметрий есть пере¬ нос, а не центральная симметрия, 84. 1) Если #^ (X) = Xt и #** (Xj) = Х2, то по определению гомотетии имеем: ОХ2 = kj5x, = k2 (kOX) = (Мг)0X. т. е. Х2 — образ X при гомотетии tf*>4 Так как X — произволь- j_ ная точка плоскости, то tf** ° Щ = #£-*«. 2) Гомотетия Н% — преобразование, обратное гомотетии Нк0. Вследствие 1) и 2) множе¬ ство гомотетий с общим центром образует группу. 85. Переносы в заданном направлении не образуют группы: композиция двух переносов в заданном направлении — перенос в том же направлении, но перенос.обратный переносу Т, имеет на- правление. обратное направлению Т. Переносы, параллельныедан- ной прямой, образуют группу (необходимо договориться, чтона- правление тождественного переноса параллельно любой прямой плоскости). 86. Да. Sa о Sa = Е, S7« = Sa. 87. D6o° = {ABCDEHV Di200 = (ABCDEH\ Dieo° — (ABCDEfft . * \BCDEHAj' [CDEHAB)' * \DEHABCj' p24o° — tABCDEH\ . рэоо° _ [ABCDEH\ . p (ABCDEH\ . * [EHABCD) * * [HABCDE) ' * [ABCDEH) ’ <? = (ABCDEHV o = (ABCDEHV o (ABCDEHV lt [CBAHED)’ h \DCBAHE}' u \EDCBAH)' c e (ABCDEHN. o = (ABCDEHV o = (ABCDEH] u \AHEDCBj’ °1‘ {DCBAHE)' '• \HEDCBAy 38
88. Таблица (^^g^^J задает преобразование, обратное, а таблица ^j^g^) — преобразование, которое получится, если последовательно выполнить преобразование f три раза. Так как Оф — группа, преобразования /-1 и /3 принадлежат G&. 89. f 0 * = (ABCDEF] а о f = (ABCDEF\ ' ё (FABCDE) • 5 ' (f4BCDf/* 90. Так как S, и Z0 являются симметриями Ф, то и компози¬ ция Si ° Z0 является симметрией фигуры Ф. Представив ZQ в виде St о Sm, где т ± 1, 0 6 т, получим: $i ° ч — ^i ° (Si о Sm) = (S, о Sj) о Sm = Sm. 91. Если R и 5Й — симметрии фигуры Ф, то R^1 о S о R — тоже симметрия Ф. Пусть прямая а является образом прямой 1 при повороте R. Тогда для любой точки X 6 1 ее образ Xi при пово¬ роте R принадлежит а и, следовательно, (R-1 о Sa с R) (X) = (R-1 о Sa) (X0 = R^1 (SA*i)) = R-4X,)=X. Итак, все точки прямой 1 — неподвижные точки перемещения R~l о Sa о R. Поскольку это перемещение меняет ориентацию (повороты R и R^1 сохраняют ориентацию, а симметрия Sa ее ме¬ няет), то R~x о Sa ° R — осевая симметрия S,. Остается заметить, что повороты Reo°, R120°, R1B0° являются симметриями фигуры Ф. 92. Композиция любого числа симметрий фигуры Ф является симметрией Ф. 93. Так как композиция (па) ° Zo сохраняет ориентацию и пере¬ водит каждый луч плоскости в сонаправленный ему луч, то па о Z0—центральная симметрия (п—любое целое число). Остается заметить, что точки 0„ — (О) — неподвижные точки этого перемещения. Следовательно, точки Оп — па (О) при любом целом п являются центрами симметрии фигуры Ф. ДОПОЛНИТЕЛЬНАЯ ЛИТЕРАТУРА 1. А б р а м о в А. М. Аксиома подвижности и ее следствие. — В кн.: Преподавание геометрии в 6—8 классах. М., 1960. 2. Вейль Г. Симметрии. М., 1968. 3. Кокстер Г. С Введение в геометрию. М., 1966. 4. Ш v б н и к о в А, B., К о п ц и к В. А. Симметрия в науке и искусст¬ ве. М.. 1968.
ЭЛЕМЕНТЫ МАТЕМАТИЧЕСКОЙ ЛОГИКИ Тема «Элементы математической логики» не¬ посредственно примыкает к основному курсу математики, расши¬ ряя и углубляя те сведения из логики, которые в нем содержатся. Основная цель изучения темы — развитие культуры мышления учащихся. Этой целью определяется, общеобразовательная роль факультатива, его межпредметный характер. Факультатив по логике имеет важное прикладное значение. Основное приложение логики состоит в использовании ее ме- тодовдля проведения и проверки рассуждений. Восходящие кАрис- тотелю формально-логические методы анализа рассуждений стали доступными и. удобными в обращении благодаря использованию языка математической логики (именно поэтому факультатив по ло¬ гике и находится в ряду математических факультативов). Вторым возможным применением логики является использова¬ ние ее средств для уточнения языка. Сейчас, когда электронно- вычислительная техника проникает буквально во все сферы жизни, это приложение логики приобретает чрезвычайную актуальность. Третий аспекТ приложений логики условно можно наавать «тех¬ ническим».. Аппарат математической логики используется для ана¬ лиза и синтеза переключательных схем, имеющих разнообразные применения в технике. Все эти аспекты приложений логики отра¬ жены в содержании темы «Элементы математической логики». Важность изучения логики, ее общеобразовательная ценность становятся особенно ощутимыми, если вспомнить слова В. И. Ле¬ нина «...всякая наука есть прикладная логика» (Ленин В. И. Полн. собр. соч., т. 29, с. 183). В учебном пособии «Избранные вопросы математики» содержа¬ ние темы изложено в.двух разделах: «Логика высказываний» и «Высказывательные формы и операции над ними». Содержание первого раздела представляет собой упрощенное, однако достаточно полное^ и систематичное изложение логики вы¬ сказываний. Вводятся основные понятия и формальный аппарат; рассматриваются приложения. Второй раздел содержит элементы логики предикатов, непо¬ средственно связанные с логическими понятиями основного курса школьной математики и ее языком. 40
Содержание первого раздела естественным образом распадается на следующие смысловые «блоки»: Классическая логика (п. 1) — 1 ч. Высказывания и операции над ними (п. 2—6) — 2 ч. Логические отношения (п. 7, 8) — 2 ч. Переключательные схемы (п. 9) — 1 ч. Второй раздел включает два «блока»: Операции над высказывательными формами (п. 1—3) — 2 ч. Кванторы (п. 4—8) — 3 ч. Один час отводится на зачет по теме/ Рекомендуемое распределение часов является примерным. Если времени для изучения материала в полном объеме окажется недостаточно, то можно предложить учащимся часть материала рас¬ смотреть самостоятельно. Для этого целесообразно выделить пунк¬ ты 2 и 3 из первого раздела и( пункты 1 и 6 из второго раздела, поскольку материал этих пунктов самым тесным образом связан с основным курсом математики и до некоторой Степени знаком уча¬ щимся. Следует также иметь в виду, что пункты 8 и 9 первого раздела и пункты 5 и 8 второго раздела (они помечены звездочкой) можно рассмотреть лишь в ознакомительном плане. Кроме того, пункт 9 первого раздела и пункт 8 второго раздела можно использовать в качестве материала для рефератов или доклйдов. Упражнения к каждому пункту даны в избыточном количестве; на занятиях должны быть решены лишь некоторые из них, отобран¬ ные по. усмотрению учителя. ЛОГИКА ВЫСКАЗЫВАНИЙ Классическая логика. Занятие проводится в форме беседы с по¬ следующим выполнением упражнений на представление и проверку рассуждений с помощью диаграмм Эйлера — Венна. В беседе учи¬ тель излагает содержание первого пункта первого раздела. Прежде чем перейти к упражнениям, следует повторить с учащимися опре¬ деления операций над множествами, отношения «быть подмноже¬ ством» и их представления с помощью диаграмм. Поскольку заня¬ тие имеет лишь ознакомительный характер, упражнения на нем несут чисто иллюстративную нагрузку и не преследуют цели выработки навыков. Поэтому можно ограничиться демонстрацией образцов выполнения упражнений на доске, сопроводив ее необхо¬ димыми комментариями. Следует обратить внимание учащихся на различие между п р а в и л ь н о с т ь ю (рассуждения) и и с т и н- н о с т ь ю (посылок, заключения) и выявить зависимость истин¬ ности заключения от правильности рассуждения и истинности по¬ сылок. Заключение заведомо истинно тогда и только тогда, когда рассуждение правильно и все посылки истинны; если рассуждение неправильно или хотя бы одна из посылок ложна,'заключение мо¬ жет быть как истинным, так и ложным. 41
Высказывания и операции над ними. В результате изучения содержания этого «блока» учащиеся должны усвоить следующие о с н о в н ы е п о н я т и я: высказывание, значение истинности высказывания, высказывательная форма, простое высказывание, сложное высказывание, логические операции: отрицание, конъюнк¬ ция, дизъюнкция, импликация, эквиваленция. В с п о м о г а- т е л ь н о е п о н я т и е: таблица истинности. Понятия «выска¬ зывание» и «высказывательная форма» (предложение с переменными) не являются совершенно новыми для учащихся, так как использу¬ ются в основном курсе. Следует подчеркнуть, что высказывательная форма — не всякое предложениес переменными, а лишь такое, которое при подстанов¬ ке вместо всех переменных их значений становится высказыва¬ нием. Например, предложение «Сколько жителей в городе N ?» содержит переменную N, но не является высказывательной формой, так как при подстановке вместо N названия города не становится высказыванием (высказывание не может быть вопросительным пред¬ ложением). Учащиеся знают, что всякое высказывание либо истинно, либо ложно. Остается объединить понятия «истинное высказывание» и «ложное высказывание» общим термином «значение истинности выс¬ казывания». Говоря о простых и сложных высказываниях, надо разъяснить, что с точкизрения логики единственной характеристикой, един¬ ственным свойством простого высказывания является его значение истинности; сложное высказывание характеризуется структурой (логической формой), отражающей способ, которым оно образовано из простых высказываний, и определяющей его значение истин¬ ности. Прежде чем давать табличные определения логических опера¬ ций, следует убедить учащихся в их необходимости, показав на примерах неоднозначность логических связок в естественном язы¬ ке. Этой цели может послужить, например, вопрос о значении ис¬ тинности предложения «Число 2 — четное или простое». Разноре¬ чивые ответы учащихся обнаруживают неоднозначность союза «или», который употребляется как в разделительном, так и в нераздели¬ тельном значениях, и необходимость уточнения его смысла с по¬ мощью определения. Предложения «Если идет дождь, тр крыши мокрые» и «Если Иванов увлекается историей, то Петрор ничем, кроме хоккея, не интересуется» иллюстрируют неоднознедрост^ союза «если ♦.., то», который в первом предложении выражает Причинно-следственную связь, а во втором — синонимичен союзу «а». Полезно предложить учащимся самим подобрать примеры упот¬ ребления одного и того же союза в разных значениях. Вводя определения логических операций, надо отметить, что при этом возникают некоторые расхождения с обычным словоупот¬ реблением. Так, например, едва ли кто-нибудь согласится считать 42
истинным предложение «2 X 2 = 4 или белые медведи живут в Аф¬ рике»; однако в силу определения дизъюнкции это предложение истинно. Наибольшие расхождения с привычным словоупотреблением вы¬ текают из'определения импликации. Здесь возникают так называе¬ мые «парадоксы импликации». Примерами таких парадоксов могут служить предложения «Если 2 X 2 = 5, то существуют ведьмы» и «Если 2 х 2 *= 5, то Волга впадает в Каспийское море»; истинность этих предложений, вытекающая из определения импликации, плохо согласуется с нашими обычными представлениями. Однако в мате¬ матике союз «если..., то» всегда используется в полном соответ¬ ствии с определением импликации. Это можно показать учащимся на следующем примере. Пусть дано предложение «Если натураль¬ ное число п делится на 4, то оно четно». Признавая справедливость этого предложения для любого натурального n, мы обязаны считать истинными такие высказывания, как «Если 16 делится на 4, то 16 — четное число», «Если 18 делится на 4, то 18 — четное число», «Если 17 делится на 4, то 17 — четное число», что соответствует первой, третьей и четвертой строкам таблицы истинности, определяющей импликацию (см. с. 87 пособия). Ложного высказывания, соответ¬ ствующего второй строке таблицы, в силу справедливости исход¬ ного утверждения мы не получим ни при каком значении п. 0 символике. Наряду с принятыми в пособии символами для отрицания, конъюнкции, дизъюнкции* импликациииэквиваленции употребляются и другие символы. Так, например, импликацию и эквиваленцию обозначают иногда символами =^ и <=» соответственно. Выбор символов, используемых в пособии для учащихся, продик¬ тован следующими соображениями. Черта над сложным логическим выражением, изображающая его отрицание, освобождает от необходимости заключать это выраже¬ ние в скобки. Символы А и V просты в начертании; их симметричная форма соответствует коммутативности конъюнкции и дизъюнкции; сход¬ ство между собой символизирует равноправие этих операций, а сходство с символами f] и U напоминает о связи конъюнкции с пересечением множеств и дизъюнкции с объединением множеств. Принятие символов ^ и «+ для обозначения импликации и эквивален^ии обусловлен^ тем, что символы => и <н> используются в осцррном курсе для обозначения отношений следования й равно¬ сильности. Логические отношения, О с н о в н ы м и п о н я т и я м и это¬ го «блока» являются следующие: эквивалентность логических вы¬ ражений, законы алгебры логики (коммутативность, ассоциатив¬ ность, дистрибутивность, отрицание конъюнкции, отрицание дизъ^ юнкции, двойное отрицание); эквивалентность, А^В&ьЕ^ А (закон контрапозиции), эквивалентности, выражающие имплика¬ цию и эквиваленцию через конъюнкцию, дизъюнкцию и отрицание; упрощение логических выражений, отношение логического следо¬ 43
вания. В с п о м о г а т е л ь н ы е п о н я т и я: тавтология, эк¬ вивалентности А Д A$=* Л, А V A<^>Hy используемые для упро¬ щения логических выражений. В курсе алгебры VII класса вводятся отношения следования и равносильности (эквивалентности) для высказывательных форм (предложений ^переменными); наличие или отсутствие этих отно¬ шений между предложениями зависит, вообще говоря, от множе¬ ства, на котором предложения рассматриваются, В логике выска¬ зываний наличие следования или эквивалентности между предло¬ жениями зависит только от их структуры, представляемой логи¬ ческими выражениями, соответствующими этим предложениям. Например, предложения «х — ромб» й «диагонали х взаимно пер¬ пендикулярны» равносильны на множестве параллелограммов и неравносильны на множествечетырехугольников. Предложения же «Если х — ромб, то диагонали х взаимно перпендикулярны» и «Ес¬ ли диагонали х не перпендикулярны, то х не ромб» равносильны (эквивалентны) на любом множестве в силу эквивалентности соот¬ ветствующих им логических выражений А ~> В и В ^ А. Перед тем как приступить к изучению отношений эквивалент¬ ности и логического следования, нужно повторить сведения об этих отношениях, известные учащимся из основного курса, и выяснить их общую основу: а) предложения А и В равносильны, если они одновременно истинны либо одновременно ложны; б) из предло¬ жения А следует предложение В, если всякий раз, когда истинно Д.истинно и В. В ходе изучения отношений эквивалентности и логического следования следует обратить внимание учащихся на их специфику в логике высказываний. Учащиеся должны овладеть двумя методами установления факта эквивалентности логических выражений: табличным и методом пре¬ образований. Для применения табличного метода достаточно знать только определения логических операций. Использование метода преобразований становится возможным только при наличии некото¬ рого запаса уже доказанных эквивалентностей. Следует подчерк¬ нуть универсальность табличного метода, его потенциальную при¬ менимость во всех случаях. Чтобы можно было сравнивать таблицы истинности различных выражений, нужно составлять их каким-нибудь единым, стандарт¬ ным способом. Покажем один из таких способов на примере выра¬ жений А А В и А А В А (С V Q- Поскольку общее число раз¬ личных переменных в заданных выражениях равно трем, сводная таблица истинности для них будет иметь 8 строк по числу различных упорядоченных наборов значений переменных (число таких набо¬ ров для п различных переменных равно 2я). Над первыми тремя столбцами таблицы располагаем переменные в алфавитном порядке. Первый слева столбец заполняем сверху вниз четырьмя И, затем четырьмя Л, второй столбец.— двумя И, двумя Л, затем снова двумя И и двумя Л, в третьем столбце И и 44
Л чередуются построчно. Остальные столбцы заполняются согласно определениям логических операций. Сравнивая столбцы, соответствующие данным выражениям, ви¬ дим, что они совпадают, т. е. А Л B<=*A Д В Л (С v Q- А в с с А Л В сус (А А В) А (С V Q И и и л и и и И и л и и и и И л и л л и л и ч л л и л и л л и и л л и л л и л и л и л л л и л л и л л л л И ' л и л С помощью таблицы легко убедиться, что истинной теореме А ~+ В может соответствовать как истинное, так и ложное обратное пред¬ ложение В ^- А. _ _ В силу эквивалентности А ^- В «=> В ^- А вместо данной теоре¬ мы можнолоказывать противоположную обратной, и наоборот, а вместо теоремы, обратной данной, теорему, противоположную дан¬ ной, и наоборот. В пособии дается определение следования одного логического выражения из другого. При проверке правильности выводов из не¬ скольких посылок это определение используется так: 1) формализуются все посылки и заключение; 2) составляется конъюнкция формализованных посылок; 3) если логическое выражение, соответствующее заключению, нстинно во всех случаях, когда истинна конъюнкция формализо¬ ванных посылок, то заключение следует из посылок, т. е. рассуж¬ дение правильно. Если хотя бы в одном случае конъюнкция посы¬ лок истинна, а заключение ложно, то рассуждение неправильно. Для проверки правильности вывода из нескольких посылок удоб¬ но применять так называемый «метод обратного рассуждения». По¬ кажем на примере, в чем заключается и как.применяется этот метод. Пусть требуется проверить, следует ли из посылок А ^- В, С v В, С заключение А. Допустим, что заключение не следует из посылок, т. е. что есть такой набор значений истинности А , В и С, при котором все посылки истинны, а заключение ложно. Зафикси¬ руем это допущение в таблице, 48
В эту жетаблицузапишемслед- ствия изсделанногодопущения: 1) А — истинно; 2) А истинно, А •+ В истинно, следовательно, В истинно; 3) С истинно, следо¬ вательно, С ложно; 4) С V В ис¬ тинно, С ложно, следовательно, В истинно. Получили противо¬ речие: В и В одновременно ис¬ тинны. Следовательно, наше допущение ложно: заключение сле¬ дует из посылок, т. е. рассуждение правильное. Рассмотрим еще одии пример: А -*■ (В у Q; В Д С; следова¬ тельно Л. Допустим, что рассуждение неправильное, т. е. сущест¬ вует такой набор значений истинности Л, В и С, при котором по¬ сылки истинны, а заключение ложно. Тогда из истинности В Д С следует, что В истинно и С истинно. Получили такой набор значе¬ ний А, В и С, при котором посылки истинны, а заключение ложно (проверЁте). Значит, рассуждение неправильное. Переключательные схемы. Здесь показывается на простей¬ ших примерах приложение ап¬ парата логики высказываний к переключательным схемам. Основные понятия. Соответствие между: а) последо¬ вательным соединением переклю¬ чателей и конъюнкцией;, б) па¬ раллельным соединением переключателей и дизъюнкцией; в) ин¬ версными переключателями и операцией отрицания. Всякая переключательная схема с последовательными или па¬ раллельными Соединениями переключателей может быть описана с помощью логического выражения. Наоборот, каждому логическо- му выражению, не содержащему никаких символов операций, кро- мё символов конъюнкции, дизъюнкции или отрицания, соответст¬ вует переключательная схема. Описание схемы логическим выраже¬ нием дает возможность решать средствами логики высказываний задачи: а) анализа схемы; б) упрощения схемы; в) синтеза схемы. а) Анализ схемы состоит в определении условий работы данной схемы. Чтобы решить эту задачу, достаточно составить таблицу ис¬ тинности выражения, соответствующего данной схеме; по таблице видно, при каких наборах состояний переключателей цепь замкну¬ та, и при каких — разомкнута. б) Для того чтобы решить задачу упрощения схемы, достаточно упростить выражение, соответствующее данной схеме, и по выра¬ жению, полученному в результате упрощения, составить схему, которая «эквивалентна» данной, т. е. работает так же, как и данная, при меньшем числе переключателей. И л А + (В V С) ВАС А В с И Л А^В А сув С А В В С 46
в) Синтез схемы состоит в составлении схемы по заданным ус¬ ловиям ее работы. Условия работы схемы можно представить в виде таблицы истинности; по ней составляется выражение, соот¬ ветствующее искомой схеме, а затем и сама схема. ВЫСКАЗЫВАТЕЛЬНЫЕ ФОРМЫ И ОПЕРАЦИИ НАД НИМИ Операции над высказывательными формами. Здесь известные учащимся понятия «высказывательная форма» и «логические опера¬ ции» рассматриваются во взаимной связи и в связи с операциями над множествами. «Мостик» между логическими операциями и опе¬ рациями над множествами перебрасывается с помощью понятия «множество истинности высказывательной формы». Логическим опе¬ рациям над высказывательными формами соответствуют операции над их множествами истинности, а логическим отношениям между высказывательными формами соответствуют отношения между множествами. Конъюнкции соответствует пересечение множеств, дизъюнкции — объединение множеств, отрицанию *— дополнение множества до уни¬ версального. Отношения следования и равносильности (эквивалентности) ин¬ терпретируются отношениями включения и равенства множеств со¬ ответственно. Такая интерпретация логических операций и отно¬ шений доставляет очень ценную в методическом отношении возмож¬ ность сообщить наглядность абстрактным логическим понятиям. При изучении содержания этого «блока» следует как можно больше иполнееиспользовать теоретико-множественные иллюстрации; они помогают выяснить сущность логических понятий и облегчают их усвоение. Все основные понятия этого блока — высказывательная форма, кйнъюнкция, дизъюнкция, отрицание, импликация и экви- валенция высказывательных форм, отношения следования и равно¬ сильности (эквивалентности) высказывательных форм, явно или неявно (без введения термина) присутствуют в основном курсе и в той или иной степени знакомы учащимся. На эти знания следует опираться на всем протяжении изучения «блока», напоминая уча¬ щимся те сведения, которые изучались ранее, и связывая их с но¬ выми. Операции над множествами и отношения «быть подмножеством» и равенства (совпадения) множеств, выступающие здесь как вспо¬ могательные понятия, также известны учащимся из основного курса. Следует обратить особое внимание на понятие «логическое сле¬ дование», котороеявляетсятруднымдля учащихся в силу того, что непосредственные следствия из его определения психологически не очевидны, даже парадоксальны. Отметив, что в отличие от логиче¬ ского следования в логике высказываний следование между выска¬ зывательными формами зависит от множества, на котором эти фор¬ мы рассматриваются, полезно рассмотреть следующееупражнение. 47
Задайте множество М значений переменной п так, чтобына этом множестве из предложения т четно» следовало предложение т кратно 3». В качестве элементов этого множества учащиеся предло¬ жат числа 6, 12, 18 и т. д. Тогда следует задать вопрос, можно ли включить в М числа 1, 2, 3. Возникающие при этом сомнения раз¬ решаются обращением к определению (Р =>■ Q, если всякий раз, когда истинно Ру истинно и Q) и его теоретико-множественной ин¬ терпретации. Числа 1 и 3 можно включить в искомое множество, так как эти значения-переменной дают не запрещенные определени¬ ем сочетания значений истинности Р и Q^— «Л — Л» и «Л — Я»; число 2 не должно входить в искомое множество, так как при п = 2 предложение Р истинно, а Q ложно, что запрещено определением. Учителю надо иметь в виду, не заостряя на этом внимания уча¬ щихся, чтотеоретико-множественная трактовка бинарных логиче¬ ских операций и отношений непосредственно приложима лишь к предложениям с одинаковыми переменными; при этом если число переменных больше двух, то для них должен быть фиксирован не¬ который порядок. Поясним сказанное примерами. 1. Уравнения х = 0 и х + у — у = 0 равносильны, но множе¬ ства их истинности не совпадают: Тд^0 = {0}; Тл+у_у==0 — множество всевозможных пар вида (0, у), где у — любое число. 2. Множества истинности неравенства х > Q и у > 0 совпада¬ ют, но эти неравенства не равносильны: ни одно из них не следует из другого. 3. Пара (1, 2) принадлежит множеству истинности уравнения х + 2y = 5, если для него зафиксирован алфавитный порядок пе¬ ременных, и не принадлежит, если принят обратный порядок пе¬ ременных. Таким образом, одно и то же предложение, конечно же равносильное самому себе, может иметь различные множества ис¬ тинности в зависимости от порядка, принятого для его пере¬ менных. В первом примере можно «исправить положение», введя недо¬ стающую переменную у с нулевым коэффициентом: предложения х + о • у = 0 и х + у — у = 0 равносильны и множества их ре¬ шений совпадают. Цванторы, Понятие «квантор» не является совсем новым для учащихся. По существу новым является лишь термин, так как с кванторами учащиеся встречаются повседневно при изучении ма¬ тематики на уроках. В формулировке любой теоремы присутствует или подразумевается слово «всякий». Опровергая какое-либо ут¬ верждение всеобщего характера, т. е. относящееся ко всем элемен¬ там некоторого множества, учащиеся доказывают, что в этом мно¬ жестве существует элемент, для которого данное утверждение не выполняется. Слова «всякий» и «существует» и есть кванторы общ¬ ности и существования, соответственно.Однако, несмотря на боль¬ шую практику обращения с кванторами, учащиеся не осознают их свойств, не являющихся интуитивно очевидными, и В связи с этим испытывают затруднения и допускают ошибки. 48
При изучении этой подтемы учитель должен научить учащихся: а) записывать символически предложения с кванторами; б) переводить на естественный язык символические записи пред¬ ложений с кванторами; в) строить отрицания предложений с кванторами; г) выявлять в предложениях, сформулированных на естествен¬ ном языке, подразумеваемые кванторы. Возможность формирования перечисленных умений заложена в системе упражнений, данной в пособии. Эта система избыточна; учитель имеет возможность выбиратьупражнения по своему усмот¬ рению. ПРИМЕРНОЕ СОДЕРЖАНИЕ ЗАЧЕТА Ha' зачете рекомендуется проверить следующие знания и уме¬ ния. 1) Знание определений логических операций и их связей с опе¬ рациями над множествами. 2) Умение выделить логическую форму предложения на уровне логики высказываний. 3) Умение составить таблицу истинности для данного логиче¬ ского выражения. • 4) Умение строить отрицания сложных предложений и предло¬ жений с кванторами. 5) Знание определений эквивалентности и логического следо¬ вания для выражений логики высказываний и для высказыватель¬ ных форм. 6) Знание теоретико-множественной интерпретации эквивалент¬ ности и логического следования высказывательных форм. 7) Умение устанавливать наличие или отсутствие эквивалент¬ ности и логического следования между предложениями. 8) Знание соотношений между любой данной теоремой и об¬ ратным, противоположным, обратным противоположному ей утверждениями. 9) Знание основных законов алгебры высказываний. Ш) Умение упрощать выражения логики высказываний. 11) Умение записывать символически предложения естествен¬ ного языка, явно или неявно содержащие кванторы, формулировки теорем и определений: 12) Уметь решать задачи типа 16, 18, 19, 21, 22, 24, 25, 28, 33, 34, 36, 41, 64, 67, 72, 73, 75, 77, 79, 81, 84, 86, 88, 90, 91, 111, 116, 117, 118. Ответы, решения, указания к упражнениям 1. а) Рис. 1 (а, b); б) рис. 2, рис. I (а, б); в) рис. 3; г) рис. 2, 3, 4. Следует подчеркнуть, что слово «некоторые» в логике имеет смысл «хотя бы один, а может быть и все». 49
Рис. 2 Рис. 3 Рис. 4 Рис. 5 2, Доказательство непосредственно ус¬ матривается из диаграмм, см. рис. 1 (а, б), 2, 4. 3. Рис. 3. 4. Рис. 2, 4. 5. Не следует; на рисунке 5 приведен опровергающий пример. 6- Рассуждение неправильно» см. рис. 5. Надо пояснить, что значит «пра¬ вильное рассуждение». Рассуждение назы¬ вается правильным,, если заключение следует из посылок. 7. Рассуждение пра¬ вильно: если посылки истинны, то заключение истинно. 8. Рас¬ суждение правильно. 9. Рассуждение неправильно. 10. Рассуж¬ дение неправильно. 11. Рассуждение неправильно. 12. а) Если все х являются у, то некоторые у не являются х\ рассуждение неправильно; б) если ни один х не есть у, то ни один у не есть я;рассуждение правильно (см. ответ к упр. 3); в) если все х являются у ri все х являются г, то все z являются у; рассуждение неправильно; r) рассуждение неправильно; д) рассуждение непра¬ вильно; е) рассуждение правильно; ж) рассуждение правильно; з) рассуждение правильно; и) рассуждение неправильно. 13. а) Рассуждение неправильно; б) рассуждение неправильно; в) рассуждение правильно. 14. а), б), г). 15. Истинные высказывания: а), г), д); ложные высказывания: б), е); предложения в), ж), з) высказываниями не являются. 16. Истинные высказывания: а),б),г),д), к), л),м), р); лож¬ ные высказывания: в), з), и); остальные предложения — высказы- 60
вательные формы. Заметим, что предложения ж) и и) — тождест¬ венно истинные высказывательные формы, а о) — тождественно ложная высказывательная форма. 17. Земля вращается вокруг Солнца и имеет формушара. Земля вращается вокруг Солнца или имеет форму шара. Если Земля вра¬ щается вокруг Солнца, то она имеет форму шара. Земля вращается вокруг Солнца тогда и только тогда, когда она имеет форму шара. Земля не вращается вокруг Солнца или она не имеет формы шара. 18. а) А и В; б) А или В; в) если А, то В; г) А или В; д) А тогда и только тоща, когда В. 19. а) Число 5 — не делитель числа 542. б) Автомобиль имеет право ехать вперед на красный свет, в) Не существует параллело¬ граммов с прямыми углами. Иначе: все параллелограммы — беэ прямых углов, г) Уравнение 2x* — Зх + 1 = 0 не имеет целого корня, д) Не все корни уравнения 2x2 — Зх + 1 = 0 целые. Иначе: существует корень уравнения 2x2 — Зх + 1 = 0, не являющийся целым, е) Не все натуральные числа делятся на 2. Иначе: сущест¬ вует натуральное число, которое не делится на 2. ж) Существует натуральное число, делящееся на 2. з) Не существует целого чис¬ ла, делящегося на все целые числа. Иначе: «Всякое целое число делится не на все целые числа» либо «Для всякого целого числа <?у- ществует целое число, на которое оно не делится». 22. а) 2 > 2 ложно; следовательно, 2 ^ 2 истинно; б) 3 > 5 ложно; следовательно, 3 ^ 5 истинно; г) «Существует русское слово, содержащее три одинаковые гласные подряд». Это предложе¬ ние истинно, так как такое слово действительно существует, на¬ пример, «длинношеее»; следовательно, данное предложение ложно. 23. Истинные конъюнкции: а), з), к); ложные конъюнкции: б), е), ж); истинныедизъюнкции: в), г), д), л); ложная дизъюнкция: м). 24. д) Неверно, что я купил велосипед и участвовалв сорев¬ нованиях по велоспорту, ж) Я не купил велосипед или не участ¬ вовал в соревнованиях по велоспорту, з) Я не купил велосипед и не участвовал в соревнованиях по велоспорту или я путешествовал по СССР. _ _ _ _ 25. a)_C; б) А Д В; в) С Д В; г) А Д С; д) С Д В) е) А у С; ж) А V В. 26. а), в), г). 27. Истинные высказывания: а), б), д), е), з). 28. Истинные высказывания: а), б), в), д), ж), з), и). 29. а) А истинно; б) В ложно; в)' В ложно; г) А истинно; д) А истинно; е) В ложно; ж) А истинно; з) В ложно. 30. а) Если число 729 крат¬ но 9, то сумма его цифр кратна 9; б) если сумма цифр числа' 729 кратна 9, то это число кратно 9; в) число 729 кратно 9 тогда и только тогда, когда сумма его цифр кратна 9. 32. а) А ^ (D Д В); б) (С V А) Д D; в) D Д (В~(1 Д C)). 33. а) (Л Д В) ^ С; б) A*+ (С v D)- 35. Длятогочтобыдоказать, что логическое выражение — тавтология, достаточно составить таблицу истинности для этого выражения и убедиться, что в столб¬ це значений выражения нет ни одного JI. 61
36. А в А А^В АЛ В (A+B) v (AAB) И и Л И И И И л Л И Л и Л и И И Л и л л и л Л л А в А А В ву А (ААВ)^(ВуА) И и И И И И л Л и И Л и Л и И л л л л И А В с А^В В^С (A^B)A(B^C) А^С ((A^B)A(B^C))^(A^C) И и и и и и и и И и л и л л л и И л и л и л и и и л л л и л л и л и и и и и и и л и л И' л л и и л л и и и и и и л л л и и и и и Выражения r), д) — тавтологии; остальные выражения не яв¬ ляются тавтологиями. 37. Вместо переменных можно подставить любые высказывания, заботясь лишь о том, чтобы разным переменным соответствовали раз¬ личные ло смыслу высказывания, а одинаковым переменным — од¬ но и то же высказывание. 38. Логические выражения, ложные при любых зЯачениях ис¬ тинности входящих в них переменных, называют всегда ложйыми. 39. а) Да; б) нет: предложения могут быть оба ложными. 40. На примере эквивалентности д) покажем три cnojo6a докаэйтед^ства эквивалентности выражений. 62
1) Составим таблицу истинности выражений, эквивалентность которых надо доказать. , А В в А А В АЛ В (А А В) V (А Д В) И И Л И Л И j И Л И Л И И Л И Л Л Л Л Л Л И Л Л Л Сравнивая столбцы со значениями выражений Л и (Л Д В) у V (Л Д В), видим, что они совпадают, т. е. выражения эквива¬ лентны. _ _ 2).(i4 Д В) v iA А В)«=>Л Д (В у B)^>A Д #«Л. Экви¬ валентности вида (Л Д В) v (А Д В) <=> А и (Л v Я) Л (Л v 5) <=» <=» Л называются законами склеивания. Ими удобно пользоваться при упрощениях выражений. 3) если Л истинно, то при истинном В истинно Л Д В, а при ложном В истинно' Л Д В, т. е. дизъюнкция (Л Д В) у (Л Д В) влюбом случае истинна. Если Л ложно, то дизъюнкция (Л Д В) v у (Л Д В) ложна, так как ложны обе составляющие ее конъюнк¬ ции, Эквивалентность доказана. 41. а) (Л Д В Д С) у (Л ДВД Д D) у (Л Д_B Д С) V (Л Д D) ^ В <=>((Л Д В) Д (С V Q) у у (Л Д В Д D) V (Л А D Д В)<^«Л Д В) Д И) у ((Л Д_В)Д Д (D у D)) « (Л Д В) у ((Л _A В) Д Я) « Л Д (В у В) «• «=» Л Д И ^ Л. б) (Л Д В) у Л ^ В« (Л Д В) у (А Д В) <=> <=> Л +* В. в) (Л Д В Д С Д Л) у (Z? Д В Д D) у (В Д С) ФФ «=» (В Д С А Л) V (В А Л) v (В А Q<=>J7 у (В Д С) «=>В Д С 42. Упростим инструкцию. Введем обозначения: Л — капитан присутствует на судне; В — с судна выгружают груз; С — рулевой присутствует на судне. Запишем логическое выражение, соответ¬ ствующее инструкции, упростим его: (Л + В) А (В ^ (1 ^f)) «=» (А у В) А (Zf у (Л ^С))« «=>(Л V В) А (В у Л у С) «■ (Л А В) у (Л А Л) у у (Л А С) у (В Д В) у (В Д Л) у (В А С) <=> (Л А В) у V Л у (Л А СХ V В у (В Д Л) у (В Д С) Ф=> (Л А В) у V (Л А В) у В у (Л А С) у (В А С) <=> В у В у (Л 7\ С) у v (В Д С) Ф* В у (Л Д С) <=> B^ (А. А Q. Если с судна не выгружают груз, то рулевой обязан присутство- вать на судне вместе с капитаном. 45. а) Если вы не находитесь в Африке, то вы не находитесь южнее Москвы. Если вы находитесь южнее Москвы, то вы находитесь в Африке. 63
46. В учебном пособии указание к этому упражнению ошибочно помещено под № 45. X : (Л А В А С) V (А A B_A Qv (Л А б А С) V (^ABAC)V V И А в А С); Г : (Л А 5 Л С) V (А Л В Л С) v (J Л В Л С) V (ЛАВ Л С). 47. В ответе к этому упражнению, помещенном в учебном по¬ собии, допущены опечатки. Приводим правильное решение. Пусть А означает «Выход ведет на свободу», а В — «Ты правдив». Искомый вопрос С должен- быть таков, чтобы за ним следовал ответ «Да» в том и только в том случае, если выходведет на свободу. Составляем таблицу: Таблице соответствует вы¬ ражение (Л А В) V (Л V Щ> равносильное Л ^ В. Искомый вопрос: правда ли, что выход ведет на свободу тогда и только тогда, когда ты правдив? 48. а) Проверку можно про¬ извести двумя способами. Пока¬ жем оба способа на примере. Из таблицы видно, что если вы- ражениеХ А Y истинно, товы- ражение X также истинно, т. е. из первого выражения следует второе. 2) Допустим, что из первого выражения не следует второе, т. е. что есть такой набор зна¬ чений переменных, при которых X А Y истинно, а X ложно; но если X А Y истинно, то по оп- ределениюконъюнкцииХистиннои Y истинно. Полученное проти¬ воречие опровергает допущение и тем самым доказывает, что из X и Y следует X, т. е. высказываниеХАУ=^Х истинно, б) Ложно; в) лож¬ но; г) истинно; д) ложно; е) ложно; ж) ложно; з) истинно; и) ложно. 49, а) (X ^ Y) A X => Y. Форма рассуждения правильная; в самом деле, допущение об одновременной истинности посылок и ложности заключения приводит к противоречию: если Y — ложно и X ->■ Y истинно, то X обязан быть ложным, б) Рассуждение имеет ту же форму, что и а), и, следовательно, является правильным, в) Рассуждение неправильное; для выражений (X ^ Y) А Y и X, представляющих конъюнкцию его посылок и заключение, сущест¬ вует набор значений переменных, обращающих первое в И, а вто¬ рое — в Л. Так будет при ложном X и истинном Y. г) Рассуждение А Б Желаемый ответ С И И Да И И Л Да Л Л И | Нет л л Л Нет и 1) X Y хдг X и И и и и л л и л и л л л л л л 54
правильное: выражение ((X ^- У) Д Y) ^- X — тавтология. 50. а) Такие рассуждения либо оба правильны, либо оба не¬ правильны. б) Правильное рассуждение может иметь ложное за¬ ключение в том и только в том случае, если хотя бы одна из посылок ложна. Неправильное рассуждение может иметь как истинное, так и ложное заключение, в) Заключение истинно, г) Хотя бы од¬ на из посылок ложна, д) Рассуждение неправильное. 51. Нельзя. Из посылок не следует заключение. 52. Можно. Из выражения X ^- Y следует выражение Y ^- X. Ппиярп заключение не всегда истинно, так как не всегда истинна поЩлка. B3. Подразумеваемая посылка: «Настоящий мужчина — не трус». Переформулировав посылки и заключение в виде импликаций, получае^рэссуждение следующей формы: (X ^- Y) Д (К ^- Z)=> => Z ^" X. Эта форма — правильная (из первого выражения следу¬ ет второе). 54. а) Форма рассуждения: {(А ^ В) Д (C + D)) Д ((D Д В) ^ Е) Д £ =>A yC. Рассуждение правильное. Док з з а т е л ь с т в oi допустим, что р^суждение неправильное, т. е. возможен случай, когда все посылки истинны, а заключение ложно. Йз (5) следует, что А ложно и С ложно, т. е. А истинно и С истинно. Из (1) и (8) следует, что В истинно. Из (4) следует, что Е ложно. Из (3) и (11) сле¬ дует, что D Д В — ложно. Из (10) и (12) следует, что D лож¬ но. Из (9) и (13) следует, что С ~^ D ложно. Сравнив (2) и (14), видим, что получено про¬ тиворечие, которое опровергает допущение о неправильности рассуждения и тем самым дока¬ зывает его правильность. б) Рассуждение неправильное. в) Рассуждение правильнее. 55. Из условий 1 и 2 следует, что цех С обязан принимать уча¬ стие в утверждении проекта, когда в нем принимает участие цех А. 58. (X Д Y А Z) V (* A Y Д Z) V (X А 7j\ Q V V (* А 7 А Z)^(X Д Y^ v (*_A Ц У (X Д Y Д Z)t* «■ (X А (Y v 2)) V (* А У A Z). и Л (1) А + В (2) C^-D (3) (DAB)^E (4) Е (8) А (9) С (10) В (5) АуС (6) А (7) С (11) Е (12) DAB (13) D (14) C^D 55
59. Искомая схема описывается выражением, соответствующим таблице: (Строки, в которых искомое вы¬ ражение ложно, опущены.) 60. а) Т = {—3; 5}; б) ис¬ тинное высказывание; в) лож¬ ное высказывание; г) истинное высказывание;д) Т —объедине¬ ние множеств чисел, кратных 9, и чисел, не делящихся на 3; е) истинное высказывание; ж)ложное высказывание; з) ис¬ тинное высказывание; и) Т — множество всех прямоугольников; к) истинное высказывание. 61. а) 7'yi(jj, = l -* -j; ТВ{хГ ] oo; 3[; б) ТАМ~ ~ l^j' Тв{х) = 0» В) ^i4(*) = 0’ ^B(x) “* 0* 62. Т = {2, 4, 6, 8}. 64. а) Т — множество людей, живущих в Москве и преподаю¬ щих в школе, д) Т — объединение множества людей, не живущих в Москве, и множества людей, преподающих в школе, ж) Множе¬ ство истинности то же, что в д). 65. Данная эквиваленция истинна для всех х. 66. б. 67. а) Т = {—2; 2} U {—3; 3} = {—3; —2; 2;3}. б) Т = = {2; 4} f) {—4; 4) = {4}. в) Т — R \ {—4; 2} (дополнение мно¬ жества { -4; 2} до R). г) Т = R. д) Т — R \ {—2} (множествовсех действительных чисел, за исключением —2). е) Т — {4}. ж) Т »= = [0; oc[. з) Т = [—1; 9]. и) Т = {—|}« к)Т = ]-oo; 2[. 68. а) См. рис. 1, а; в) см. рис. 2; д) см. рис. 3.. 69. а) см. рис. 3; б) см. рис. 1, б; в) см. рис. 3. 70. a)Q (*) ^P (x); б) P_{x) ^ Q(*); Р (*) ~> Q (*). Р (*) ^- Q (*); Q (*) ^P (*); Q(*)^P(*). 71. Указание. Воспользуйтесь правилом: если импликация Р (*) ^- Q (x) истинна для всех * (из области определения данных предложений), то Р (*) — достаточное условие Q (*), а Q (*) — не¬ обходимое условие Р (*). 72. См. указание к упр. 71. 73. а) Не следует; б) следует. 74. а) Любое подмножество множества натуральных чисел, не содержащее чисел одновременно четных и не делящиеся на 3, на¬ пример, {1, 3, 5, 6, 7, 9}. в) Любое множество слов, не содержащее трехсложных слов, в которых'буква «а» встречается болеедвух раз, например: {абажур, пароход, керосин, каша, треугольник, пара¬ бола}. X V z F И И И И И И л И И л И И л И И И 56
75- В таблице «+» обозна¬ чает, что формы равносильны, а «—», что неравносильны. 76; а) Любое подмножество N, не содержащее четных чисел, не кратных 3, и нечетных чисел, кратных 3. в) Любое числовое множество, не содержащее —1. 77- а) Из второй следует первая, так как {l; 2}c ]-3; 3[. в) Нет. д) Из второй следует первая. 78. а) [1; oo[; б) ]-oo; 3]; в) ]3; oo[; г) ]-oo; 0]; д) R', е) R \ ж) ]—<»; —7[. 79. а) Нет; б) да; в) да; r) да; е) нет; ж) нет; з) да; и) нет; к) да. 80. ty 0', в) ]—oo; 0j; г) j—oo; 0[ U]0l «»[• 81. а) Или; б) и; в) или; г) или; д) и; е) и; ж) и; з) и, или, и. 84. а) Для всякого действительного числа х х — 1 = х; (Л). б) Существует действительное число х, такое, что х — 1 — х; (Л). в) Для всякого действительного числа х х ■— 1 ф х; (И). г) Суще¬ ствует действительное число х, такое, что х — 1 ф х; (И). 85- а) Всякое натуральное число четно; (Л). б) Существует не¬ четное натуральное число; (И). в) Существует натуральное число, кратное 4; (И). г) Всякое натуральное число некратно 4; (Л). 86. а) (Зх € R) Рх (*); б) (Vx € N) Ps (x)-, в) (3* € R)P* (x), г) (3* € Q)P4 (x). 87| а) Для всякого х (х + 1)2= хг + 2x + I; (И). б) Сущест¬ вует х, 'такой, что х + 1 = 2; (И). 88. а) Vx(x = х); б) Vx (х <j х); в) Vy (у2 J> 0); г) Vx (х > > 0 V х < 0 V х = 0); д) Эх (х — 2 = 5); е) Зх (ахг + bx + c=0); ж) 3x(f (x) = 0); з) Зх (х € X); и) (Эх € X)P_(x). 90. а) Зх (Р (x) Л К (x)); _б) (Vx > 2) (Р (x) ^ К (x)); в) Vx (Р (x) ^ N (x)); г) Эх (К (x) A Р (x)). 91. а) Зх (|х| = —х); б) Vx (x* > 0); в) Эу (уй + 2 < 0); г) v* (sin z Ф 2). д) Существует слово, число букв в котором равно 4. е) Сущест¬ вует пункт х, все дороги из которого ведут на юг. ж) Во всяком треугольнике у сумма внутренних углов равна 2 d. 92. а), б) 3 и P^ («); в) Зх А (x); г) 3y Q fy); д) 3т В (m). _ 93. а) Квадрат любого рационального числа не равен 2, VxP(x). б) Всякий человек имеет мать, Vy Q ^y). в) Для всякогодействитель- ного числа х хг 4- 1 ^ 0. г) Все люди смертны. 94. Эквивалентность а) означает, что операция навешивания квантора всеобщности дистрибутивна относительно конъюнкции; эквивалентность б) — что операция навешивания квантора суще¬ ствования дистрибутивна относительно дизъюнкции. а б в г Д е + + 4- — + + + — i + — — ~“ 67
97. Истинные высказывания: а), б), д), е). 98. Истинные вы¬ сказывания: а), б), в), г), д), е). 99. а) (V* € N) (Vy € N) (xy j p^-(x : р V У i P)Y, б) (Ух € Р) (Vy € Р) (3z € Р) (у € 2 Д 2 || х). 100. а) Для всякого х существует а, такое, что Зх -}- 1 = ах\ (И), б) Существует х, таксе, что для всякого а (Зх + 1 = ах); (Л), в) Существует b, такое, что для всякого а существует х, такое, что (х2 + ах + b = 0); (И), г) Ложно. 101. л: . а = х. 102. а) Vx (2х > 0); б) (Vy > 0)3* (2х = у). 103. Из второго предложения следует первое. Вообще: Эл-Vy Р (x, у) => Vy3xP (x, у). 105. Обозначим через Р (x, у, г) высказывательную форму «В классе х ученик у решил задачу г», а) Vx3y3z Р (x, у, z); б) 3xVy3z Р (x, у, z); в) Vx3yVz Р (x, у, г); r) VxVy3z Р (x, у, г); д) 3zVx3y Р (x, у, z)\ е) 3xVyVz Р (x, у, г); ж) Vz Зх Vy Р (x, у, г). 106. а) (Зх € N) (Vy € N) (х < у); б) (Vx 6 N) (Зу 6 N) (x<y); в) (Vx € Q) (Vy € Q) (х = у v * > У V х < y)- 108. б), в),г), д), ж). 109. а) 0; б) 0; в) [—1; 1]; г) R-, д) ]1; <»[; е) [0; oo[. 110- а) Зу (х = 2y); б) 3y3z (х = у* + г2). 111. Указание. Представьте каждое неравенство в виде конъ¬ юнкции или дизъюнкции болёе простых неравенств. Например, *2~4 >0 e=> (х2—4 ^0Ax2 — 16>0)v(^-4<0A^-16 <0). X2 — 16 Найдите пересечения множеств решений неравенств, составляющих конъюнкции, а затем — объединение этих пересечений. 112. См. указание к 111. 113. Указание. Изобразите на координатной плоскости множе¬ ства истинности каждой из выскавывательных форм, составляю¬ щих конъюнкцию (дизъюнкцию); найдите пересечение (объедине¬ ние) этих множеств. 114. (Vx е NHP (x) v Q (x)). _ 115. a} ЗхЗуР (x, у); _ б) Vx3yP (x, у); в) 3xVyP (x, у); r) 3yVxP(x, у); д) Vy3xP (x, у); е) VxVy P(x, у). 116. а) Отрицание данного высказывания «Существует простоечет- ное число» истинно: 2—простоечетноечисло. Следовательно, данное высказываниеложно.б)Отрицаниеданного высказывания «Сущест¬ вует четырехугольник, диагонали которого взаимно перпендикуляр¬ ны, не являющийсяромбом»—истинное высказывание. Следовательно, данное высказывание ложно, в) «Существует иррациональный ко¬ рень уравнения |х| = —1» — ложное высказывание. Следователь¬ но, исходное высказывание истинно, г) «Существует элемент пустого множества, не принадлежащий множеству Мъ — ложное высказы¬ вание. Следовательно, исходное высказывание истинно, д) «Суще¬ ствует решение неравенства х2 < 0, не являющееся решением урав¬ нения х2 — 9 = 0» — ложное высказывание. Тем самым исходное высказывание доказано, е) «Существует квадратное уравнение, не 58
имеющее действительного корня» — истинное высказывание; та¬ ким уравнением является, например, х2 + 1 = 0. Тем самым ис¬ ходное высказывание опровергнуто. 117- а) (Vx € Q) (Зу € Q) (х + у = х); (Зх € Q) (Vy 6 Q\ {x + у Ф х). Существует рациональное число х, такое, что для всякого ра¬ ционального числа у х + у Ф х; (Л). б) (Зх € Л0 (Vy 6 N) (х + у = у); (Vx € Л0 (Эу € JV) (х + у- Ф у). Для всякого натурального числа х существует натуральное число у, такое, что х + у Ф у; (И). в) (Эх € Z) (Vy € Z) (х + у = 0); (Vx € Z) (Эу € Z) (х + у Ф 0). Для всякого целого числа х существует целое у, такое, что х + у ф 0; (И), д) Существует натуральное число х, такое, что для всякого натурального у ху фх; (И), ж) Существует число, не равное 0, произведение которого со всяким числом не равно едини¬ це (Л). з) Для всякого числа существует другое число, не равное 0, произведение которого с первым не равно 1 (И). 118. а) Существует город, во всяком районе которого есть шко¬ ла, в каждом классе которой есть ученик, имеющий хотя бы одну тройку, б) В каждом городе есть район, во всякой футбольной коман¬ де которого найдется игрок старше18 лет. в) Существует город, на к&ждой улице которого в каждом доме есть окно, выходящее не на юг. г) Во всякой книге есть страница, во всякой строке которой буква «ы» встречается меньше двух раз. д) Существует город, на каждой улице которого есть дом, в котором нет однокомнатных квартир. , 119. Пусть 1 — данная прямая на плоскости Р.хпринимает значения из множества точек этой плоскости, а yi и у2 принимают значения из множества прямых этой плоскости. Тогда аксиома па¬ раллельности запишется так: VxVyjVyj ((x g 1 Д х € Ух A х С у2 А Уь l| 1 А у2 II l)^-yi ,= у2). Отрицание аксиомы параллельности: вне данной прямой сущест- вует точка, через которую проходит более одной прямой, параллель¬ ной данной. 120. а) Функция f — периодическая «=> (3/ Ф 0) Vx (f (х — /) = = f (x) = j (x+ Q). б) (V/ ф 0)3x (/ (х — 0 Ф i (x) V / (Х + t) ф f (x)). 121. а) Допустим, что (3/ Ф 0)Vx (я2=(*+/)2). Нох2—(х + tf^> <=> 1 (2x ^- 1) = 0. При х Ф —^- 1 (2л? ^- /) Ф 0, т. е. (V/ Ф 0) 3x(x2 Ф (х + lf). Следовательно, функция / — неперио¬ дическая. 59
122. a) f четная <=» (Vx € D (f)) (-x С D (f) Л f (—х) = f (x)); б) f нечетная <=> (Vx € D (f)) (-x € D (f) Д/ (-x) = —f (x)); в) (3x t D tf)) ( x i D tf) V f (-*) Ф - / (x)). 124. в) Пусть X — множество четырехугольников; P (x) — х есть параллелограмм; S (x) — х имеет центр симметрии, тогда дан¬ ная теорема запишется так: (Vx £ X) (Р (x) ^- S (x)). д) Пусть Р — множество прямых на плоскости. Данная теоре¬ ма: (Va 6 Р) (V6 € Р) (Vc € Р) ((а || b Д b || с) ^ а || с); (И). Обратная теорема: (Va 6 Р) (V6 6 Я) (Vc € Р) (а |] c^- (а || b Д Л b || c)); (Л). Противоположная теорема: (Va € Р) (V6 € Р) (Vc € Р) ((a Jft b v 6 +Г с) ^- a +f с); (Л). Теорема, обратная противоположной: (Va € Р) (V6 € Р) (Vc £ Р) (a tf с ^- (a # b v V fr^fc)). Отрицание: (3a € Р) (ЭЬ € Р) (Зс € Р) (a || b Д b || с Д Aatfc). ДОПОЛНИТЕЛЬНАЯ ЛИТЕРАТУРА Г ж е г о р ч и к А. Популярная логика. М., 1979. К а л у ж н и н JI. А. Элементы теории множеств и математической логики в школьном курсе математики. М., 1978. К у т а с о в А. Д. Элементы математической логики. М., 1977. П у х н а ч е в Ю. B., П о п о в Ю. П. Математика без формул. Вып. 3. М., 1979.. . Столл Р. Множества. Логика. Аксиоматическиетеории. М., 1968. С т о л й р А. А. Как мы рассуждаем? Минск, 1968.
МНОЖЕСТВА НА КООРДИНАТНОЙ ПЛОСКОСТИ С координатной прямой, координатной пло¬ скостью и графиками учащиеся впервые знакомятся на уроках ма¬ тематики в V классе, а в VI классе у них формируется понятие об уравнений с двумя переменными и его графическом'представлений. В дальнейшем, на протяжении всего школьного курса математики, В соответствии е. программой рассматриваются простейшие виды функциональных зависимостей и их графики. Изучение материала, представленного в пособии, позволит учи¬ телю углубить и обобщить знания учащихся о графиках и геомет¬ рических преобразованиях и привести их всистему. Кроме того, появляется возможность более углубленно рассмотреть с учащими¬ ся приложения координатного метода. Изучение материала сопровождается решением значительного числа задач, что способствует развитию аналитического и графи- леского мышления и приобретению наЬыков, которые будут полез¬ ны при изучении любой отрасли естественно-математических наук. Наряду с задачами тренировочного характера предлагаются задачи на обоснование те^ или иных теоретических положений, углубляю¬ щие или расширяющие основное содержание темы. Тема рассчита¬ на HaJ6 ч. Примерное распределение учебного времени. Вывод уравнений линий по их геометрическим свойствам — 6 ч. . Графики и геометрические преобразования — — 4 ч. Построение графиков функций — — 5 ч. Зачет по теме— — 1 ч. ВЫВОД УРАВНЕНИЙ ЛИНИЙ ПО ИХ ГЕОМЕТРИЧЕСКИМ СВОЙСТВАМ О том, что линиям на плоскости соответствуют уравнения с дву¬ мя переменными; учащиеся узнают в связи с изучениемв курсе алгебры темы «Уравнения и неравенства с двумя переменными». Теперь эти представления закрепляются и углубляются. Прежде чем приступить к выводу уравнений линий по их гео¬ метрическим свойствам, надо вспомнить сведения об использова¬ нии метода координат при выводе уравнений линий, известных учащимся из основного курса. 6i
Нужно напомнить учащимся, что метод координат —это способ определять положение точки с помощью чисел, позволяющий описывать геометрические объекты на «языке чисел». Такое описа¬ ние бывает полезно, например, если нужно рисунок, составленный из точек, передать по телеграфу или сообщить вычислительной ма¬ шине. Учащиеся должны знать, что значение координаты данной точ¬ ки на данной прямой зависит от выбора положительного направле¬ ния, начала координат и единицы масштаба. Учащиеся должны понимать, что в выбранной системе коорди¬ нат каждой точке плоскости соответствует вполне определенная пара чисел х и у и, обратно, всякая пара чисел х, у определяет на плоскости единственную точку, абсцисса которой равна х, а орди¬ ната — у. Следует подчеркнуть, что задать точку — это значит за¬ дать ее координаты, а найти точку — значит найти ее координаты. Рассматривая с учащимися формулы расстояния между двумя точками на прямой и плоскости (пункты 1 и 2), следует обратить вни¬ мание на то, что хотя при их выводе обращаются к чертежу, одна¬ ко они не «привязаны» к положению точек, фиксированных на дан¬ ном чертеже, а справедливы во всей общности. Производя вычисле¬ ния по этим формулам, не нужно думать, какую точку считать первой, а какую — второй. Это совершенно безразлично, так как |*1 - Х2\ = |*г — *J И (*i — X2)2 = (*2 — Xj)2. В пункте 1 предлагается самостоятельно доказать формулу, выражающую координату середины отрезка. Для доказательства можно либо рассмотреть все возможные случаи расположения то¬ чек (концов отрезка), либо записать доказательство таким образом, чтобы оно годилось сразу же для всех случаев: | *cep Ху | | *cep *8 ( ^^ С^сер *l) С^сер *2)^ ^" 2<сер = *1 + *2 =>*cep = -1 ^^ *—• Формулы расстояния между двумя точками применяются при решении разнообразных задач и поэтому должны быть хорошо ус¬ воены учащимися. Этому может способствовать знание их словесных формулировок. Для точек прямой такая формулировка дана в по¬ собии; для точек плоскости — расстояние между двумя точками равно корню квадратному из суммы квадратов разностей одноимен¬ ных координат этих точек. Прежде чем перейти к изучению конкретных линий (прямой, окружности, эллипса и др.), полезно познакомить учащихся с об¬ щей постановкой задачи. Пусть на плоскости произвольно задана линия. Выберем в этой плоскости декартову систему координат и рассмотрим произволь¬ ную точку этой линии. Точка, перемещаясь по данной линии, ме¬ няет свои координаты; но изменяясь, они (координаты) остаются связанными некоторым условием, характеризующим линию как 62
множество точек. При нарушении этого условия точка «сходит» с линии. Уравнение, связывающее координаты, называется уравне¬ нием линии, если соблюдаются два условия: 1) координаты всякой точки, лежащей на линии, удовлетворя- кут этому уравнению; 2) координаты всякой точки, не лежащей на линии, не удовлетворяют этому уравнению. Таким образом, можно сказать, что точка задается парой коор¬ динат, а линия — уравнением с двумя переменными. Из общего курса математики учащимся уже известно, что мно- жебтво точек, равноудаленных от оси абсцисс на расстояние /, пред¬ ставляет собой прямую, параллельную оси Ох и пересекающую ось Оу в точке (0, /), и что уравнение этой линии имеет вид у — 1 = 0. Известно также, что множество точек, равноудаленных от начала координат на расстояние r, представляет собой линию (окружность), уравнение которой л? + у2 — г2 = 0. Полезно заметить, что не всякое соотношение между координа¬ тами х и у задает линию на плоскости. Например, уравнению х2 + у2 + 1 = 0 не удовлетворяют координаты ни одной точки пло¬ скости, а потому не существует линии, соответствующей данному уравнению. Это уравнение определяет пустое множество точек. Уравнению ху = 0 удовлетворяют координаты точек двух линий (осей координат). Уравнение х2+у2 — 0 определяет одну-един- ственную точку — начало координат. Из сказанного вытекает важность постановки двух задач: 1) дана линия как множество точек, составить уравнение этой линии; 2) дано уравнение, связывающее координаты х и у, построить линию, определяемую этим уравнением. Чтобы составить уравнение линии в декартовых координатах, нужно каким-то образом задать условия, определяющие ее положе¬ ние относительно координатных осей. В учебном пособии рассматриваются несколько видов уравне¬ ний, определяющих прямые линии. Изучение материала лучше на¬ чать с повторения уравнения прямой линии вида у = kx + L Это можно организовать, рассмотрев задачу 26, где требуется изобра¬ зить различные семейства прямых линий, задаваемые уравнения¬ ми этого вида. Из курса VI класса учащиеся знают, что угол на¬ клона прямой к оси абсцисс зависит от коэффициента k и что при одном и том же k прямые параллельны. Они знают, что при k > 0 угол наклона прямой к оси абсцисс острый, а при k < 0 — тупой и что 1 +~ ордината точки пересечения с осью ординат. Учащиеся умеют построить линию, определяемую рассматриваемым уравне¬ нием. Восьмиклассники умеют также решать и обратную задачу, т, е. они могут записать уравнение прямой, используя известные координаты ее точек пересечения с осями координат. Полезно вы¬ полнить два-три таких упражнения. Затем учащимся сообщается другой способ задания прямой: указание координат точки пересечения прямой с перпендикуляром 63
к ней, проведенным из начала координат. Далее выводится нормаль¬ ное уравнение прямой (это название происходит от слова «нормаль», равнозначного слову «перпендикуляр») и дается понятие об общем уравнении прямой. Следует обратить внимание учащихся на то, что любая прямая может быть задана нормальным уравнением и, обратно, всякое нор¬ мальное уравнение определяет прямую. Любое уравнение вида Ах + By + С — 0, в котором А и В не равны нулю одновременно, можно преобразовать в нормальное урав¬ нение; следовательно, всякое уравнение .такого вида определяет прямую. Чтобы общее уравнение прямой Ах + By + С = 0 преобразо¬ вать в нормальное уравнение, достаточно обе его части разделить на число УА2 + В2, взятое со знаком, противоположным знаку С. Полезно предложить учащимся преобразовать несколько общих уравнений прямых в нормальные уравнения, а затем построить эти прямые. При этом следует предусмотреть различные случаи. При¬ ведем примеры. Общее уравнение: Нормальное уравнение: а) Зх — 4y +-20 « 0, -х + 4y ~ 4 = 0. 5 5 б) Зх + 4y = 0, ^ х + 4^ — °. 5 о в) Зх = 5. х —— = 0. 3 С уравнением окружности с центром в начале координат восьми¬ классники уже знакомы. Вывод общего уравнения окружности (пункт 4) основан на использовании ее основного геометрического свойства: расстояние от любой точки окружности до центра по¬ стоянно/ Учащиеся должны уметь по известным координатам центра А (а, b) окружности и ее радиусу r записать уравнение окружности в виде (х — tif + O' — b)2 = t* (см., например, задачу 27), а также уметь привести к такому виду любое уравнение окружности. Целе¬ сообразно познакомить учащихся с условиями, необходимыми для того, чтобы уравнение второй степени с двумя переменными было уравнением окружности. Эти условия выводятся следующим об¬ разом. Переписываем уравнение окружности (х — с)а 4^ iy — b)* = гг в виде *8 Н- у2 — 2ax — 2by + а2 + b2 — гг = 0. Коэффициенты этого уравнения сравниваем с соответствующими коэффициентами общего уравнения второй степени сдвумя переменными Ах3 +Bxy+ + Су2 + Dx + Еу + F = 0: A = l, В = 0, C= 1, D = —2a, E=—2&,f=a2 + 62—r\ 64
Заметим, что в уравнении окружности: а) отсутствует член с дроизведением xy, б) коэффициенты при х и у равны. Если хотя бы Щящ> из этих условий не выполнено, уравнение не является уравне- фгём окружности. . Для того чтобы привести уравнение общего вида к виду (х — а)2+ ф.& — 6)а =' г2, достаточно: 1) сгруппировать члены, содержащие одноименные переменные, «заключить их в скобки; 2) свободный член перенести в правую часть; 3) каждое выражение вскобках дополнить до квадрата суммы ;йЛй разности двучлена, а сумму чисел, добавленных в скобках, Ьдшбавить к правой части уравнения; ' 4) записать выражения в скобках в виде квадрата суммы или разности двух чисел. Геометрическое свойство, используемое при выводе общего ^равнения окружности, присуще всем окружностям. Однако суще¬ ствуют окружности, имеющие свои, отличные от другихокружно- стей свойства. Большой интерес у учащихся вызывает обычно вьь зод уравненйя и построение окружности Аполлония. Задачу 35 и связанные с ней задачи 36—39 рекомендуется рассмотреть на занятий.* f Далее (в пункте 5) рассматриваются другие линии (парабола, &ллипс и гипербола), представляемые уравнениями второй степени с двумя переменными. Эти линии обладают общим свойством: для точек линий отношение расстояний е до точки F — фокуса и до йрямой / — директрисы постоянно. Число е называется эксцентри¬ ситетом линии. Вид линии зависит от значения эксцентриситета. Позже, при изучении геометрических преобразований учащиеся прзнакомятся с исследованием и упрощением этих уравнений. Одной из ва$<ных геометрических задач является задача отыс¬ кания точек пересечения двух данных линий (пуйкт 6). Эта задача может быть решена без чертежа, если известны уравнения данных линик ее решение сводится к решению системы этих уравнений. Поэтому для решения таких задач (например, задачи 45) достаточ¬ но наличия у учащихся умений решать системы уравнений с двумя переменными, которые формируются у них при изучении алгебры в VIII классе. Весь материал этой главы может быть изучен на трех занятиях. На. первом занятии рекомендуется повторить координатный ме- ТОД (задачи 1, 2, 3, 6, 9) и рассмотреть вывод уравнения прямой лйнии, а также решить задачи 26, 11, 14, 16, 17, 22. Остальные за¬ дачи можно предложить учащимся для самостоятельного решения дома (решение задач 23, 24 полезно обсудить). Второе занятие можно посвятить выводу уравнения окружности я решению задач 27, 2&, 29, 31, 34—39. Другие задачи можно пред- дожить учащимся решить самостоятельно. , Ha третьем занятии должны быть рассмотрены уравнения па¬ раболы, эллипса и гиперболы, а также задачи, связанные с отыс¬ 3 Заказ 43 65
канием точек пересечения линий и с изображением графиков урав¬ нений или неравенств (задачи 41, 42, 45, 46 (в, г, д, ж, з, и), 48, 49 (а, г), 50 (а, г, е). Остальные задачи учащиеся смогут решить дома. Задачи, предлагаемые в пособии, должны быть решены без об* ращения к чертежу, а с использованием аналитического выраже¬ ния геометрических свойств точек, составляющих рассматриваемые в задачах множества. Однакоиз методических соображенийсовсем отказываться от геометрических образов не следует, особенно по¬ лезно графическое изображение в целях самопроверки. ГРАФИКИ И ГЕОМЕТРИЧЕСКИЕ ПРЕОБРАЗОВАНИЯ Повороты и осевые симметрии на координатной плоскости изу¬ чаются в курсе геометрии VII и VIII классов. Здесь эти сведения углубляются и обогащаются новыми. Учащиеся должны уметь найти координаты образов заданных точек при данном преобразовании, а также при последовательном выполнении нескольких преобразований. Для композиции преобра¬ зований они должны суметь записать формулы, ее определяющие/ В пункте 1 пособия йаряду с известными учащимся^еометри- ческимипреобразованиями рассматривается новое преобразование— инверсия относительно окружности. Поэтому учащимся полезно вначале показать примеры преобразования конкретных фигур, отмечая простейшие свойства инверсии: 1) инверсия есть взаимно¬ однозначное отображение плоскости с удаленной точкой О на себя; 2) всякая внутренняя точка окружности (кроме точки О) отобра¬ жается во внешнюю; 3) окружность инверсии отображается на се¬ бя; 4)-прямая, проходящая через центр инверсии* отображается на себя; 5) всякая прямая, не проходящая через центр инверсии, ото¬ бражается на окружность, проходящую через центр инверсии (справедливо и обратное утверждение: всякая окружность, прохо¬ дящая через центр, отображается на прямую, не проходящую через центр), и другие. Прибор, позволяющий строить фигуру, инверсную данной в заданной инверсии, называется инверсором. Его учащиеся смогут изготовить самостоятельно. Так, инверсор Паселье состоит А из 6 стержней, соединенных шар¬ нирно (рис. 1); при этом отрез¬ ки ОА и ОВ конгруэнтны и че¬ тырехугольник AMBM'— ромб, точка' 0— неподвижная точка— центр инверсии. Если точка М описывает какую-нибудь фигу¬ ру, то точка M' будет описывать инверсную фигуру в инверсии с центром в точке Оикоэффициен- том инверсии, равным числу Рис. \ОА\2—IOiWP. Надо заметить, 66
что инверсня находит применение в технике для превращения кру¬ гового движения в прямолинейное. - Умение находить координатную запись обратного преобразова¬ ния понадобится при составлении уравнения образа линии. Одна- чКО пункт 2 может оказаться трудным для всех учащихся и не пред¬ назначен для обязательного изучения. Можно ограничиться час¬ тичным рассмотрением материала (примеры 1—3). Получен- йых знаний будет достаточно для исследования симметрии линий (пункт 3). Для уравнений параболы, эллипса и гиперболы в пункте 4 по¬ казано преобразование их уравнений путем рассмотрения линий в новой системе координат. Учащиеся должны пснимать, что в данном случае переход к новой системе координат осуществляется выббром начала координат в «удобной» точке и параллельным сдви¬ гом осей координат. При помощи анализа полученного уравнения 4инии выявляется ееформа. Для лучшего понимания свойствизу- яаемых фигур мoжrfo рекомендовать кнйгу А. И. Маркушевича «сЗамечательныефигуры» (Наука, 1978). В ней содержатся рекомен¬ дации к построению фигур и описание экспериментов, иллюстри¬ рующих свойство фокусов фигур. Эту кштгу можно предложить кому-нибудь из учащихся для подготовки доклада. Материал гла¬ вы рассчитан на два занятия. На цервом занятиирекомендуется рассмотреть пункты 1—Зи решить задачи в52, 60, 62. Остальные задачи можно предложить учащимся для самостоятельного решения. На втором занятии дол- ^кен быть рассмотрен материал пункта 4 и целесообразно решение задач 64, 67, 68, 69. Остальные задачи учащиеся смогут решить дома. ПОСТРОЕНИЕ ГРАФИКОВ ФУНКЦИЙ Часть материала этой главы знакома учащимся по общему кур¬ су алгебры. Изложенные в ней сведения углубляют знания уча¬ щихся и приводят их в систему. Надо обратить внимание учащихся на то, что известный им спо¬ соб построения графиков функций «по точкам» громоздок и к тому же не всегда приводит к цели, так как без предварительного иссле¬ дования функции могут быть пропущены характерные точки изо¬ бражающей ее линии: вершины, точки пересечения с осями коорди¬ нат и др.; может исказиться характер линии, например не выявлена ее симметрия. Поэтому построению графика должно предшествовать исследование общих свойств заданной функции, нахождение харак¬ терных точек графика и исследование поведения его на разных уча¬ стках между этими точками. Для лучшего усвоения каждого из рас¬ сматриваемых шести этапов исследования (пункт 1) рекомендуется использовать соответствующие наглядные представления, а для закрепления в сознании учащихся этапов исследования полдййю решить задачи 71, 73, 76, 77. Учащиеся должны хорошо усвоить 3* 67
ход исследования функции и только после этого строить ее график (задача 70). Для проверки правильности построения и для уточне¬ ния графика можно рекомендовать учащимся вычислить координат ты нескольких, наиболее характерных точек. При построении графиков целесообразнообратиться к вспомо¬ гательным приемам — геометрйческому преобразованию, «сложе¬ нию» и «умножению» графиков. В учебном пособии такие приемы рассматриваются на примерах построения графиков квадратичной и дробно-линейной функций (пункты 2 и 3). После нахождения характерных точек для более точного построения графика рекомен¬ дуется его вычерчивать не от руки, а с помощью шаблонов,'изго- товленных из плотной бумаги. Большой интерес, но и наибольшие затруднения вызывает у учащихся построение графиков функций, содержащихзнак модуля. Впервыесфункцией вида у = |х| учащиеся встречаются в VII клас- ce; ee график строится на основании определения абсолютной вели¬ чины. На факультативном занятии слёдует повторить это построе¬ ние и,используя геометрические преобразования, выполнить по¬ строение графиков фуркций вида: у = 1*1 — 1, у = |*| + 1, у = |лг '+ll, у — | X — 1|. Полезно также выполнить некоторые «действия» с построенными графиками.например построить график функции вида 'y=l* + -U+4*-H. В пунктё 4 приводится пояснение к построению графиков функ- ций,- содержащих знаки модуля. Сначала рекомендуется решить задачу 81, а затем приступить к построению графиков функций, указанных в задаче 80. Весь материал этой главы рассчй^ан на три занятия. Первое занятие надо-посвятить исследованиюфункций и по¬ строению графиков на основании выполненного исследования. Мож¬ но решить задачи 71, 73, 76 (а, в), 77 (а, в) и 70 (а, б,в, д.). Остав¬ шиеся задачи учащимся полезно решить дома. На втором занятии можно рассмотреть применение геометриче¬ ских преобразований к построению графиков функций, «сложение» и «умножение» графиков и решить задачи 78 (а, в, д, ж, з)и 79 (а, б). На третьем занятии рекомендуется рассмотреть.графики функ¬ ций, содержащих знак модуля, на примере функций из задач 80, 81; 82. ПРИМЕРНОЕ СОДЕРЖАНИЕ ЗАЧЕТА Вывод уравнений' линий по йх геометрическим свойствам 1. Что значит задать точку на прямой (плоскости)? Найти точку на прямой (плоскости). 2. Как найти расстояние между точками прямой (плоскости)? 68
3. Какие виды уравнения прямой вы знаете? 4. Как построить прямую по уравнению: а) нормальному; б) в Отрезках; в) общему? 5. Напишитеуравнение окружности. 6. Как по уравнению второй степени с двумя переменными уз- уять, является ли оно уравнением окружности и если является, #b каким образом по нему построить окружность? 7. Каким общим свойством обладают парабола, эллипс и гипер- 9&6ла? В чем различия между ними? #. Задачи типа: 14,17, 20, 27, 28, 46, 49. Графики и геометрические преобразования 9. Назовите известные вам виды геометрических преобразова* $йй. Охарактеризуйте каждое из них. Ю. Поккжите применение геометрических преобразований к исследованию симметрий линий по их уравнениям. 11. Как по уравнению второй степени узнать, уравнением ка¬ кой линии (окружности, эллипса, гиперболы или параболы) оно является? 12. Задачи типа: 52, 54, 62, 64, 67, 68, 69. Построение графиков функций 13. Задачи типа: 70, 78, 79, 80. Ответы, решения, указания к упражнениям .4. а) 5; б) 8; i) 17; г) 4; д) 3. 2. {3; 7}, {-7; -3*, {-4; —2}, {3}. 3. Ь4; 4[, ]-l; 3[, ]-oo;-l[U]3;+oo[, [4;6], ]-oo; 4[ U tf; + oo[, [3; 4] и [6; 7]. 4. В (-1), С (2), Е (5). 5. \АВ\ = \АО\ = 5. в. а) (—4, —2), (3, ^). (4, ^); б) (4, 2), (-3,8), (—4, 4); fi) (4, -2), (-3, -8), ir~4. -4); г) (2, -4), (8,3>, (4. 4); д) (-2,4), (-8, -3), (-4, -4). 8. а) М (11, ^); б) ЛГ(—13, 11). 9. Л0С£> — ромб. \АВ\ = 10, следовательно, (х — 8^ + (у + 3)2 = 100, \ВС\ — Ю.следователь- яо, (х — 10)2 + O' — 11)2 = 100. О т в е т. (16,3), (2,5). 10. V^6, VT7, K42. 11. KAx + ХВу + ХС = 0, ХА = cos а, i,B -sin а, iC — —р, ХаА2 + №Вг — cos2 а 4^ sin2 а = 1, или X* (i48 + В2).— 1. Считая Я, >0, получим ^ = —=L=b^-. Заметим, ' " Y,A* + В* Что знак X выбирается в зависимости от знака В (противоположен *МУ),
12. Пусть |ОМ| = а и М € Ох, |ОЛЧ = b и N Z Oy. Так как точка М (а, 0) принадлежит рассматриваемой прямой, то ее коор¬ динаты удовлетворяют уравнению Аа + С = 0, откуда а -. л с с Аналогично, b . Из первого уравнения А = ——, из второ- С го В = ;людставляя эти значения в данное уравнение прямой, ь получаем: —с £ — с 4- + 'c = о или - + L = i. а b а ^ b Уравнение прямой, записанное втакойформе, носит название ypae- нения в отрезках. 13. Заметим, что (аи bJ и (щ, Ь2) сутьсоответствен- но координаты.оснований перпендикуляров, опущенных из начала координат на данные прямые. Для доказательства того факта, что угловые коэффивденты параллельных прямых равны, предполо¬ жим, чТо kx ф k2t тогда система (у = *1^+ ьи {y = k2x + bn, имеет решение: х = AzrA, у = fc ь*~ь' + 6 fti-fej ' ^-fca т. е. имеется точка пересечения прямых, а это противоречит усло¬ вию. 14. |ЛШх|=|АШг|, (х— 1)2+ (у — 6)2 = (x'+l)* + (х — 4)г, откуда х + у — 5 -=0. 15. Указание. Воспользуйтесь условием] OAJ = \ ОВ\ = | ОС|, где 0 — центр искомой окружности. О (5,5; 4,5). 16. Указание. Убедитесь, что координаты точки Мх удовлетворяют данному уравнению при любом значении k. 17. Зх — у — 20 = 0. 18. Пусть прямая у —ух = k (х — x,), проходяш^я черезточку Мх (Xj, yi), проходит также через точку М2 (х2, у2). Тогда у2 — ух = k (х2 — Xi), отсюда k = -Уа ~.ffA. 4-*i 19. Указание. Подставьте координаты точек в уравнение рас¬ сматриваемой прямой. 20. Уравнения сторон: х + 8y — 11 = 0, 6x + 5y + 20 = 0, 5x — Зу — 12 = 0. Уравнения медиан: 7x + 13y + 9 = 0, 4x — 1 \у — I = 0, 5x — 2y — 8 = 0. 21. х — у + 2 = 0, Зх + 2y — 19 = 0, х + 4y — 3 = 0. 22. Пусть. М. (x, у) — произвольная точка рассматриваемого множества. Учитывая, что |ЛМ|2 — \ ВМ |2 = cP, получим: (х — хх)2 + O' — ytf — (х — х2)2 — (у — у2)2 = d? или 2x (х2 — xt) + 2y (у2 — y,) — d? + х\ + у\ — х\ — у\ = 0, т. е. получили уравнение вида Ах + By — С = 0. Следовательно, рассматриваемое множество точек является прямой. 23. Пусть Ot и 02 — центры данных окружностей ГL и Г2, гх и r2 — их радиусы 70
Рис. 2 (гя ^^i), М7\ и МТ2 — касательные к ним, проведенные из точки iW, принадлежащей искомому множеству. По условию \МТ^ — = ШТ^2. ЙспользуятеоремуПифагора, можно записать: |M0i|2— — I ОхТгР - |MO^ -\0№ или |М02|2 - \М0х\г = т\ — rf, следовательно, искомое множество точек принадлежит прямой, перпендикулярной 0Y0^ (см. задачу 22). 24. Указание. Докажите, что все точки рассматриваемой прямой обладают свойством точек У1 ) jff ^>/. л/у y4 м\/ ПУ ул% Г 4y^ NX, о ^A^>r V^ X^XV yf h )з Л X 7 / V=^ S^k=-f -9j Рис. 5 Рис. 6 71
ui (а.Ь) \ ~\ ^_J 0 х Рис. 8 Рис. 9 из задачи 23 (если А — одна из точек множества, то | OsA |2 — — \01А¥‘ — r® — ф. После этого воспользуйтесь решением задачи 23. -Множество точек — это объединение двух лучей прямой, про¬ ходящей через точки пересечения двух окружностей. 26. а) См. рис. 2; б) см. рис. 3; в) см. рис. 4; г) см. рис. 5; д) см.рис. 6. 27. хг + у2 — 2x —2% = 0. 28. а) Окружность с. центром в точке (—1, 0) и радиусом 1; б) окружность с центром в точке (—1, 0) и радиксом 2; в) точка(—1, 0); г) пустоемножество. 29. а)См. рис. 7; б) см. рис. 8; в) см. рис. 9. На каждом рисунке изображеца одна окружностьсемейства. 30. х2 + у2— 2ay=l. 31. Указание. а) Заметьте, что точки, задаваемыр уравнением у = У г2 — х2, удовлетворяют системе g>0 (JC® + уа = r3. .6) Выделите полный квадрат из трехчлёна, стоящего под кор- нем» и воспользуйтесь решением задачи а). 32. а) (х — 3)2 + O' — 4)2 = 25; б) не существует, так как точ¬ ки лежат на однои прямой. 33. Две окружности: х? + у2 — 34x — — 34y + 289 = 0 и дс® + у2 — 10* — 10y + 25 = 0. 34. В усло¬ вии задачи имеется пропуск. Его следует сформулировать так: до¬ кажите, что множество точек, сумма квадратов расстояний которых до точек А (—а, 0) и В (а, 0) равна с* является окружностью. Пусть М (дг, у) — точка рассматриваемого множества, тогда |МА2| + \МВг\ = с, {x + o)*^+ (х — af + 2y2 = с, откуда х2 + у2 = ■0 ~ 2а , т. е. данное множество точек является окружностью с центром в точке (0, 0) и радиусом у с — 2а® да: 35. Пусть М (x, у) — точка рассматриваемого множества, тог- LMi=X(X^l); ^ + ^tX>X2; \ВМ1 (* — “)а + У2 72
(х + о)2 + у2 = l2 (х — af + xy, - х2 (1 — X2) + 2ах (1 + Я2) + а2 (1 — Я2) + у2 (1 — Щ = 0; х2 + 2ax f +^ + а2 + у2 = О, 1 — А2 *_,.. („\ l — ^2\2 ■ 2 4<>W lx + a — +у2 — . \ l-*2 J * (l-tf)> Следовательно, рассматриваемое Множество является окруж- £рСТЬЮ. 36. Центр окружностй Аполлония в точке^—а | ^^ j^, oj, радиус (а > 0, Я > 0). U-^l Окружность Аполлония пересекает ось абсцисс в точках ^£гви*ёь 4> 37, См. рис.- 10. 38. Допустим, что окружности Аполлония с фиксированными точками А и В и различными значениями Xt и Х8 таресекаются в точке М. Тогда j^j, = Aj = ?12. Получено проти¬ воречие: Докажите, что- , 39. При X ^ 0 окружностй стягиваются к точке А. При Л тК+с© окружности стягиваются к точке В. Рис. 10 73
У{ y=x/ 7- AS*=i ^ 0 х 'У л Рис. 15 Рис. 16 у\ У| 7 / Г ^ -1 “0 1 ^T -7 ^ 2 7 *x “ -/ Рис. 18 Рис. 19 41. Указание. Надо показать, что для эллипса верно равенство / ер У = / ер , \з / е2р \2 lll-e*l/ \У"| I —<sa V \| I — е81/ ’ а для гиперболы — равенство ( вар \а ^ / ер \2 / ер \а ll 1 — е“ I/ Ml-e*lj \^ I 1—е* I/ ' 42. 2ep. 74
43. Для эллипса р = —. b - - е = J^ /а? — *2 » для гиперболы р = ^=====, е = V_^_^J^_ 44. о = -, b = сК!.Ге1. е с
Рис. 29 45. а) (7, 2) и (2, 7); б) (12, 5), (12, -5), (-12, -5), (-12, 5); в). (5, 2), (-5, -2), (2, 5), (-2, -5); г) (4, 3), (—3, -4); д) (1, 1>, (|, I). 46. а) См. рис. 11; 6)tM. рис. .12; в) см. рис. 13; г) см. рис. 14; д) (х — 2y) (лг — у) = 0, см. рис. 15; е) (х — Зу) (х — у) = 0, см. рис. 16; ж) см. рис. 17; з) (х2 :— — 1) (у2 — 1) = 0, см. рис. 18; и) ом. рис. 19.47. л® —у2 + 2Ax + 2By+C = = 0, тогда (х + А)3— O' — В)г — А2 + +B2 + С = 0, (x-y + А + B) (x + y+ +A — В) — (Л? — В2 — С) = 0. При С = Л2 — В2 графиком является объ¬ единение двух прямых. 48. Например, YxYyY^—xY2 — y= 0; это уравнение равносильно х — 0 V y'= 0 v х = 2 V У ~ 2 и fO<x<2, lO<y<2. ' 49. а) См. рис. 20; б) см. рис. 21; в) см. рис. 22; г) см. рис. 23. 50. Для Л fl В. а) См. рис. 24; б) см. рис. 25; в) см. рис. 26; г) см. рис. 27; д) см. рис. 28; е) см. рис. 29. 51. Воспользуйтесь «геометрическим» определением, инверсии. Другой способ ^ алгебраический; покажите, что последователь¬ ное. выполнение преобразований инверсии по формулам 12 при¬ водит к точке (x, у). 52. Точка Л (5; —2) отображается в точку Л'' (11; 16), а точка В (0; 8) — в точку B' (—29; —4). Заметьте, что всепреобразования переводят отрезок в отрезок. 53. fx’ = х2 — у2, ((x'f = х* — 2*V + У4, (y' = 2xy. l002 = 4х2у2. j_ _ Откуда (л:')2 + (у')а = (л? + у2)2, т. е. х2 + у2 = Y(*')2 + fy')2- Из. первого уравнения: у2=л?—л', а поэтому Xs Xй = у2 + я', а поэтому у! _ V(x')* + (yy-x' _ W)*+foT + *' 2 . При y'' > 0 х = -Ym -У У + (y'f +X' , у = = yvK при y' s^ 0 х — v (*у+юг+х’ )*+W)*-X' 54- f ° 8 = fx' = (х — у)3, 1 У' = (х- g°f у) (х2 + 4xy + у2). fx' = 2y (Зх2 + у2), = [y' = —6y (х2 — у2). Отображения f о gu g о f не перестановочны. 76
55. а) Прямая Ах + Ву — 0 отображается на прямую + B_iV i0 1 t r\п i У /Ч.О ' (*')а + (y'f (*')2 + (y')* i3. е. на Ax' + By' = 0 (на себя), -б) Ах + By + С = 0 (C^= 0), А—т¥- + в ^ +C = 0, (JO* + Ofl* М2 + (yV с ((x'f + ty')2) + Ar*x' + Br*y' = 0, r* (Л» + В2) 4C* * а 'это уравнение окружности. в) Указание. Воспользуйтесь решением задачи б). __r) Указание. Примените инверсию к окружности, не проходя¬ щей через центр инверсии. 56. [—; 1]. 57. х'2 — у'1 — 4. 59. Последовательное выполне- 4. йие симметрий относительно прямой у = х, а затем относительно оси ординат задается условиями (x' = —у, ly' = *. ~ а поэтому образом заданной прямой, является прямая Ay — Вх + ,+ С = 0. 60. Заметьте, что'прямая и ее образ в задаче 59 всегда. Перпендикулярны. Воспользуйтесь условием параллельнести пря¬ мой Агх + Вху + Сх = 0 и образа другой прямой —В2х + А2у + Чг С2 = 0, тогда —■ = Y< откуда АХА2 + B^B^ — 0. - 61. х + - —&2 2 + Зу + 3 = 0. 62. а) Четыре оси симметрии: _ оси координат и прямые у = dzx; б) две оси симметрии — оси координат; в) четыре оси симметрии: оси координат и прямые у = ±х; г) ось симмет¬ рии — прямая у — х; д) нет симметрий; е) две оси симметрии: оси координат; ж) ось симметрии — прямая у — х\ з) нет симметрий. <63. Указание. Заметьте, что графики обоих неравенств симмет¬ ричны относительно осей координат. ’ 64. а) jg + у— — 1; б) ^ + Y^ = 1; в) воспользуйтесь резуль¬ татом задачи 44, ^ + £ = 1; г) g + f = 1. 65. При уменьшении b от а до 0 фокусы расходятся, а директри¬ са приближается к оси. 66. Если и и v — длины, отсекаемые на осях Ох к Оу концами данного отрезка, то для любой точки отрезка U — X у . t X у и можно записать = — или I — = k. и V и V 77
78
67. 1; 1. а) £-£ = 16 9 68. См. рис. 30. 69. а) (х + 3)2 + 4 ty- — 2)2 = 25; б) 9 {x + I)2 + 16 (у + 1)2 = = 144; в) (х + 4)2 — ty + 3)2 = 7. 70. а) См. рдас. 31; б) см. рис. 32; в) см. рис. 33; г) см. рис. 34; д) см. рис. 35; е) см. рис. 36; ж) см. рис. 37; з) см. рис. 38. 71. а) Нечетная; б) четная; в) четная; г) ни четная, ни нечетная; д) четная; е) четная. 72. Воспользуйтесь определением четной (нечетной) функции. 73. а) у = (5x4 + 2ха + 2 х + 1) + .+(-3*3 - 7x) ; + + в) у = '*3- 4 = -- ( 2 V Xя— 4 -r>-4' L{ 2 {■ х2 + 8 —хь — 4 )• ! + 8 + + Рис. 38 ,x2 + 8 х2 + 8 74. Пусть f (x) = ф (x) + Я (x), где ф (-x) = ф (x), а Н = —Н (x). Имеем систему уравнений (-x) Отсюда: ГН*) = Ф(х) + Д(х), 1 f (—*) = Ф (*) — Н (*)• <P (*) = ~ (f (*) + / (—*)), Н (x) = itf(x)-/(-x)). 76. а) Полюс х — —2. X —2 — 0 —2 + 0 У + 00 —оо б) Полюсы х — —3; 1. X _3_oJ-3 + 0 1 — 0 1 +0 У —оо +oo —оо +oo 79
в) Полюсы х = ±2; ±3. X —3 — 0 ^3 + 0 —2 — 0 —2 + 0 2 — 0 2+0 3 — 0 3 + 0 У оо +0° +oo —оо +oo —оо —оо +oo г) Полюс х = —1. X 1—0 i4-o 2 — 0 2 + 0 3—0 3 + 0 У —оо +oo +oo —оо —оо +eo 77. а) X —оо +oo У 3 — 0 3 + 0 х —OO +QO У; —0 - +0 х —оо +oo У —оо +oo X —оо - +о° ' У 1—0 1+0 78. а) См. рис. 39; б) см. рйс. 40; в) см. рис. 41; г) см. рис. 42; д) см. рис. 43; е) см. рис. 44; ж) см. рис. 45; з) см. рис. 46. 79. а) См. рис. 47; б) см. рис. 48; в) см. рис. 49; г^см. рис. 50. 80. а)См. рис. 51;б)см. рис. 52; в)см. рис. 53; условиеследуетчитать так: у = | \х — 11 — 11; г) см. рис. 54; д) см. рис. 55; е) см. рис. 56; ж) см. рис. 57; з) см. рис. 58; и) см. рис. 59; к) см. рис. 60. 81. См. рис. 61; 6) см. рис. 62; в) см. рис. 63; г) см. рис. 64; д) см. рис. 65; е) см. рис. 66; ж) см. рис. 67. 82: а) См. рис. 68; б) см. рис. 69; в) см. рис. 70; г) см. рис. 71. 80
81 82
84
г лх, Рис. 71 ДОПОЛНИТЕЛЬНАЯ ЛИТЕРАТУРА Б р о н ш т е й н И. Н. Эллипс. — Кваит, 1975, № I, с. 2. Гипербола. — Квант, 1975, № 3, с. 16. Парабола. — Квант, 1975, № 4, с. 9i Г е л ь ф а н д И. М., Г л а г о л е в а Е. Г. и др. Метод координат. 5^ изд. М., 1973. Г е л ь ф а н д И. М., Г я а г. о л е в а Е. Г. и др. Функции и графики. &* изд. М., 1973. Г у*т % н м а х е р В. -Пантограф. — Квант, 1977, № 10, с. 55. Дополнительные главы по курсу математики. Учебное пособие по факульта¬ тивному курсу для учащихся 7—8 классов /Сост. К. П. Сикорский. М., 1974. Дополнительные главы по курсу математики 9 класса для факультативных занятий /Сост. П. В. Стратилатов. М., 1970.
БЕСКОНЕЧНЫЕ МНОЖЕСТВА Содержание темы представляет собой продол¬ жение и развитиетеоретико-множественной линии, пронизывающей основной курс математики. При изучении алгебры и геометрии уча¬ щиеся имеют дело как с конечными, так и с бесконечными множе¬ ствами, производят операции над ними, рассматривают отношение «быть подмножеством». При изучении темы «Бесконечные множе¬ ства» на факультативных занятиях раскрываются специфика бес¬ конечных множеств, их удивительные свойства. На этом интерес¬ ном, увлекательном материале углубляются известные учащимся из основного курса понятия отношения эквивалентности, отноше¬ ний «больше», «меньше», развивается понятие числа, расширяется представление о математической абстракции бесконечности. . Содержание темы, на изучение которой отводится 12 ч, естест¬ венным образом расчленяется на следующие смысловые «блоки»: Мощность множества (пункты 1—4) — 6 ч. Множества мощности континуума (пункты 5—7) — 4 ч. Несуществование множества наибольшей мощности (пункт 8) — 1 ч. Один час отводится на зачет по теме. Сведения из истории (пункт 9) по усмотрению учителя можно рассказать учащимся на первом занятии, либо после изучения темы, либо распределить на несколько занятий, выбирая подходящие фрагменты. Пункт 8 дается в ознакомительном норядке. МОЩНОСТЬ МНОЖЕСТВА О с н о в н ы е п о н я т и я: взаимно-однозначное соответст¬ вие, эквивалентные множества, мощность множества, счетное мно¬ жество, несчетное множество, отношения «^», «<» на множестве мощностей. В с п о м о г а т е л ь н ы е п о н я т и я: правильная часть множества, кортеж. Центральным в этом блоке является фундаментальный резуль¬ тат теории множеств: «Всякое бесконечное множество эквивалентно некоторой своей правильной части». Этому утверждению предше¬ ствуют теоремы о счетных множествах: 1) любое бесконечное под- 86
множество счетного множества счетно; 2) любое бесконечное мно¬ жество содержит счетное подмножество. Отдельно рассматривается (без доказательства) теорема Кантора — Бернштейна: если из двух множеств- Каждое эквивалентно подмножеству другого, то такие множества эквивалентны. Начать изучение темы целесообразно с введения понятия взаим¬ но-однозначного соответствия. В пособии это понятие вводится как соответствие, порождаемое обратимым отображением множест¬ ва А на множество В (понятие обратимого отображения известно учащимся из основного курса).Далеевводится понятиеэквивалент- ных множеств как множеств, между элементами которых сущест¬ вует взаимно-однозначное соответствие. % После этого целесообразно ввести понятие мощности множества как общего свойства всех эквивалентных друг другу множеств. Мощность множества А обозначается так: | А |. Мощность конечного множества есть число его элементов. После введения понятия «мощность множества» наряду с термином «эквивалентные мндже- ства» можно в качестве его синонима употреблять термин «равно¬ мощные множества». Естественно поставить перед учащимися вопрос: можно лисрав- цивать мощности бесконечных множеств так же, как мы сравнива¬ ем мощности конечных множеств, т. е. целые неотрицательные чис¬ ла? Ответу на этот вопрос и посвящено в основном содержание дан¬ ной темы. Сначала выясняются сходство и различие между равномощностыо конечных и бесконечных множеств. В обоих случаях отношение равномощности обладает свойствами рефлексивности, симметрич¬ ности и транзитивности. Однако только для бесконечных множеств имеет место такой удивительный факт: бесконечное множество мо¬ жет быть равномощно своей правильной части, т. е. подмножеству, не совпадающему со всем множеством. Это можно продемонстриро¬ вать учащимся на ряде ярких примеров (см. упр. 4, 5). Уяснив су¬ щество этого факта, учащиеся будут подготовлены к пониманию следующего определения: если множество А равномощно подмно¬ жеству В' множества 5, то мощность А не больше мощности В (fi4l< |В|). Далее вводится понятие с ч е т н о г о множества. На примерах счетных множеств конкретизируются рассмотренные ранее отноше- £шя равномощности и |Л|^|5|. Сначала целесообразно рассмот¬ реть простые примеры установления счетности множеств путем ну¬ мерации его элементов, т. е. установления взаимно-однозначного соответствия между элементами данного множества и множества натуральных чисел. Для этой цели удобно взять множества четных чисел, нечетных чисел, целых чисел, множество чисел, кратных некоторому заданному числу, множество rt-x степеней натуральных чисел, множество натуральных степеней какого-нибудь числа ~ят. п. Вболеесложных случаяхцелесообразно использовать «метод числовых кортежей», описанный в пособии. 87
Затем доказываются две теоремы: 1) любое бесконечное подмно¬ жество В счетного множества А счетно; 2) любое бесконечное мно¬ жество А содержит счетное нодмножество. Из пбрвой теоремы и тео¬ ремы о счетности множества числовых кортежей следует, в частно¬ сти, что всякое бесконечное подмножество.множества числовых кор¬ тежей счетно. Этот результат используется при доказательстве счетности множеств методом кортежей^ Из второй теоремы вытека¬ ют утверждения фундаментального характера: 1) мощность любого бесконечного множества не меньше чем мощность счетного множе¬ ства; 2) любое бесконечное множество равномощно своей правиль¬ ной части. Приведем доказательство второй теоремы. Каждое бес- конечное множество М содержит счетное подмножество. Пусть это будет А = {alt а2, •••> ял» •■•}• Разобьем А на два счетных подмно¬ жества: ^Ai ^ %» ^3> ^5»,а’ И А2 6^2» ^4> ^i• • ■ * Между элементами А и Аг можно установить взаимно-однозначное соответствие. Это соответствие можно продрлжить до взаимно-од- нозначногосоответствия между элементами множеств А U (М\ A)= = M и Аг U (М \ А) = М \ А2, например поставив в соответствие каждому элементу из М\Л сам этот элемент. Но множество М\А2 — правильная часть М. Теорема доказана. Рассмотрев счеТные множества, естественно поставить перед учащимйся вопрос: а есть ли бесконечные множества, не являю¬ щиеся счетными? Ответом на этот вопрос служит доказательство несчетности множества точек отрезка. МожеМ ли мы утверждать, что мощность множества точек отрезка больше мощности счетного множества? Пока нет, так как отношения «больше», «меньше» для мощностей бесконечных множеств еще не определены. Учащиеся уже знают, что значит мощность множества В не больше мощности множестваЛ.Такговорятвтомслучае,если|£| = [Л^гдеЛхсзЛ. Логически существует четыре возможности: 1) В равномощно подмножеству А, но Л-не равномощно ника¬ кому подмножеству В; 2) В не равномощноникакому подмножеству Л, и А равномощно подмножеству В; 3) В равномощно подмножеству 4> и А равномощно подмноже¬ ству fi; 4) ни одно из множеств А и В не равномощно никакому подмно¬ жеству другого. Для конечных -множеств А и В реализуются только первая и вторая возможности. В первом случае |В|< | Л[, во втором |Л|< <|В|."Для бесконечных множеств может иметь место и третья возможность. Четвертый случай невозможен и для бесконечных множеств. В случае 3, т. е. когда каждое из множеств А и В равно¬ мощно подмножеству другого, А и В равномощны (согласнотеоре- ме Кантора — Бернштейна, формулировка которой дана в посо¬ бии). 68
Сообщив учащимся теорему Кантора — Бернштейна без доказа- дельства (если |Л| ^ |В| и ]В| ^ |Л|, то |Л| = |В|), естественно перейти к определению: «|Л| < |Д|, если А равномощно подмно¬ жеству В, но не равномощно 5». Теперь мы можем сказать, что мощность счетного множества меньше мощности множества точек отрезка, так как они не равномощны и множество точек отрезка .как всякое бесконечное множество имеет счетное подмножество. Таким образом, на множестве мощностей введены отношение «равенства» и отношение «меньше». Про любые две не равные мощ¬ ности можно сказать, что одна из них меньше другой. Наименьшую мощность среди бесконечных множеств имеет счетное множество. В конце темы учащиеся узнают, что не существует бесконечного множества наибольшей мощности. МНОЖЕСТВА МОЩНОСТИ КОНТИЙУУМА О с н о в н ы е п о н я т и я: мощность континуума, иррацио¬ нальное число, действительное число, алгебраические и трансцен¬ дентные числа. * 1 Здесь рассматриваются различные примеры множеств мощности континуума, т. е. множеств, равномощных множеству точек отрезКа £0; I]. Доказывается равномощность множеств точек любых отрез¬ ков, открытых промежутков, прямой, квадрат Полезйо сообщить :учащимся, что множества точек плоскости и пространства также тимеют мощность континуума. ~~ JB добавление к тем сведениям о рациональных числах, которые учащиеся получают при изучении основного курса, следует отме¬ тить: , 1) поскольку множество рациональных чисел счетно,., а множе¬ ство точек координатной прямой имеет мощность" континуума, то на прямой есть точки, не соответствующие никаким рациональным числам; эти «пробелы»заполняются новыми числами — иррацио¬ нальными; 2) множество точек координатной прямой, которым не соответ¬ ствуют никакие рациональные числа, имеет мощностб континуума; это следует из теоремы: «Если множество А бесконечно, а множест¬ во В конечно или счетно, то множества А U ^ и А равномощны». Значит, множество иррациональных чисел имеет мощность конти¬ нуума, т. е. иррациональных чисел «гораздо больше», чем рацйо- нальных. В то 'же время иррациональных чисел «столько же», сколько иррациональных вместе с рациональными. Наряду с .разбиением множества действительных чисел на ра¬ циональные и иррациональные вводится другая классификация :множества R: алгебраические числа и трансцендентные числа. До¬ казывается, что множество алгебраических чисел счетно, из чего следует, что множество трансцендентных чисел имеет мощность континуума. 89
НЕСУЩЕСТВОВАНИЕ МНОЖЕСТВА НАИБОЛЬШЕЙ МОЩНОСТИ В качестве подготовки к рассмотрению теоремы, составляющей содержание этого блока, следует предложить учащимся найти чис¬ ло всех подмножеств одноэлементного, двухэлементного, трехэле¬ ментного множеств. Для одноэлементного множества {a} имеем два подмножества: 0, {a}. Для двухэлементного множества {a, b} имеем 4 подмножества: 0, {a}; {b}; {a, b}; для трехэлемент¬ ного — 8 подмножеств и т. д. Получаем бесконечную возрастающую последовательность с общим членом 2п, где п — число элементов (мощность) исходного множества. После этого естественно перейти к вопросу о мощности множе¬ ства всех подмножеств бесконечного множества. В пособии доказы- ваетсятеорема: «Множество всех функций, заданных на множестве R и принймающих значения 0 и 1, имеет мощность, большую, чем \R\». Отсюда следует, что существуют множества, имеющие мощ¬ ность, большую, чем мощность континуума. Можно предложить учащимся доказательство более общего" ут¬ верждения: «Множество всёх подмножеств данного множества М имеет мощность, большую, чем |М!» (это утверждение справедливо как для конечных, так и для бесконечных множеств). Приведем до¬ казательство. Пусть даны множество М и множество Р его под¬ множеств. Докажем, во-первых, что |Af| ^ |Р| и, во-вторых, что \М\Ф\Р\. - Назовем Рг множество одноэлементных подмножеств множества М\ Pi c= Р и IPil= |М|.Следовательно, \М \ ^ |Р|.Предположим, что \М\ = |Р|, т. е. что между элементами М и Р установлено взаимно-однозначное соответствие:* а € М соответствует А € Р; b € М соответствует В € Р й т. д. Докажем, что существует такое подмножество X множества М, которое не совпадает ни с однйм из множеств Л, В, С, ... и т. д. Каждое из множеств Л, В9 С, ... ли¬ бо содержит соответствующий ему элемент множества М, либо не содержит его. Те множества, которые .содержат соответствующие им элементы, будем называть черными, а множества, не содержа¬ щие соответствующие им элементы, белыми. Составляем множество X из тех элементов множества М, которые соответствуют белым множествам. Множество X является подмножеством множества М, т. е. элементом Р (X с= М и X С Р). Покажем, что множеству X не может соответствовать никакой элемент из М. Допустим, что такой элемент х существует. Каким тогда будет множество X — белым или черным? Допустим, что X белое множество, т. е. х $ X. Тогда, по определению X, х € X. Полученное противоречие показы¬ вает, что X не является белым множеством. Допустим теперь, что X — черное множество, т. е. х€ X. Тогда по определениюХ х£ X, т. е. множество X не является и черным. Отсюда следует, что эле¬ мент х множества М, соответствующего его подмножеству X, не существует, т. е. взаимно-однозначное соответствие между элемен¬ тами множеств М и Р установить нельзя. Тем самым доказано, что 90
|Af| ф |Р|. Ранее было доказано, что \М \ < |Р|. Следовательно, |АЛ < \Р1 Итак, для каждого множества существует множество, мощ¬ ность которого больше мощности данного множества. Следователь¬ но, м н о ж е с т в а н а и б о л ь ш е й м о щ н о с т и ц е с у- щ е с т в у е'т. Создателем теории множеств Г. Кантором было высказано пред- гголожение, вошедшее в историю математики под названием «кон¬ тинуум-гипотеза». Оно заключалось в том, что не существует мно¬ жества, мощность которого больше мощности счетного множества, HQ меньше мощности континуума. В обобщенном виде «кбнтинуум- гипотеза» формулируется так: для любого множества М первая мощность, превосходящая Ш|, есть мощность множества всех подмножеств множества М. В 1900 году немецкий математик Давид Гильберт поставил проблему проверки «континуум-гипотезы», на первое место среди нерешенных математических проблем. В 1938 году немецкий логик Курт Гёдель доказал, что «континуум-гипоте¬ за» не может быть опровергнута традиционными средствами мате¬ матики. В 1966 году американец Поль Коэн доказал независимость «континуум-гипотезы» от остальных аксиом теории множеств. Та¬ ким образом проблема проверки «континуум-гипотезы» решена пол¬ ностью: это утверждение нельзя ни доказать^ни опровергнуть средствами теории множеств. ПРИМЕРНОЕ СОДЕРЖАНИЕ ЗАЧЕТА 1. Приведите пример взаимно-однозначногосоответствия между множествами: а) конечными; б) бесконечными. 2. Приведите примеры соответствия, не рвляющегося взаимно¬ однозначным. 3. Что такое мощность множества? 4. Какова мощность множества: а) четных чисел, не превосхо¬ дящих 100; б) натуральных чисел, не-превосходящих 100, в записи которых есть хотя бы одна цифра 9? ■ '' 5. Какие множества называются эквивалентными? Приведите примеры эквивалентных множеств: а) конечных; б) бесконечных. 6. Какое множество называется счетным? Приведите примеры счетных множеств^ 7. Докажите, что множество целых чисел счетно. 8. Докажите, что множество числовых кортежей счетно. 9. Что можно утверждать о мощности множества, являющегося а), пересечением; б) объединением конечного или счетного числа счетных множеств? Приведите примеры. 10. Приведите примеры бесконечного подмножества счетного множества. 11. Что можно сказать о мощности бесконечного подмножества счетного множества? 01
12. Докажите, что всякое бесконечное множество имеет счетное подмножество. 13. Может ли конечное множество быть эквивалентным своей правильной части? 14. Докажите, что любое бесконечное множество эквивалентно своей правильной части. 15.' Что значит:|Л| ^ |В|? 16. Сформулируйте теорему Кантора — Бернштейна. 17. Что значит: |Л| < |В|? 18. Счетно или несчетно множество чисел:_ а) рациональных; б) иррациональных;.в) действительных? 19. Счетно или несчетно множество точек: а) отрезка; б) пря¬ мой? 20. Как называется мощность множества точек отрезка [0; 1]? 21. Установите, какие из данных множеств имеют мощность континуума: а) множество точек отрезка [—1; 1]; б) множество то¬ чек луча; в) множество точек прямой; г) множество точек окруж¬ ности; д) множество-точек- квадрата; е) множество точек координат- 1 ' ной прямой с координатой вида —, где п € N; ж) множество точек Л плоскости с целочисленными координатами; з) множество точек плоскости; и) множество точек пространства; к) множество всех подмножеств множества целых чисел; л) множество всех функций с областью определения [0; 1]. 22. Какие числа называются алгебраическими? Какие числа называются трансцендентаыми? • 23. Докажите, что: а) множество алгебраических чисел счетно; б) множество трансцендентных чисел имеет мощность континуума. 24. Существует ли бесконечное множество: а) наименьшей мощ¬ ности; б) наибольшей мощности? . 25. Докажите теорему о существовании множества мощности, большей мощности данного множества. На зачете следует предложить следующие задачи из пособия: 4—13, 15, 18—20, 23—26, 29 или им подобные. Ответы, решения, указания к упражнениям 1. Каждому элементу множества А поставим в соответствие этот же элемент; очевидно, что такое соответствие взаимно-однозначное, следовательно, любое множество эквивалентно самому себе. 2. Ес¬ ли существует взаимно-однозначное соответствие между элемента¬ ми множеств А и В, то оно же является взаимно-однозначным со¬ ответствием между элементами множеств В и А, следовательно, еслй А ~ Б, то и В ■— А. 3. Пусть установлены взаимно-однознач¬ ные соответствия между элементами множеств А и В .и элементами множеств В и С. Установим соответствие между элементами мно¬ жеств А и С: каждому а € А поставим в соответствие такое с € С, чтобы при этом в первом соответствии а было сопоставлено с b, а 82
во втором — Ь с Cz Следовательно, А ~~ С. 4. Указание. Каждому 'ii € N поставьте в соответствие' п2 € В. 5* Указание. Каждому n € Л/ поставьте в соответствие 10* € В. 6. Указание. Каждой точке на плоскости поставьте в соответствие круг единичного радиу¬ са с центром в этой точке. 7. Нет, так как одна и та же точка на плоскости является центром многих окружностей. 8. Указание. Каждой окружности на плоскости поставьте в соответствие вписан¬ ный в нее квадрат со сторонами, параллельными координатным осям. Вместо вписанных квадратов можно взять описанные. 9. Ука¬ зание. Соответствие устанавливается ортогональным проектирова¬ нием полуокружности на диаметр. 10. Указание. Соответствие yc- танавливаетсяпроектированиемточек параболыу = х2 на осьабсцисс. 11. Из трех данных случаев отображение*^*2 задаетъзаимно-одно- значное соответствие только между теми множествами то¬ чек, для которых * ^ 0 и у ^ 0. 12. Указание. п € N по¬ ставьте в соответствие 2л. 13T. Берем вспомогательный отрезок [Л, В], параллельный и конгруэнтный отрезку[а, 6] коор¬ динатной прямой, и параллельным проектированием' устанавли¬ ваем взаимно-однозначное соответствие между точками отрезков [а, b^] и [4, В]. Затем устанавливаем взаимно-однозначное соответ¬ ствие между точками отрезка [Л, В] и отрезка [с, d] координатной прямой центральным проектированием, если [а, Ь] и [с, d] не кон¬ груэнтны, и параллельным проектированием, если [а, b] и [с, d] конгруэнтны. 14. Каждой точке плоскости соответствует бесконеч¬ ное множество окружноетей с центром в данной точке. Поставив в соответствие каждой точке плоскости одну из таких окружностей, получим взаимно-однозначное соответствие между множеством то¬ чек плоскости и подмножеством множества окружностей. Следова¬ тельно, мощность множества точек плоскости не больше мощности множества окружностей на плоскости. Г5. Каждому положйтель- ^roM^ чидлу соответствует бесконечное множество окружностей с ‘радиусом, длина которого равна данному числу. Взаимно-одно¬ значное соответствие получим, если, например, каждому пояожи- тельномучислу поставим в соответствие окружность с центром (0, 0) и радиусом, длина которого равна этому числу. Таким обра¬ зом, множество i?+ эквивалентно подмножеству множества А всех окружносуей на плоскости, т. е. |Я+ | ^ \А\. 16. Указание. Доста¬ точно доказать, что множество точек стороны квадрата эквивалент¬ но множеству точек а) -другой его стороны; б) днагонали, 17. По условию А — В'\ где В' cz В и В — C', где С' cz С. Из множества ,C' выделим подмножество С", состоящее из тех элементов, которые соответствуют элементам В'\ тогда будем иметь А ~ В' и В ~ С", откуда (по свойству транзитивности эквивалентности множеств) А ~ С'\ причем С" с С. Следовательно, \А \ ^ |С|. Докажем те- керь, что А не эквивалентно С. Допустим, что А эквивалентно С, S’, е. между элементами А и С существует взаимно-однозначное со¬ ответствие. По условию В ~ C', где С* cz С. Выделим из А под¬ множество Л' тех элементов, йоторые C00TBeTCTBjrc>T элементам C';
тогда будем иметь В — С и С' ~~ А\ откуда В — Л\ Из В ^ А\ где А' cz А и Л — В\ где В' с В9 следует, что Л ~ В (по теореме Кантора — Бернштейна), а это противоречит условию. Следова¬ тельно, |Л| =т^= |Cj. Теорема доказана. 18. а) Указание. Каждой точке плоскости Л (p, q), где р и q — четные числа, соответствует числовой кортеж (p, 9); докажите, что это соответствие взаимно-однозначное, б) См. указание к Ь8а. в) Каждому линейному уравнению ах + b = 0 с целыми коэф¬ фициентами можно поставить в соответствие числовой кортеж (k, \а\, |М. /), k = 1 при а > 0, (1 = 1 при b > 0, где k = 2 при а < 0 и и = 2 при b < 0, 1/ = 3 при b = 0. Для каждого линейного уравнения, такой кортеж — единствен¬ ный, и если два уравнения отличаются хотя бы одним коэффициен¬ том, то соответствующие им кортежи различны. Следовательно, множество линейных уравнений с целыми коэффициентами экви¬ валентно бесконечному подмножеству множества числовых корте¬ жей, т. е. счетно, г) Каждой положительной конечной десятичной дроби а, аи а2» Яз ••• ап соответствует числовой кортеж (а, аь а2, лэ, •••» ял), причем различнымдробям соответствуютразличные корте¬ жи. Множество таких кортежей счетно, следовательно, множество всех положительных конечных десятичных дробей счетно. Очевид¬ но, что множество отрицательных конечных десятичных дробей эк¬ вивалентно множеству положительных конечных десятичных дро¬ бей, следовательно, множество всех конечных десятичи&х дробей счетно, д) Каждому кубическому уравнению вида а0хв + a^ + + а2х + а3 = 0, где а0, aif а2, а3 — натуральные числа, соответст¬ вует числовой кортеж (a0t al9 a2t а3). Докажите, что соответствие взаимно-однозначно, е) Указание. Воспользуйтесь утверждением сМно^кество всех дробей счетно», доказанным в пособии на с. 176. ж) Указание. Перенумеруйте все целые числа (например, так, как показано в пособии на с. i75). Тогда каждому уравнению будет со¬ ответствовать числовой кортеж, состоящий из номеров его коэффи¬ циентов, причем различным уравнениям будут соответствовать раз¬ личные числовые кортежи* з) Рассмотрим множество'точек плоскости с рациональными координатами, абсциссы которых совпадают. Очевидно, что это множество эквивалентно множеству рациональ¬ ных чисел, т. е. счетно. Для каждой фиксированной ординаты мно¬ жество точек с всевозможными рациональными абсциссами также счетно. Таким образом, множество всех точек плоскости с рацио¬ нальными координатами является объединением счетного множе¬ ства счетных множеств, откуда следует, что оно счетно. Эту же за¬ дачу можно решить методом кортежей, не прибегая к теореме о счетностичмножества рациональных чисел, и) Указание. Восполь¬ зуйтесь методом кортежей, к) Внутри каждой петли восьмерки возьмем по точке с рациональными координатами. Из геометриче- 94
;„ских соображений очевидно, что любая другая восьмерка, для ко¬ торой эти две точки также будут находиться внутри ее петель, пере¬ сечет первую. Такимобразом, каждойвосьмерке можно поставить в соответствие две точки с рациональньщи координатами так, чтс * различным (непересекающимся) восьмеркам будут соответствовать разные пары точек. Множество таких пар точек счетно, откуда множество непересекающихся восьмерок тоже счётно. 19. Множество точек квадрата несчетно, так как содержит не¬ счетное подмножество, например множество точек диагонали. 20. Проведемдиаметр окружности, и каждойточкеодной из по¬ лученных полуокружностей'поставйм в соответствие ее проекцик (ортогональную) на диаметр. Это соответствие взаимно-однознач¬ но, т. е. множество точек полуокружности несчетно. Мощность мно¬ жества точек окружности не меньше мощности точек полуокруж¬ ности, следовательно, множество точек окружности также -несчетно. 21. Примерами бесконечных последовательностей, составленных т цифр 0,1,2, могут служить следующие: 01022102..., 22111002201 .:.. Предположим, что множество таких последовательностей счетно, т. е. все они перенумерованы: а1 — ^11^i2^13 ■-- а1 п 0С2 = ^21^22^23 •-• а2п (через атп обозначена п-я цифра в последовательности amf атп либо О, либо 1,'либо 2). Построим новую последовательность р = bJbJ>^ .:. Ъп по такому, например, правилу: Ъп = 0, если апп равно 1 или 2, Ъп — 1, если ацп = 0. Такая последовательность р отличает¬ ся от каждой из последовательностей ап хотя бы я-м членом. Ока¬ залось, что перенумерованы не все последовательности. Получен¬ ное противоречие показывает, что мйожество последовательностей указанного вида несчетно. 22. Не может (см. текст пособия, с. -180). 23. Пусть некоторое положительное число х выражается бесконечной десятичной дробью e0, 0i02 .*. ап ••• ат —; л-е десятичное приближение числа х с избытком равно а0, сца2 • •• ап + Ю“л и» следовательно, предст^в- йяет собой конечную десятичную дробь с нё более чем п значащими цифрами после запятой (см. курс IX класса). Если ап ф 9, то ^e десятичное приближение числа х имеет вид а0, а%а2 ... (ап + 1), т. е. содержит ровно п цифр после запятой. Допустим, что все прибли¬ жения числа х с избытком, начиная с n-го, совпадают и в то же время существует т > n, такое, что атФ 9. Но тогда тте приближение имеет больше цифр после запятой, чём n-e, т. е. этй приближения ве.могут быть равными. Полученное противоречие доказывает, что если все десятичные приближения числа х с избытком, начиная с n-ro, совпадают, то 95
все цифры в десятичной записи этого числа, начиЛая с a„+t, девятки. 7 24. Да, существует. Это число 0,777 ... = 0, (7) = —. В самом деле, всякое другое положительное число, меньшее 1, отличается от данного хотя бы одной цифрой после запятой; поскольку цифры, отличающиеся от 7, не могут быть ни 8, ни 9, то они будут меньше 7. Следовательно, всякое положительное число, меньшее 0,9, не содержащее в записи 8 и 9 и не равное 0,777..., будет меньше, чем 0,777..., т. е. 0,777... — наибольшее из чисел, меньших 0,9 и запи¬ сывающихся без цифр 8 и 9. 25. Не существует. Д о к а з а т е л ь- с т в о. Допустим, что такое число a еуществует. Поскольку 0,999... = 1, то в записи a должна быть цифра 6," т. е. a — '=0,999 ... 96999... . Но увеличив т на 1, т. е. передвинув 6 на (т + 2)-e место и поставив на (m + l)-e место 9, получим число, большее a. Следовательно, наибольшего числа a не существует. 26.' Это число, представленное периодическойдесятичной дробью 0,888... = 0,(8), является рациональным, равным —. 27. Множество точек отрезка [0, 1], оставшихся после удаления следующих отрезков: 1) [0,5; 0,*6]; 2) [0,05;0,06]; 3) [0,15;0,16]; 4) [0,25; 0,26]; 5) [0,35; 0,36]; 6) [0;45; 0,46]; 7) [0,65; 0,66]; 8) [0,75; 0,76]; 9) [0,85; 0,86]; 10) [0,95; 0,96] (отрезок [0,55; 0,56] входит в отрезок [0,5; 0,6]); lI)-91) [0, a05; 0,a06]; [0,al5; 0,ol6]; [0,a25; 0,a26]; [0,a35; 0,a36]; [0,a45; 0,a46]; [0,o65; 0,a66]; [0,a75; 0,a76]; [0,a85;_0,a86]; [0,a95; 0,a96],. где а принимает значения йз множества {0, 1, 2, 3, 4, 6, 7, 8, 9}. 28. Каждому неотрицательному десятично-рациональному чис¬ лу a, ata2a3 ... ап (ап Ф 0) cootBeTCTByer числовой' кортеж (о, Oi, аг, aa а„); для разных неотрицательных десятично¬ рациональных чисел такие числовые кортежи различны. Следова¬ тельно, множество неотрицательных десятично-рациональных чи¬ сел счетно. Очевидно, что множество отрицательных десятично¬ рациональных чисел также. счетно. Таким образом, множество всех десятично-рациональных чисел счетно, поскольку объедине¬ ние конечного или счетного множества счетных множеств счетно. 30. Задачу рекомендуется предложить учащимся решить са¬ мостоятельно и поручить кому-нибудь подготовить ее решение в ввде сообщения. Можно применить метод установления взаимно¬ однозначного соответствия, использованный в, пособии для отрез¬ ка и квадрата, 31. Среди действительных ч:;сел, меньших 1, не являются образами внутренних точек квадрата с вершинами A(0; 0), В (1;Ф), C (1; 1), D (0; 1), такие, например, числа, как 0,1479192939..., 0,78090909..., 0,159999..., т. е. числа, которые получились бы в результате «перетасовки» десятичных знаков деся- ee
тично-рациональных чисел, записанных с помощью бесконечного множества девяток. 32. Целесообразно предложить учащимся под¬ готовить решение этой задачи в форме реферата, воспользовавшись дополнительной литературой. Примерный план реферата. 1) Переформулировать задачу следующим образом: «Доказать, что множество точек счетномерного пространства имеет мощность континуума». 2) Доказать, что мощность континуума имеет множество точек: а) плоскости (двумерного пространства); б) трехмерного простран¬ ства; в) n-мерного пространства; г) счетномерного пространства. ДОПОЛНИТЕЛЬНАЯ ЛИТЕРАТУРА Детская энциклопедия, 3-еизд. т. 2. В и л е н к и н Н. Я. Рассказы о множествах. М., 1972. К у р а н т P., Р о б б и н с Г. Что такое математика. М., 1967. Стюарт Я. Концепции современной математики. Минск, 1980.
ПРИЛОЖЕНИЯ ТЕОРИИ ГРАФОВ При изучении этой темы целесообразно вы¬ делить три подтемы: «Знакомство с графами», «Сетевые графики» и «Транспортные сети». Учебный материал попервым двум подтемам содержится в I, IV и VII главах книги Березиной Л. Ю. «Графы и их применение» (пособие для учителей. М., Просвещение, 1979*). Учебный материал по третьей подтеме («Транспортные сети») по¬ мещен в данном пособии на с. 129. На изучение темы программой отведено 12 ч. На знакомство с графами, т.е. на рассмотрение задач, приводя¬ щих к понятию «граф», и на изучение основных понятий, — 4 ч. На изучениеподтемы«Сетевыеграфики»—3 ч. Навводнуюбеседу по материалу § 1 и § 5 главы VII и на правила построения сетевых графиков (§ 2 главы VII) — 2 ч; на рассмотрение критического пути (§ 3 главы VII) — 1 ч. На изучение.подтемы «Транспортные сети» — 3 ч. Из них 2 ч — на решение двух экономических задач, приводящих к транспорт¬ ным сетям; 1 ч — на уточнение основных понятий, представление о которых учащиеся получили на двух предыдущих занятиях в про¬ цессе решения задач. На обобщение и повторение пройденного, на знакомство с исто¬ рией теории графов — 1 ч. На проведение зачета — 1 ч. Тема носит ярко выраженную прикладную направленность. На простых примерах учащимся показывается, как можно применять язык теории графов к решению различных практических задач. Теория графов — молодая область дискретной математики. Первой монографии по теории графов немногим более сорока лет. Но методы теории графов завоевали признание не только математи¬ ков, но и инженеров, экономистов, психологов, лингвистов, биоло¬ гов, химиков. Использование языка и методов теории графов часто ускоряетрешениепрактическихзадач, упрощает расчеты, повышает производительность научной, инженерной и конструкторской мыс¬ * В дальнейшем это пособие в тексте обозначается номером 3, под которым оно указано в списке рекомендуемой литературы на с. 110, 98
ли. Именно запросы практики в значительной степени способству¬ ют интенсивному развитию теории графов. Понятие «граф» очень емко и тесно связано с многими основными пбнятиями, на которых строится здание математики, в том числе и школьной. О степени общности этого понятия свидетельствует уже то, что оно включает понятие «бинарное отношение», которое, в свою очередь, охватывает столь общие понятия, как отношения родства, тождества, конгруэнтности, равновеликости, подобия, параллельности, перпендикулярности, делимости и др. С отноше¬ ниями, а следовательно исграфами, связано такое важное понятие математики, как «функция» (или преобразование). Естественно, что язык графов начал проникать в школьные учебники. Так, в учебнике математики IV и в учебнике математики V классов графы используются для наглядного изображения по¬ рядка вычисления значения выражения, заданного схемой, а также для формирования у учащихся представления о действии, обратном данному. В учебнике «Алгебра» для VI класса рисунки графов ис¬ пользуются также в целях наглядности при объяснении учащимся, соответствиекакого вида называется функцией. В том же VI классе с помощью рисунков графов у учащихся формируется представле¬ ние о таких свойствах бинарных отношений, как рефлексивность, симметричность, транзитивность. В VIII классе рисунки графов участвуют в формировании понятия «функция, обратная данной». Графы (нередко под другими названиями) проникли в новые учеб¬ ники химии, биологии, географии, где они использованы для на¬ глядного и экономного описания различных схем организаций, логических возможностей, классификаций. Применение графов помогает думать, объяснять, наглядно представлять, поэтому их использование в различных школьных учебниках имеет естествен¬ ную тенденцию к развитию. Лри изучении темы «Приложения теории графов» учащимся нужны в качестве опорных знания элементов теории множеств и владение понятием «отношение между парами элементов множе¬ ства», которые они приобрели на уроках математики. Приводим содержание темы «Приложение теории графов». Знакомство с графами: задачи, приводящие к графам; основные понятия теории графов (полный граф, степень вершины, путь в графе, цикл, связность графа, мост, дерево, лес, ориентированный граф). Сетевые графики: работы и события; правило построения сете¬ вых графиков; критический путь; поиск резервов времени; из исто¬ рии сетевого планирования и управления. Транспортные сети: задачи, приводящие к транспортным сетям; основные понятия; две теоремы; алгоритм нахождения максималь¬ ного значения потока; родственные задачи. Из истории теории графов*.' * Параграфы и пункты в соответствующем учебном материале имеют такие же названия. 4* 99
ЗНАКОМСТВО С ГРАФАМИ О с н о в н ы е п о н я т и я: граф, вершина графа, ребро графа, степень вершины, путь в графе, цикл, дерево, ориентированное ребро, ориентированный граф. В с п о м о г а т е л ь н ы е п о н я т и я: лес, длина пути, путь в ориентированном графе, степень входа вершины, степень выхода вершины, источник, сток. При их введении подбираются знакомые учащимся ситуации, которые интерпретируются спОмо- щью графов наиболее наглядно. При формировании стержневого понятия «граф» и некоторых других основных понятий на первых занятиях с учащимися рас¬ сматривается яесколько задач, разных по условию и структуре. Их решение подводит учащихся к представлению о графе и одно¬ временно показывает возможные ситуации, которые целесообразно моделировать графами, а также приемы решения разнородных задач с использованием рисунков графов. С зтой целью приведены, на¬ пример, в § 1 первой главы задачи с решениями под номерами 1.1, 1.2, 1.3. С той же целью можно использовать также следующую задачу, решение которой с использованием графов вызывает обыч¬ но у учащихся значительный интерес. Рассматриваемый в процессе решения этой задачи граф дает представление о двудольном графе, о графе с цветными ребрами. 3 а д а ч а. Четыре одноклассника — Володя, Таня, Оля и Се¬ режа — выбраны классным собраниемдля работы вшефском, культ¬ массовом, спортивном и учебном секторах школы. Опреде¬ лить, кого из ребят в какой из названных секторов выбрали, если известно, что: 1) Володя и член культсектора живут в одном доме, и они вме¬ сте с Олей втроем ездят в школу на автобусе; 2) на классндм собрании было решено в спортсектор избрать мальчика; 3) член учебного сектора и Сережа вместе ходят на теннисный корт; 4) член спортсектора и Володя — большие друзья; 5) Оля сидит за одной партой с членом учебного сектора. Р е ш е н и е. В задаче рассматриваются два множества (множе¬ ство ребят и множество секторов), содержащие каждое по четыре элемента. Требуется установить взаимно-однозначноё соответствие элементов одного множества с элементами другого. Элементы этих множеств изображены на рисунке I кружками. Всего кружков 8, и они разделены на две группы (множества) по четыре кружка с помощью двух замкнутых линий. Если элемент 1-го множества не соответствует элементу 2-го множества, то соот¬ ветствующие кружки на рисунке соединяем штриховыми отрезка¬ ми, если же элементы друг другу соответствуют, то их кружки соеди¬ няем сплошными отрезками. Так, согласно условию 1, Володя и Оля не избраны в культсектор. Значит, кружки, изображающие 100
Шефский Культмас- Cnopmu6- УчеЬный f сектор содыйсектор ныйштор сетор ' Рис. I Рис. 2 Володю и Олю, и кружок, изображающий культсектор, соединяем штриховыми отрезками. Если и все остальные условия изобразить схематично, то получится рисунок 1. По нему видно, что кружок *0» может соответствовать только кружку «ш. с.», а кружок «с. с.»— кружку «С» и т. д. Проводим соответствующие сплошные линии (рис. 2). С рисунка 2 остается лишь считать готовый ответ. Вот еще один пример аналогичной задачи (такие задачи принято называть «логическими», хотя^ точнее их следовало бы называть задачами на направленный перебор возможностей). Купленные в подарок игрушки (пистолет, сумочку, куклу и машинку) уловили в четыре коробки, по одной игрушке в каждую. Требуется узнать,, что положено в каждую коробку, если известно следующее: машинка и пистолет не в красной коробке; коробка с сумочкой находится между синей коробкой и коробкой с куклой; в зеленой коробке не сумочка и не машинка; желтая и зеленая ко¬ робки находятся около коробки с пистолетом. О т в е т. Пистолет — в синей, сумочка — в красной, кукла — в✓ зеленой, машинка — в желтой коробках. (Заметим, что аналогичные задачи приведены вместе с решениями в журнале «Математика в школе» № 2 за 1972 год в статье JI. Ю. Березиной «Графы помогают решатьлогические задачи»; в книге «Внеклассная работа по математике» для учащих¬ ся IV—V классов (под. ред. С. И. Шварцбурда. М., Просвещение, 1974). Такого типа задачи систематически публикуются в журна¬ ле «Наука и жизнь» в разделе «Психологический практикум».) После решения пропедевтических задач внимание учащихся обращается на то обстоятельство, что справиться им с ними помога¬ ли рисунки-схемы, составленные из множества кружков и множе¬ ства отрезков, соединяющих некоторые из этих кружков попарно. Рассмотрение таких схем и приводит к понятию графа. В результате у. учащихся формируется представление о графе как о совокупности двух множеств — непустого множества круж¬ ков (или точек) и связанного с ним множества отрезков. Связь эта выражается в том, что оба конца любого из этих отрезков должны Яринадлежать кружкам из заданного им множества. Кружки теперь переименовываются в вершины графа, соединяю- |дие их отрезки — в ребра графа, а рисунок — в рисунок графа. 101
При переводе условия задачи на язык теории графов элементам заданного в ней множества ставятся в соответствие вершины графа, а отношению между парой элементов — ребро, соединяющее соот¬ ветствующие вершины. Если рассматриваются отношения двух видов, то ребра графа удобно окрашивать в два цвета. В приведен¬ ной выше задаче по существу рассматривались ребра двух видов (цветов) и вершины двух видов (цветов). Далее вводятся новые понятия, перечисленные выше. При этом всякий раз акцентируется внимание учащихся на том, зачем вво¬ дится данное понятие и как оно работает. Так, степень вершины служит для характеристики именно вершины, а не графа в целом. Введение понятия «путь в графе» позволяет подойти к классифика¬ ции графов с точки зрения наличия или отсутствия в них циклов и т. д. С этой целью используется следующий материал пособия [3]: § 2 и § 3 из первой главы, § 1 из четвертой главы. Теоремы 1.1—1.4, являющиеся обобщением решенных в тексте задач, и теоремы 1.5 и 1.6 на факультативных занятиях могут не рассматриваться. Перечисленные теоремы хорошо усваиваются даже и наиболее слабыми учащимися, но .при повторном чтении ими материала, когда они свободно владеют всем остальным. Для усвоения, закрепления итекущегоконтролязнанийисполь- зуются следующие упражнения из пособия [3]: 1.1, 1.3, 1.5, 1.6, 1.9—1.12, 1.14, 1.18, 1.20, 1.22, 1.33, 1.34, 1.37, 1.40—1.42, 1.46, 1.58, 1.59, 4.1—4.3. Это набор-минимум упражнений. Для более прочного усвое¬ ния можно предложить дополнительно любые другие упражнения из пособия [3], взятые из соответствующих параграфов. СЕТЕВЫЕ ГРАФИКИ О с н о в н ы е п о н я т и я: работа, событие, сетевой график, продолжительность пути, критический путь. В с п о м о г а т е л ь н ы е п о н я т и я: ранний срок наступ¬ ления события, поздний срок наступления события. Все вводимые здесь понятия имеют естественную природу. Для того чтобы учащиеся почувствовали это, знакомство с ними целе¬ сообразно начинать после рассмотрения отдельных работ и их вза¬ имосвязей, составляющих какой-то достаточно известный, близкий им комплекс работ, например комплекс работ по строительству школы, озеленению школьного участка, проведениюэкзамена,про- ведению КВН и т. д. При рассмотрении примеров следует обратить внимание уча* щихся на то, что отдельные работы из данного комплекса могут вы¬ полняться только в определенной последовательности, другие не¬ зависимо друг от друга. Если отдельным работам ставить в соответствие ориентирован¬ ные ребра некоторого графа, а событиям (как итогам выполнения отдельных работ) — вершины, причем время, необходимое для вы¬ 102
полнения этих отдельных работ, проставлять над соответствующими р'ебрами, то весь комплекс работ может быть наглядно изображен в виде рисунка-схемы, который называется сетевым графиком. Сетевой график помогает учащимся заранее узнать время, не¬ обходимое для выполнения данного комплекса работ, выделить наиболее ответственные участки, найти резервы времени для отдель¬ ных работ и событий. Учащихся следует познакомить с основными правилами построе¬ ния сетевых графиков, дать, им самостоятельно построить и просчи¬ тать сетевые графики нескольких знакомых комплексов работ или фрагментов этих комплексов. Здесь можно разделить расчетную и графическую работу среди учащихся так, чтобы они чувствовали, что умение строить и рассчитывать сетевые графики действенно помогает рациональному планированию разнообразной деятельно¬ сти любого коллектива, а также отдельного человека. Соответствующийучебный материал находится вглаве VII по¬ собия [3]. При недостатке времени § 4 (о резервах времени) опускает* ся яли в ознакомительном порядке рассказывается о возможности определения по сетевому графику резервов времени для работ, не лежащих на критическом пути (по материалу § 4). Для усвоения, закрепления и текущего контроля знаний пред¬ лагаются следующие упражненид из пособия [3]: 7.1—7.5, 7.9—7.18, 7.21, 7.22, 7.26—7.28, 7.33. ТРАНСПОРТНЫЕ СЕТИ О с н о в н ы е п о н я т и я: сеть, пропускная способность ребра, транспортная сеть, поток по ребру, поток в транспортной сети, пропускная способность разреза. Предлагаемая методика решения двух практических задач (о расяределении потока машин по улицам города и на составление плана перевозок товара с фабрик в магазины-заказчики, требую¬ щего минимальных затрат на транспортировку) поможет учащимся получить представление о таких важных понятиях, как «разрез в сети» и «пропускная способность разреза». При этом учащиеся убе¬ дятся в справедливости основной теоремы теории сетей: «В любой транспортной сети максимальное значениепотока равно минималь¬ ной пропускной способности любого из возможных разрезов». После решения первой задачи полезно на этом же занятии выделить и прокомментировать все понятия, которые при решении возникали и использовались, а также основной вывод — теорему Форда и Фал- керсона. Эта теорема лежит в основе решения класса задач, называе¬ мых сетевыми задачами о потоках; ее доказательство на занятиях факультатива не рассматривается. Соответствующий учебный материал содержится в статье «Тран¬ спортные сети» данного пособия* 103
ИЗ ИСТОРИИ ТЕОРИИ ГРАФОВ В 1736 году великий математик Леонард Эйлер нашел решение головоломки, носящей название «Проблема кёнигсбергских мостов»*. Река Прегель, протекающая через Калининград(прежде город назывался Кёнигсбергом) омывает два острова (рис. 3). Берега реки с островами были во времена Эйлера связаны мостами так, как это показано на рисунке. В головоломке требовалось найти маршрут, проходящий по всем четырем участкам суши по одному разу. При этом через каждый из мостов можно проходить только по одному разу, а конец и начало пути должны совпадать. Л. Эйлер доказал, что маршрута, который бы отвечал условиям головоломки, не существует, и разработал теорию решения такого рода головоломок. Владея материалом вводной части курса «Зна¬ комство с графами», нетрудно воспроизвести идею рассуждения Л. Эйлера. Для этого нужно предварительно заменить рисунок 3 схемой, приведенной на рисунке 4, где острова и берега изобража¬ ются точками. Схема, приведенная на рисунке 4, не является, строго говоря, графом: на ней имеются кратные ребра**. Тем не менее 1736 год, когда эта головоломка была решена, принято считать годом рожде¬ ния теории графов. Спустя сто с лишним лет, в 1847 году немецкий ученый Г. Кирх¬ гоф разработал эффективную методику определения значения силы тока b электрической цепи, используя методы и понятия, йолучив- шие позднее права гражданства в теории графов. Еще 10 лет спустя , английский математик А. Кели (мать его была русской, он владел русским языком и следил за русской математической литературой; он оказался среди тех немногих математиков, которые с самого начала поняли и поддержали идеи Н. И. Лобачевского) разработал теорию деревьев для подсчета числа изомеров насыщенных углево¬ дородов СяН2я+а с данным числом п атомов углерода'. Заметьте, за головоломками последовали серьезные праЛиче- ские исследования. Так нередко случалось в исяории математики. * Леонард Эйлер (1707—1783).— уроженец Швейцарви, большую часть своих научных работ написал в Россни. своей второЙ родине. ** Такие схемы называют мультиграфами.
Рис. 5 Вспомним задачу об удвоении куба в геометрии, головоломки, свя¬ занные с азартными карточными играми, которые стимулировали возникновение теории вероятностей. Однако, по обилию головоло¬ мок, с которыми она оказалась связанной, теория графов среди ма¬ тематических дисциплин явля'ется, несомненно, рекордсменом. Ha- :вовемлишь некоторые: «Игра в 15», «Кругосветкам У. Гамильтона, *задача о раскрашивании географической карты в четыре цвета (ре¬ шена только недавно), многочисленные задачи на шахматной до¬ ске [7]. Помогают графы и школьникам при решении олимпиадных Эадач,которые, какизвестно,требуютмаксимальной математической изобретательности при минимальных знаниях учащихся^ £5]. Ре¬ шение головоломок приводило исследователей к поискам обобщений и разнообразных следствий, что позволяло им строить разделы бу¬ дущей теории. Когда же стимулом для развития теории служили проблемы прикладного характера, то нередко решение такого рода пррблем давало вклад сразу в несколько разделов будущей теории. Последние десятилетия — свидетели бурного развития теории графов, на что заметное влияние оказывают запросы все новых ЗДластей приложений. Теория графов уже применяется в таких областях, как физика, химия, генетика, П£ихология, социология, Экономика, математическая лингвистика, теория планирования и управления, электроника, электротехника, проектирование вычи¬ слительных машин. Теория графов тесно связанатакже со многи¬ ми разделами математики, среди которых теория групп, топология,, ^комбинаторика, теория вероятностей. Теория графов привлекательна еще и тем, что в ней наряду с ре¬ шенными задачами и проблемами существуют задачи нерешенные. Некоторые из них имеют элементарную занимательную форму, вы¬ глядят как головоломка или олимпиадная задача. Например, из¬ вестно, что на бумаге в клетку можно нарисовать 5 фигур из 4 кле¬ ток так, чтобы в каждую клетку можно было пройти из соседней через сторону (рис. 5). Известно далее, что таких фигур из пяти кле- ток12, из 6—35, ..., из 10—4271. А сколько таких фигур из 11 клеток и вообще из п клеток — неизвестно [10]. Как знать, может случиться, что решение этой или другой подобной задачи послужит стимулом для развития одного из разделов теории графов. Но это может случиться в данном случае лишь тогда, когда используемый автором исследования метод рассуждений окажется применимым к более широкому классу задач. . Головоломки, аналогичные только что рассмотренной, возника¬ ют довольво часто, и это обстоятельство свидетельствует лишний раз о том, что учащимся, заинтересовавшимся исследовательской 105
работой в области теории графов или ее приложений, предстоит много увлекательных и перспективных дел. Для желающих про¬ должить изучение теории графов можно назвать несколько книг, содержащих материал разной трудности, но всегда интересный, если в нем достаточно разобраться (см. список дополнительной ли¬ тературы). ДОКЛАДЫ И РЕФЕРАТЫ ]. Учащимся, заинтересовавшимся теорией графов, можно пред¬ ложить подготовить реферат или доклад. Приведем некоторые воз¬ можные темы. Плоские графы. Графы с цветными ребрами. Ориентированные графы. Основным учебным материалом для подготовки этих докладов может служить материал глав с соответствующим названием в по¬ собии [3]. Дополнительная литература названа в пособии [3] на стра¬ нице 134. 2. Учащимся, заинтересовавшимся сетевым планированием и управлением, можно предложить подготовить реферат или доклад на тему «Расчет сетевых графиков», в который войдут и правила определения резервов времени. В качестве основногоможет быть использован учебный материал главы VII пособия [3], а дополни¬ тельная литература указана там же на странице 133. 3. Учащимся, заинтересовавшимся сетевыми задачами и желаю¬ щим глубже познакомиться с ними и методами их решения, можно предлбкить подготовить реферат на тему «Алгоритм нахождения максимального значения потока». В качестве основного пособия при подготовке реферата можетбыть использован учебный материал, приведенный на страницах 138 и 142, где показана работа алго¬ ритма на примере простой сети. В качестве дополнительной лите¬ ратуры можно порекомендовать: [1], гл. 6, § 2—7, гл. 7, [9]. 4. Графы в играх и головоломках (список литературыприведен на странице 134 в пособии [3]). 5. Графы и их роль в школьных учебниках. При работе над этой темой учащимся нужно самостоятельно проанализировать школьные учебники по разным предметам и найти примеры исполь¬ зования графов в качестве иллюстративного материала. Отметим здесь некоторые, на наш взгляд, удачные примеры ис¬ пользования рисунков графов как средства наглядности в школь¬ ных учебниках биологии, географии, химии. Уже на форзацах учебника зоологии для VI—VII классов изо¬ бражена классификация основных типов животных в виде графа с цветными вершинами, представляющего собой лес; вершины этого графа окрашены в шесть цветов, что соответствует,числу основных групп в зоологии: типы, классы, отряды, семейства, роды, виды. Здесь рисунок графа дает наглядное представление о сущности клас¬ 106
сификации. Рисунки графов, называемых деревьями, используются и в параграфе об основных этапах эволюции животных. Здесь схе¬ матично изображено эволюционное дерево типов животных и эво¬ люционное дерево классов хордовых. Среди учебников и учебных пособий географии для средней школы наиболее широко используются рисунки графов в учебном пособии для VIII класса «Экономическая география СССР». Уже на форзаце изображена очень емкая по содержанию и в то же время наглядная схема народного хозяйства Советского Союза, представ¬ ляющая не что иное, как рисунок графа. Можно представить, на¬ сколько усложнилась бы работа учителя и было бы затруднено вос¬ приятие учащимися сути этойсистемы, если бы эту сложную систему прщнлось описывать словами. Рисунки графов удачно использо¬ ваны, например, и для наглядного изображения схемы использова¬ ния угля в народном хозяйстве, структуры машиностроительного завода и его производственных связей, структуры сельского хозяй¬ ства Советского Союза, схемы использования горючих сланцев, схемы внутрипроизводственных связей республик Закавказья. В учебниках и учебныхпособияхпоистории для средней школы рисунки графов использованы мало. Так, в учебнике по истории для VII класса изображено ориентированное дерево, соответствую¬ щее системе управления в Российском государстве XVI—XVII ве¬ ков, а также ориентированный граф, соответствующий системе го¬ сударственного управления при Петре I. Если просмотреть учебники старших классов, то можно увидеть, что в учебнике общей биологии для IX—X классов графыдшироко используются, например, в параграфе об образовании новцк видов для изображения схемы дивергенции и эволюции систематических групп, для изображения структуры вида. В главе «Учение .о клет¬ ке» графы используются для представления структурных формул белковых молекул. При изучении экологических систем рисунок ориентированного графа используется для отражения основных пйщевых связей в пресноводном биоценозе. В учебнике по химии для старших классов постоянно исполь¬ зуются наглядные изображения структурных формул предельных углеводородов в виде рисунка графа, вершины которого соответст¬ вуют атомам, а ребра — валентным связям между атомами, н уча¬ щиеся знают, что порядок взаимосвязи атомов при одномл том же качественном и количественном составе молекул влияет на свойст¬ ва вещества. Учащимся известно также, что знание о существова¬ нии именно двух рйзличных структурных формул бутана помогли А. М. Бутлерову предпринять попытки синтезировать бутан дру¬ гого строения. Было бы хорошо отыскать и другие, еще не исполь¬ зованные авторами возможности наглядного изображения с помо: щью графов различных схем организаций, классификаций и т. п. Сообщение о содержании проделанной работы на занятии пока¬ зало бы школьникам новые для них стороны применения математи¬ ки в изучении других предметов. 107
В конце темы «Приложения теории графов» целесообразно про¬ вести зачет, на котором учащиеся должны показать, как они овла¬ дели основными понятиями графов, а также такими понятиями, как «работа», «событие», «сетевой график», «критический путь», «крити¬ ческие работы». Учащиеся должны также продемонстрировать умение упорядочить в «технологической последовательности» от¬ дельные работы известных им комплексов,строитьсетевые графики несложных комплексов работ, рассчитывать их, т. е. определять критический путь, его продолжительность, наиболее напряженные работы. Учащиеся должны уметь по ребрам несложной сети распре¬ делить поток заданного значения, уметь построить разрез сети и подсчитать его пропускную способность, найти максимальное зна¬ чение потока. ПРИМЕРНОЕ СОДЕРЖАНИЕ ЗАЧЕТА 1. Нарисуйте полный граф с 6 вершинами. 2. Чему равна степень какой-нибудь вершины полного графа с 20 вершинами? 3. Сколько ребер у полного графа с 20 вершинами? 4. Изобразите граф с 6 вершинами, в котором есть двацикла, каждый из которых проходит через все вершины графа. 5. Изобразите три разных графа, с 6 вершинами каждый, у ко¬ торых нет ни одного цикла. 6. Приведите примеры связных и несвязных графов с 6 верши¬ нами. = 7. Нарисуйте граф, содержащий три моста. 8. Нарисуйте два связных графа с 5 вершинами, каждый так, чтобы один граф был деревом, другой деревом не являлся. В чем их отличие? 9. Придумайте жизненную ситуацию, описываемую ориентиро¬ ванным графом с 5 вершинами. 10. Назовите работы и события, которые можно выделить при проведении, например: а) субботника; б) шахматного турнира; в) встречи футбольных команд из двух разных школ. 11. Изобразите транспортную сеть с. 11 вершинами и 20 ребра¬ ми. Проведите три ее разреза и подсчитайте их пропускные спо- co6nocTiL На зачете можно использовать также следующие упражнения из пособия [3]: 1.2, 1.8, 1.15, 1.24, 1.35, 1.36, 1.38, 1.43, 1.45, 1.47, 4.2, 4.3, 7.6—7.8, 7.13, 7.19, 7.20, 7.23, 7.27. Ответы к упражнениям из пособия [3] (Приводятся только те ответы, которых нет в пособии.) 1.3. а) 3; б) 6; в) 10. 1.6. 5. 1.9. Не найдется. 1.10. Не найдет¬ ся. 1.12. Две. 1.14. Существует. Один. SCDAEHFBR. 4.1. б) Е — источник; В — сток; в) 1; г) 2; д) такой нет. 4.2. От А до В четыре пути. S (АВ) = U S (CA) = оз, S (AC) = 1. 108
7.1. а) Подвоз материалов; геодезическая съемка местности; расчистка площадки для строительства; разработка проекта здания цжолы; монтаж фундамента и др. ' : 7.2. j) Проведение первой репетиции; распределение ролей; Шроба на роли; работа над ролями и др. 7.3.- К событиям: 1, 3, 5, ^,8,11;кработам:2,4,6,9,10.7.7. а) <0;-l>, <0;4>,<l;2>, -<4;5>; б) <0;1>, <0;4>, <l;2>, <4;5>, <5;6>; а) <0; 1>, <l;2>, <2;3>. 7.13. а) b, d; б) a,b,d. 7.14. b. 7.15. b. 7.16. а. Ответы к упражнениям из статьи «Транспортные сети» 1. Одно из распределений может быть таким: <Poi = 7; Фоз ^3 3; Фи — 3; Фхз = 4; ф32 = 3; ф2Э = 0; Фа« =“ 4; ф2„ = 6; а другое: ф01 = 6; ф03 = 4; ф1а = 3; Ф13 = 3» Ф32 — 1» Фаз ~ 0; фЭд — 7; ф2л = 3. 2. Другой вариант может быть таким: " Фо1 = ®> Фоз = 5; ф13 = 5; ф1а = 3; ф23 = 0; ф32 = 3; Фзл = 7; Ф2Я = 6. Н .H 3. а) Например, так: ф0£ = 8; ф0э = 4; ф3х =? I; Фм =f, 1; ^Ряя = 2; ф12 — 9; ф2л = 10. б) Сеть пропускает поток, значение которого больше 12, на- рример значением 13. ®) /max (Т) .— 14; ф01 = 10; Фоз = 4; фй = 2; фм = 0; фЭп= 2; Vn = 12; фал = 12. 4- fmvkT) = 15- °Дин из вариантов ф01 = 5; ф03 = 4; Ф08 = 6; 4>ia-= 4; Фхз = 0; ф14 = 1; ф68 = 2; ф* = 4; фБ4 = 3;, ф6ч = 3; Фм = 1; Фаи = 5;-ф*ч„ = 7. 5. 1) 12; 13; 10. 2) Например, Р = {В0, Вх), а (P, Р) = 12; Р - {fio, Яв>. а (P, Р) - 12; Р = {В0, Blt В3, Вь, В4>, а (P, Р) = 14. 3) Р « {Bo}. 4) Нельзя. 5) fmn (T) = 10. 6) Пропускные способности всех •Возможных разрезов не меньше /тах (T). 109
ДОПОЛНИТЕЛЬНАЯ ^ЛИТЕРАТУРА 1. Б а с а к е р Р., С а а т и Т. Конечные графы и сети. М., 1974. 2. Б е л о в В. B., В о р о б ь е в Е. М., Ш а т а л о в В. Е. Теория гра¬ фов. М., 1976. 3. Б е р е з и н а Л. Ю. Графы и их применение. Пособие для учителей. М., 1979. 4. Б е р ж К. Теория графов и ее применение. М., 1962. 5. Венгерские математические олимпиады /Сост. Кюршак Й., НейкомиД., Хайош Д., Шурани Я. М., 1976 (раздел III). 6. Г а с с С. Путешествие в страну линейного программирования. М., 1973, 7. Г и к Е. Я. Магематика на шахматнойг доске. М.» 1976. 8. Г р о с с м а н И., М а г н у с В. Группы и их графы. М., 1971. 9. Оре О. Графы и их применение. М., 1965. 10. Ф о р д Л. Р., Ф а л к е р с о н Д. Р. Потоки в сетях. М., 1966. 11. X а р а р и Ф. Комбинаторные задачи перечисления графов, статья в сборнике «Прикладная комбинаторная математика». М., 1968. 12. X а р а р и Ф, Теория графов. М., 1973.
АЛГОРИТМЫ И ПРОГРАММИРОВАНИЕ Электронные вычислительные машины (ЭВМ) все решительнее проникают в самые разнообразные сферы челове¬ ческой деятельности. Все в большей мере возникает необходимость подготовки специалистов, способных управлять работой ЭВМ и ^использовать неисчерпаемые возможности вычислительной тех¬ ники. Знание основ машинной математики становится в наше вре¬ мя элементом общей культуры. Сведения по программированию в факультативном курседля VIII классадаютвозможность учащимся расширить свои представления оматематике, ееметодах и практиче¬ ских приложениях и углубить знания о программировании, полу- ченныеимивкурсематематики VIII класса. В данном случае занятия по программированию не должны ста¬ вить своей целью обучение технологии программирования. Кроме прикладной направленности, они должны сыграть существенную роль в общематематическом развитии учащихся. Программой факультативного курса «Избранные вопросы мате¬ матики» предусмотрено изучение в VIII классе темы «Алгоритмы ц программирование», для которой выделено 12 ч. В качестве учеб¬ ного материала по данной теме рекомендуется использовать содер¬ жание первых двух глав «Алгоритмы» и «Общие сведения об ЭВМ» учебного пособия по факультативному курсу для учащихся VIII— IX классов. «Программирование» (авт. Антипов И. Н. М., «Про* свещение», 1976 г.). Примерное распределение учебного времени. Общие сведения об алгоритмах — 2 ч. Некоторые формы описания алгоритмов — 2 ч. Блок-схемы — 2 ч. Назначение ЭВМ. Устройство и схема работы ЭВМ. Учебная дюдель ЭВМ — 3 ч. Примеры программ — 3 ч. В каждом конкретном случае может возникнуть необходимссть в сокращении имеющегося учебного материала. Укажем здесь воз¬ можности такого сокращения. 1. В§ 1главы1можно ограничиться разборомменьшегочисла примеров «бытовых» алгоритмов. 2. Иллюстрацию выполнения программы на учебной ЭВМ мож- lU
но значительно сократить, используя заготовленные заранее рисун¬ ки (§ 3, гл. II). 3. В § 4 главы II возможно рассмотреть меньшее число про¬ грамм, соответственно можно уменьшить и число упражнений. С другой стороны, возможен случай введения дополнительного ма¬ териала. Такой материал и рекомендации по его использованию даются ниже. Материал данной темы является частью общего комплекса све¬ дений по программированию, предлагаемых для факультативного изучения. Этот комплекс состоит из следующих частей: а) системы счисления и арифметические основы ЭВМ (12 ч, VII класс), б) ал¬ горитмы и программирование (12 ч, VIII класс), с) алгоритмиче¬ ские языки (20 ч, IX класс). Каждая из этих частей можетизучать- ся независимо друг от друга. Но наиболее эффективным следует считать последовательное их изучение, что может дать школьни¬ кам более полное и необходимое начальное представление о.про- граммировании. ОБЩИЕ СВЕДЕНИЯ ОБ АЛГОРИТМАХ С понятием алгоритма в неявном виде школьник постоянно со¬ прикасается. Наиболее четко алгоритмы выявляются в курсе ма¬ тематики; На данном этапе обучения не ставится цёли изучения теории алгоритмов. Школьники лишь получают общее представление о понятии алгоритма на интуитивно-содержательном уровне. Для приведения в систему знании учащихся по алгоритмам предлагается определить алгоритм как точное описание некоторого вычислитель¬ ного процесса или любой иной последовательности действий. Здесь имеется в виду такое описание последовательности выполняемых действий, следуя которому исполнитель (человек или автомат) по заданным начальным условиям получит необходимый результат. Алгоритм позволяет единым образом решать серию сходных задач. Следует обратить внимание школьников на тот факт, что многие повседневные ситуации наряду с математическими задачами могут рассматриваться с общих алгоритмических позиций. Понятие алгоритма появилось в IX веке. Под алгоритмом пони¬ мался процесс выполнения арифметических действий. Затем ал* горитм стал определяться как конечная совокупность точно сформу¬ лированных правил решения определенного класса задач. Строго* математические*определения понятия алгоритма стали появляться с возникновением теории алгоритмов (30-e годы нашего века). Пер¬ воначальной областью приложения этой теории была теоретическая математика. Дальнейшему развитию теории алгоритмов в значи¬ тельной степени способствовало появление ЭВМ- НЕКОТОРЫЕ ФОРМЫ ОПИСАНИЯ АЛГОРИТМОВ Рассмотрение средств описания алгоритмов целесообразно на¬ чать со средств родного языка, знакомых и понятных школьнику, а затем перейти к изучению других средств. Итак, школьники_зна¬ 112
комятся со словесным представлением алгоритмов. Перечисляются различные формы выражения алгоритмов. Отмечаются устная и письменная форма выражения алгоритмов, запись алгоритма в ви¬ де обычного текста, плана, инструкции и т. д. Математика дает свои средства описания алгоритмов, а именно *зык математических описаний, который наиболее четко проявляет¬ ся й легко воспринимается в том случае, когда для описания алго¬ ритмов используется формула. На данном материале желательно несколько упорядочить све¬ дения школьников о возможностях представления алгоритмов. Такая работа является подготовительным этапом к изучению блок- £хемного.представления алгоритмов и описаний алгоритмов сред¬ ствами, ориентированными на машинную реализацию алгоритмов. С целью более эффективного изучения новых средств описания алгоритмов в данном разделе рассматриваются такие составные части алгоритмических описаний, как а) линейные участки; б) раз¬ ветвления; в) циклы. В.описании алгоритма указывается последовательность дейст¬ вий, выполняемых в определенном порядке. Простейший случай — это когда действия выполняются в естественном порядке (последо¬ вательно друг за другом). В таком случае принято говорить о ли¬ лейных алгоритмах. Алгоритмы, в которых порядок выполнения Йействий зависит от некоторых условий, называют разветвляющи- кися. Алгоритмы, в которых некоторыедействия могут выполнять¬ ся многократно, называют циклическими. Целесообразно сначала познакомить школьников с линейными ^лгоритмами, потом с разветвляющимися и затем с циклическими, ^гработав приемы записи отдельных видов алгоритмов, можно пе¬ рейти к использованию описаний алгоритмов, содержащих развет- ^дения и циклы с достаточно сложной схемой их объединения. БЛОК-СХЕМЫ Блок-схемы являются одним лз средств описания алгоритмов. ;Это графический способ представления алгоритмов. Иногда гово¬ рят об описании алгоритмов на языке блок-схем. Блок-схемы на- гаядно выражают логическую суть решения задачи. Блок-схемы получили широкое распространение, и признание благодаря их применению в области машинной математики. Они Довольно независимы oT^этой области их применения. *: Существуют различные .способы начертания блок-схем. Отметим здесь некоторые особенности. 5; Вместо слов ВХОД и ВЫХОД часто используются слова НАЧА- $ЯО и КОНЕЦ. В случае машинного использования алгоритма Эместо слов ВЫХОД и КОНЕЦ употребляют слова СТОП, OCTA- ров. Задание исходных данных и запись результата (когда речь идет Ь машинном решении задач) могут быть выражены блоками ввода 113
исходных данных и соответственно вывода (или печати) результата. Встречаются изображения логических блоков в виде ромбов (вместо принятых нами овалов). В таком случае овалы могут быть использованы для других целей, например для изображения бло¬ ков начала и конца или входа и выхода. Когда блок-схемы громоздкие, блоки для удобства снабжают номерами. Несмотря на разнообразие способов изображения блок-схем в научной и учебной литературе, можно заметить, что их различие не является принципиальным. Правила построения схем носят от¬ носительный или условный характер. Усвоив один видсхем, можно без труда разобраться и воспользоваться другими видами схем, так как принцип их организации общий. В методическом плане представляет интерес перевод описания алгоритма с блок-схемы на словесное представление и обратно. В случае появления возможности более подробного рассмотрения раздела об описании алгоритмов можно рекомендовать дополни¬ тельно разобрать вопрос о детализации алгоритмов. Детализация алгоритмов* Знакомясь с самыми разнообраз¬ ными описаниями алгоритмов, можно заметить, что алгоритмы опи¬ сываются с различной степенью подробности. Представляя решение задачи в виде некоторой последователь¬ ности действий, мы тем самым разбиваем поставленную задачу на несколько подзадач. Другими словами, некоторое сложное задание или действие выражается через менее сложные действия. Алгоритм можно рассматривать как предписание, сводящее выполнение некоторого действия к выполнению других действий, которые считаются более простыми. Рассмотрим несколько при¬ меров. П р и м е р 1. Для сложения нескольких дробей с разными зна¬ менателями можно воспользоваться следующими действиями. 1. Найти наименьший общий знаменатель нескольких дробей. 2. Найти дополнительные множители для каждой дроби. 3. Вычислить сумму произведений числителей дробей на их соответствующие дополнительные множители. Эти основные действия для решения поставленной задачи могут считаться элементарными. Но для тех, кто не имеет соответствую¬ щей подготовки, необходима дальнейшая расшифровка. Можно подробнее описать нахождение наименьшего общего знаменателя и дополнительных множителей дробей. Таким образом указанные выше действия можно выразить через более простые и их считать элементарными. П р и м е р 2. Для решения задачи вычисления платы за элект¬ роэнергию можно предложить следующее описание алгоритма. 1. Вычислить плату за израсходованные кВт/ч. 2. Вычислить размер пени за просроченныедни. 3. Вычислить общую сумму платы. Этого описания достаточно для решения задачи тому, кто имел 114
опыт подсчета платы за электричество й знает, как выполнять ука¬ занные действия. При этом нетрудно заметить, что каждый пункт представляет собой некоторую самостоятельную задачу, для ре¬ шения которой, в свою очередь, нужен алгоритм, правда, более простой. Так, первый пункт можно разбить на два действия. Ы. Подсчитать количество израсходованных кВт/ч. 1.2. Умножить число кВт/ч на стоимость одного кВт/ч. Можно надеяться, что этого достаточнодля выполнения первого пункта алгоритма вычисления платы за электроэнергию. Если же окажется, что, например, описание действия 1.1. для некоторых исполнителей будет непонятно, то можно провести дальнейшую детализацию, например пояснить, как производится снятие пока¬ заний счетчика и подсчет израсходованных кВт/ч. Детализацию можно закончить, доведя описание алгоритма до элементарных действий, которые не требуют дальнейшего пояс¬ нения и являются выполнимыми. В некоторых случаях действия, приведенные первоначально в трех пунктах описания алгоритма, могут считаться элементарными действиями. В других случаях элементарными действиями являются запись показаний счетчика, умножение одного числа на другое. Все зависит от квалификации исполнителя. Рассмотренные выше примеры детализации алгоритмов дают нам некоторое представление о доведении описания алгоритма до так называемых элементарных действий. Мы соглашаемся с тем, что арифметические действия являются элементарными. Однако на более раннем этапе изучения математики приходится иметь дело с описанием алгоритмов выполнения арифметических действий, т. е. представление этих действий через более простые. Например, деление десятичных чисел сводится после некоторых преобразова¬ ний к более простому действию — к делению на целое число и т. д. Мы видим, что понятие элементарного действия носит условный характер и фактически определяется уровнем подготовки того, кто выполняет алгоритм. П р и м е р 3. Рассматривая какую-либо блок-схему алгоритма решения квадратного уравнения ах2 + bx + с = 0 с исследова¬ нием коэффициентов, нетрудно заметить, что схемой представлен план исследования и решения уравнения. Поставленная задача такого рода описанием алгоритма подразделяется на несколько более простых задач. Каждая из таких задачсоответствуетодному или нескольким определенным блокам схемы. Так, например, в схеме могут встретиться блоки следующего содержания: «Вычисление D = b2 — 4ас», «Решение уравнения bx + с — 0», «Вычисление значений корней по формулам» и т. п. Решение каждой.из этих задач на данном этапе является эле¬ ментарным. Однако схему можно далее детализировать, расшифро¬ вав некоторые блоки. Задача будет подразделена на более простые действия, которые также можносчитатьэлементарными.Очевидно, 115
доведение описания алгоритма достаточно сложной задачи до таких элементарных действий, как выполнение арифметических действий и сравнений, может привести к плохой обозримости алго¬ ритма и к нецелесообразности использования этого описания алго¬ ритма. По этим соображениям решение многих, задач представляется так называемым крупноблочным описанием алгоритмов. Например, составляется блок-схема, в которой многие из блоков содержат тот или иной алгоритм решения более простой и знакомой задачи, но этот алгоритм не расписан подробно. В случае необходимости круп- ный блок может быть детализирован, т. е. представлен более по¬ дробной схемой, и такую схему возможно изобразить отдельно вне общей крупноблочной схемы решения задачи. Так, в приведенном выше примере (решение уравнения)можнодать детальнуюрасшиф- ровку блока решения уравнения bx + с = 0 или какого-либо другого блока. П р и м е р 4. Вычисление по формуле Р = (А — В) - 0fi4 ■ (1 + 0,001 . Т). Формула является достаточно ясным описанием алгоритма. По этой формуле человек может вычислить искомое значение. В некоторых случаях требуется более подробно описать этот алго¬ ритм, расчленив формулу на более простые составные части, на¬ пример состоящие из одного арифметического действия. Такая детализация описания алгоритма необходима, например, при использовании учебной ЭВМ.. В этомслучаедетализация опи¬ сания алгоритма доводится до элементарных действий, которые вы¬ полняет машина. В описании алгоритма указывается последовательность дей¬ ствий, которые необходимо выполнить для решения некоторой задачи. Составляя описание алгоритма, мы можем быть уверены, что каждое действие может быть выполнено нами или тем, кому адресовано описание, в частности ЭВМ. Обычно предполагается, что мы понимаем смысл каждого действия, умеем выполнять и опи¬ сывать его. Эти действия принято называть элементарными дейст¬ виями по отношению к данному алгоритму. Понимание смысла и умение выполнять действие в нашем случае по своей значимости уступают фактору — умению записывать действие. Дело в том, что мы ориентируемся на машинную реализацию алгоритмов. Ма¬ шина выполняет действия в соответствии с описанием алгоритма, строго придерживаясь тех указаний, которые даны в описании алгоритма. Работа выполняется чисто механически. Человек также может не все понимать и механически выпол¬ нять некоторые действия. Разумеется, имеет место различная сте¬ пень понимания смысла действий и способность их выполнения разными людьми. Конечно, если человеку самому предстоит решить задачу, то он должен и понять смысл и выполнить действия, а не только уметь описать эти действия, В том случае, когда он состав- ii6
#яет описание алгоритма для его реализации другим человеком йли ЭВМ, то главным становится умение правильно описать каж¬ дое элементарное действие, потому что именно описание действия в его данном виде впервую очередь будет воспринято тем, кто вы¬ полняет алгоритм. Детализация описания алгоритма в основном зависит от того, кому он адресован. Если его выполняет машина, to описание доводится до элементарных действий, являющихся командами машины. При решении сложной задачи можно состав¬ лять описание алгоритма на разных уровнях, начиная с крупно¬ блочного описания с постепенной детализацией до элементарных действий, диктуемых исполнителем. УЧЕБНАЯ МОДЕЛЬ ЭВМ Структура ЭВМ демонстрируется с помощью «учебно-абстракт¬ ной ЭВМ». Для этой модели подобран предельно упрощенный вход¬ ной язык, на котором учащиеся приобретают навык составления простейших программ. Эта модель ЭВМ и. ее входной язык могут дать учащимся представление о некоторых наиболее общих основ¬ ных свойствах реально существующих ЭВМ и языков программиро¬ вания. Такая модель ЭВМ может быть использована в условиях работы школ сельских и отдаленных районов, так как она дает опре¬ деленные возможности «безмашинного» обучения программиро¬ ванию. Приемы обучения программированию, особенно на начальном этапе обучения, с использованием условной ЭВМ хорошо известны и широко распространены. Программирование на условной ЭВМ <и в содержательных обозначениях является составной частью мно¬ гих курсов программирования. Память машины. Устройство памяти иногда называют запо- минающим у^тройством (ЗУ). Память предназначаетсядля хране¬ ния информации. В ней хранятся как исходные данные для задачи, так и программа решения задачи. ХарактеристикамиЗУ являются’: объем памяти (сколько данных можёт быть размещено в памяти), время обращения к памяти (время записи информации в ячейку памяти или извлечения ее оттуда), способ обращения к памяти. Современные ЭВМ имеют несколько ЗУ, которые отличаются между собой характеристиками. Вся память машины обычно под¬ разделяется на внешнюю и внутреннюю память. В#утреннее за¬ поминающее устройство называют оперативным запоминающим устройством — оперативной памятью. Оперативная память обла¬ дает наименьшим объемом, но зато имеет минимальное время обра¬ щения по сравнению с другими видами ЗУ. Оперативная память имеет постоянную оперативную связь с арифметическим устрой¬ ством. При работе программы из оперативной памяти извлекается выполняемая команда. Затем из нее извлекаются или вызываются операнты соответствующей операции. Наконец в эту память отсы¬ лается на хранение полученный результат. 117
Внешняя память представляет собой магнитные барабаны, маг¬ нитные ленты и диски. Объем внешней памяти может быть весьма значительным, время обращения большее по сравнению с опера¬ тивной памятью. Обмен с оперативной памятью происходит минимальными пор¬ циями: записывается или считывается содержимое одной ячейки — чаще всего одно слово (одно число или одна команда). Обмен же с внешней памятью происходит значительными группами. Для учебной'машины рассматривается лишь оперативная па¬ мять. Для этой памяти берется фиксированное число ячеек, кото¬ рые пронумерованы, начиная с номера 0, т. е. номера 0, 1, 2, ... В пособии «Программирование» на рисунке 10 (с. 22) изображены 30 ячеек и отмечено, что в памяти имеются еще ячейки, например всего 100 ячеек. Для решения несложных задач обычно програм¬ мы не велики и такого количества ячеек достаточно. В случае, если с помощью этой машины начинают решать бо¬ лее сложные задачи и программы становятся длинными, а также много ячеек используется для числовой информации (исходные дан¬ ные, результаты'промежуточных вычислений и т. д.), то можно увеличить объем памяти. по своему усмотрению. В этом смысле объем памяти учебной машину практически неограничен. Номер ячейки является ее адресом в памяти. Буквенная ну¬ мерация введена для удобства программирования и для наглядно¬ сти программ. Фактически на машине используется лишь цифро¬ вая нумерация, а буквенные обозначения переменных (ячеек) но¬ сят условный характер; эти’обозначения временные, транслятор осуществляет кодировку букв в числа. Цифровые коды букв мы не используем и этим увеличиваем наглядность программы и облег¬ чаем усвоение элементов программирования. Цифровая нумера¬ ция используется лишь для указания номеров ячеек, в которых хранятся команды программы. Эти номера участвукц* в командах перехода, когда в них указывается номер команды (номер соот¬ ветствующей ячейки), на которую передается управление. И вэтом случае можно было бы использовать вместочисловых номеров яче¬ ек так называемые буквенные или относительные адреса, например перенумеровав все ячейки так: К, К + 1, ... . В этом случае пере¬ менные не должны обозначаться буквой К. Но это не упрощает ра¬ боту, и поэтому мы оставляем буквенную нумерацию лишь для ячеек, в которых содержатся значения соответствующих перемен¬ ных. Команды машины. Под «командой» принято понимать инструк¬ цию или приказ машине о выполнении какого-либо действия. Каж¬ дая ЭВМ способна выполнять несколько десятков различных ко¬ манд. Для начального изучения программирования обычно исполь¬ зуют не все команды, а лишь их небольшое подмножество, необхо¬ димое для решения несложных задач. По мере овладения приемами программирования можно изучить и другие команды машины, позволяющие составлять более эффективные программы. Для учеб¬ 118
ных целей мы ограничились следующим подмножеством команд: 1. Начало работы; НАЧАЛО 2. Конец работы; КОНЕЦ 3. Присваивание значения; А : = В 4. Сложение чисел; -С := А + В 5. Вычитание чисел; С : = А — В 6. Умножение чисел; С : = А X В 7. Деление чисел; С := А / В 8. Возведение в степень; С : = А | В 9. Ввод числовых значений; ВВОД {A) 10. Вывод числовых значений; ВЫВОД (А) lL Безусловный переход; НА N 12. Условный переход; ЕСЛИ А < В TO НА N Здесь в правой колонке приведены примеры записи команд. Через N обозначен номер команды, на которую осуществляется переход. Для простоты сначала рассматриваются первые десять команд и с их помощью составляются программы и объясняются принципы программного управления. Позднее вводятся и команды условного и безусловного перехода. С их помощью можно состав¬ лять и более сложные (содержательные, интересные) программы. Их понимание облегчено тем, что школьники к этому времени уже знакомы с машинным выполнением простейших программ. Понятие программы. Законченное описание алгоритма реше¬ ния задачи на машинном языке называют программой для данной машины. По исходным данным, следуя программе вычислений, ма¬ шина получает результат. Используя лишь первые 10 команд из всех команд учебной ма¬ шины, перечисленных в предыдущем пункте, можно составлять программы решения задач, алгоритмы которых являются линейны¬ ми (вычисление по формулам и др.). После рассмотрения про¬ граммы вычисления среднего арифметического двух чисел можно в качестве упражнения предложить составить программу вычисле¬ ния площади треугольника по заданным его основанию и высоте. В примере вычисления среднего арифметического двух заданных чисел программа составлена, можно сказать, в общем виде. По программе вычисляется значение выражения (А + В) / 2. Кон¬ кретные числовые значения переменных А и В считаются исход¬ ными данными. Числовой материал располагается непосредственно за командами программы (после команды КОНЕЦ). В учебном по¬ собии (с. 26) приводятся значения А = 3,24 и В = 15,07, Если бу¬ дет необходимо найти среднее арифметическое для другой пары чисел, то новые числа должны быть записаны за программой вме¬ сто старых. Составление программ в общем виде следует привет¬ ствовать, но полезно также знать, что программа может быть со¬ ставлена для конкретных чисел. В таких случаях числовой мате¬ риал, который принято располагать за программой, может отсут¬ ствовать, а числовые значения переменных будут задаваться в про¬ грамме командами присваивания. Приведем пример. 119
I вариант. 0. НАЧАЛО 1. А := 3.24 2. В := 15.07 3. С := А + В 4 Г •= Г / 2 • б! вывод {C) 6. КОЯ£Ц В данном случае за программой отсутствует числовой материал. Кроме того, отсутствует и оператор ввода значений переменных А и В. Исходные числовые значения переменных А и В задаются ко¬ мандами присваивания (см. команду 1 и 2). Если, составив такую программу, потребуется произвести расчет для других чисел, на¬ пример для чисел 537,235 и 2536,884, то в программе придется из¬ менить команды 1 и 2, записав в них новые числа. Можно предложить и другой вариант программы (для конкрет¬ ных числовых значений), в которой вообще отсутствуют перемен¬ ные А и В. II вариант. 0. НАЧАЛО 1. С := 3.24 + 15.07 2. С :=CI2 3. ВЫВОД (С) 4. КОНЕЦ В таком варианте для новых чисел придется изменить лишь команду 1. Как видно из примеров, наиболее употребительной для практических целей, особенно в массовых расчетах, является про¬ грамма, составленная в общем виде. Выполнение программы. Обычно команды программы выполня¬ ются, начиная с команды НАЧАЛО. Выполнив эту команду, нахо¬ дящуюся в ячейке 0, машина следующей выполняет команду из ячейки 1.. Вообще, выполнив очередную команду, находящуюся в ячейке с номером К, машина переходит к выполнению команды из соседней ячейки» имеющей номер на 1 больший, т. е. К + 1. Это естественный порядок выполнения команд в порядке их следования (последовательный порядок выполнения команд). Рассмотренные на данном этапе команды программы именно в таком порядке и выполняются. Далее приводятся команды, нарушающие этот по- следовательный^порядок выполнения. Команды проверки условий и- перехода. Команду перехода вида НА Т, где Т — номер команды, на которую передается управ¬ ление, часто называют командой безусловного перехода (обязатель¬ ного перехода). Обратим внимание на следующие случаи. Пусть имеется ко¬ манда: 19. НА 19 Выполнение этой команды, находящейся в ячейке 19, приводит к ситуации «зацикливания» — передачи управления «на себя». Это ошибка! 120
Пусть имеется команда: 19. НА 20 ^потребление в таком случае команды перехода лишено смысла. Естественный порядок выполнения команд означает, что после вы¬ полнения команды в ячейке 19 (если она не отсылает в другое ме¬ сто программы) происходит исполнение команды в'ячейке 20. По- втому указание о переходе из ячейки 19 в ячейку 20 является йз- лншним, хотя это не принято называть ошибкой. Команду проверки условия вида ЕСЛИ отношение TO НА Т принято называть командой условного перехода. Переход на коман¬ ду с номером Т осуществляется в том случае,, если отношение спра¬ ведливо. Под отношением здесь понимается выражение, состоящее из дву^ чисел или переменных, соединенных одним из знаков срав¬ нения. Примеры отношений: А > 0; 5 = В; 5 Ф 7; 5 «= 7; ...; А <C В. В этих примерах 3-е отношение справедливое, 4-е — не¬ справедливое. Справедливость других отношений зависит от зна¬ чения указанных в них переменных.Команда, в которой отношение всегда истинно, например «ЕСЛИ б > 0 TO НА 7», становится фактически командой безусловного перехода, т. е. всегда будет лереход на команду с номером Т, так как всегда верно, что 5 > 0. ПРИМЕРЫ ПРОГРАММ При разборе примеров программ рекомендуется вручную «про¬ крутить» программу, выполняя за машину команду за командой. При этом надо проследить за изменением содержимого ячеек, отве¬ денных для переменных, в процессе работы программы. Для этих целей следует составить таблицу значений переменных, участвую- .щих в программе, заполняя ее по мере «прокручивания» программы. Для примера рассмотрим программу вычисления произведения четных чисел от 2 до Т, где Т — заданное число. Обозначения: С ~- искомое произведение, А — очередной со¬ множитель, В — вспомогательная переменная для формирования четного числа. Программа № /. I вариант. . II вариант. 0. НАЧАЛО 1. ВВОД (Т) 2. С := / 3. В := 1 4 А ’ =3 2 X В 5. ЕСЛИ А > Т TO НА 9 6. С -:= С X А 7. В : = В + / 8. НА 4 9. ВЫВОД(С) 10. КОНЕЦ ' 0. НАЧАЛО 1. ВВОД (Т) 2. С := / 3. А := 2 4. С := С X А 5. А := А + 2 6. ЕСЛИ А < Т TO НА 4 7. ВЫВОД (С) 8. КОНЕЦ 121
Надо задать исходное значение Т. Пусть оно будет равно 6. Если взять Т большим, то таблица будет длиннее и «прокрутка» программы будет продолжительней. Значение Т задается и не из¬ меняется. Значения переменных А, В% С изменяются. Таблица может быть составлена различными способами» Составим ее следующим образом. № команды Т А В С Примечания J 6 2 1 3 i 1 4 2 5 Проверка условия A>T? 2 < 6 6 2 7 2 4 4 5 А > Т? 4 < 6 6 8 7 3 А I 6 m 5 А > Г? 6 = 6 6 48 7 4 4 8 5 A>T? 8 > 6 9 Вывод результата С — 48 В результате работы программы были сформированы четные числа 2, 4,6, 8 (переменная А). Четное число 8 оказалось большим Т и не включилось в произведение. Таким образом программой бу¬ 122
дет получено произведение четных чисел 2, 4, 6, т. е. С = 2 • 4 х X 6 = 48 (результат). Приведем еще примеры программ. П р и м е р 1. Составить программу вычисления суммы членов арифметической прогрессии, если даны первый член, разность и число членов. Сумма вычисляется по формуле Д ^+^тЧ.д. « . 2 Обозначения: S — искомая сумма Sn, А — первый член ait D — разность прогрессии d, N — число членов прогрессии n. Вспомога¬ тельные переменные: В — для вычисления 2аи С — для вычисле¬ ния d (п — 1). Программа № 2. 0. НАЧАЛО 1. ВВОД (А, D, N) 2. В := 2 х А 3. С : = N — 1 4. С := С X D 5. S := В + С 6. S := 5 X N 7. S := S / 2 8. ВЫВОД (5) 9. КОНЕЦ П р и м е р 2. Составить программу вычисления суммы кубов чи¬ сел 1, 2, ..., Т. Обозначения: С — искомая сумма, А —очередное слагаемое, В — счетчик. Программа № 3. 0. НАЧАЛО 1. ВВОД (7) 2. С := 0 3. В := 1 4. А := 1 5 С := С + А 6. ЕСЛИ В = Т TO НА 10 7. В := В + 1 8. А := В f 3 Q НА <Г ю! вывод (С) 11. КОНЕЦ П р и м е р 3. Составить программу решения уравнения А X X = = В с исследованием коэффициентов. Для удобства составления -программы введем вспомогательную переменную P', при этом можно условиться, что Р примет значениеО, 123
watti*#et решения, Р станет равным 1, если имеется одна решение, и Р примет значение 2, если решений бесчисленное множество. На печать будем выдавать значение Р и значение X, если будет одно решение. Программа № 4. 0. НАЧАЛО 1. ВВОД (А, В) 2. ЕСЛИ А = 0 TO НА 7 3. X:=BlA 4. ВЫВОД (X) 5. Р : = 1 6. НА 11 7. ЕСЛИ В * 0 TO НА 10 8. Р := 0 9. НА 11 10. Р := 2 11. ВЫВОД (P) 12. КОНЕЦ Если имеется одно решение, печатается сначала значение X, а затем значеииепризнака Р. В других случаях печатается только одно число —.значение Р. В данной теме рассматривались различные формы представле¬ ния и описания алгоритмов: словесное описание алгоритма с исполь¬ зованием математической символики, графическое изображение алгоритма с помоЩью блок-схем и описание алгоритма наязыке учебной ЭВМ. Полученные сведения являкл'ся начальным этапом в изучении программирования. Дальнейшим развитием рассматриваемой темы может быть: а) изучение какойглкбо реальной ЭВМ и непосредственная работа с ней; б) изучение одного из распространенных алгоритмических язы¬ ков (АЛГОЛ-бО, ФОРТРАН и др.) и программирование на нем. П Р И М Е Р Н О Е С О Д Е Р Ж А Н И Е 3 А Ч Е Т А а) Что такое алгоритм? б) Каково назначение и как изображаются арифметические и логические блоки в блок-схемном представлении алгоритма? в) Из каких основных устройств состоит учебная модель ЭВМ? г) Что понимается под программой для ЭВМ? д) Какиё команды может выполнять учебная модель ЭВМ? На зачете каждому школьнику можно предложить один-два вопроса и. задание, содержащее построение блок-схемы какого- либо алгоритма и написание программы для учебной ЭВМ. 124
Ниже дается ориентировочное содержание индивидуального зачета. 1. Из каких основных устройств состоит учебная модель ЭВМ? 2. Составить блок-схему алгоритма вычисления значениц функ- ции у = Vx +1 для всех целых значений х от 0 до В. 3. Составить программу (для учебной ЭВМ) получения значений функции у — х*—1 для значений х от —3 с шагом 0,01 до 3. Дополнительные упражнения 1. Составьте описание алгоритма вычисления значений функ¬ ции у =• Vх для всех целых значений х от 1 до А. 2. Составьте описание алгоритма получения таблицы значений функции у = х3 для значений х от —8 до 8 с шагом 0,02. ' 3. Составьте программу вычисления функции у — | х — 8|ддя целых значений х at ^60 до 60. 4. Составьте программу вычисления площади треугольника. 5. Составьте программу вычисления rt-ro члена арифметической :лрогрессии, если известны первый член, разность прогрессии и номер искомого члена. 6. Составьте программу вычисления суммы п членов арифмети¬ ческой прогрессии, если известны первый член, последний член и йисло членов. 7. Составьте программу вычисления n-го члена геометрической Прогрессии. 8. Составьте программу вычисления суммы п членов геометри¬ ческой прогрессии. 9. Составьте программу вычисления произведения нечетных №сел от 1 до n. 10. Составьте программу вычисления суммы квадратов чисел Фт 1 до п. 11. Составьте программу вычисления значения квадратного Трехчлена. 12. Составьте программу вычисления значения функции: У = Где* — 1, если х ^t 0, (* + 1, если х < 0. Ответы, решения, указания к дополнительным упражнениям 1. Описание алгоритма (предполагается, что А ^ 1). шаг 1. Положить х = 1. шаг 2. Вычислить у = Vx. шаг 3. Записать значение у. шаг 4. Положить х = х + 1. шаг 5. Если х ^ А, то перейти к шагу 2. шаг 6. Окончание работы. 125
2. Описание алгоритма. ■ шаг 1. Положить х = —8. шаг 2. Вычислить у = х2. шаг 3. Записать значение у в таблицу, шаг 4. Увеличить значение х на 0,02. шаг 5. Если х < 8, то перейти к шагу 2. шаг 6. Окончание работы. 3. Программа № 5. 0. НАЧАЛО 1. X : = —60 2. ЕСЛИ X > 0 TO НА 5 3. V : = —X + 8 4. HA 6 5. Y : = X — 8 6. ВЫВОД (Y) 7. X : = X + / 8. ЕСЛИ X < 60 TO НА 2 9. КОНЕЦ 4. Указание. Программа зависит от выбора формулы вычисления площади треугольника. 5. Вычисления выполняются по формуле а„ — аг + (и — 1) d. Обозначения: В — п-й член ап, А — первый член аг, D —разность прогрессии d, N — номер искомого члена n. Программа № 6. 0. НАЧАЛО 1. ВВОД (А, D, N) 2. В : = N — 1 3. В : = В х D 4. В : = В + А 5 ВЫВОД (В) 6. КОНЕЦ 6. Вычисления можно выполнять по формуле 5 = (gi Н~ °n) ^ п Обозначения: А — первый член аи В — член ат N — число чле¬ нов п, S — результат. Программа № 7. 0. НАЧАЛО 1. ВВОД (А, В, N) 2. S : = А + В 3. 5 : = 5 х N 4 9 • = S / 2 5. ВЫВОД (5) 6. КОНЕЦ 126
7, Вычисления выполняются по формуле ь„ = t>n-i ■ Я или bn = bi • q"^1 (п > I). Обозначения: Q —. знаменатель прогрессии q\ N — номер искомого члена п; В — первый член bit затем используется для значений Ьп-й Р — результат, используется для значений b„; Т — счетчик числа членов. Программа № -8. 0. НАЧАЛО 1. ВВОД (В, Q, N) 2. Т : = 1 3. Р : = В х Q 4. Т : = Т + 1 5. В : = Р 6. ЕСЛИ Т < N TO НА 3 7. ВЫВОД (P) 8. КОНЕЦ 8. Вычисления выполняются по формуле Обозначения: 5 — искомаясумма 5„, В — первый член blt Q — знаменатель прогрессии q, N — число членов п, С — вспомога¬ тельная переменная. Программа № 9. 0. НАЧАЛО 1. ВВОД (В, Q, N) 2. С : = Q f N 3. С : = С — / 4. S : = С X В 5. С : = Q — 1 6. S : = S / С 7. ВЫВОД (S) 8. КОНЕЦ ^. Указание. Можно воспользоваться программой № 1 (I ва¬ риант), в которой за строкой 4 записать дополнительно команду А : = А Ц- /, 10. Указание. Можно воспользоваться программой № 3, за¬ менив команду в строке 8 на команду А : = В f 2. 11. Вычисляется / значение у = АХ2 + ВХ + С по заданным значениям А, В, С и X. Программа № 10. 0. НАЧАЛО - 1. ВВОД(А,В,С,Х) 2. Y := А 3. У : = Y X X 127
4. V := Y + B 5. Y := У X X 6. Y :=»У +'C 7. ВЫВОД (Y) 8. КОНЕЦ 12. Программа № 11. 0. НАЧАЛО 1. ВВОД(Х) 2. ЕСЛИ X < 0 TO НА 6 3. Y : = X f 2. 4. Y : = Y — 1 5. НА 7 6. Y : = X + 1 7. ВЫВОД (У) 8. КОНЕЦ ДОПОЛНИТЕЛЬНАЯ ЛИТЕРАТУРА Д е м и д о в и q Н. Б., М о н а хов В. М. Программирование и ЭВМ. И. 1977. А л ф е р о в а 3. В. Теория алгоритмов. М., 1973. П е р в и н Ю. А. Основы ФОРТРАНА. М., 1972. JI а в р о в С. С. Введение в программирование, 2-е изд. М. 1977. А б р а м о в С A., А u т и п о в И. Н. Основы программирования на АЛГОЛЕ. М., 1980.
ТРАНСПОРТНЫЕ СЕТИ Рассмотрим экономические задачи, решаемые I помощью графов, ребрам которых приписаны неотрицательные числа. ЗАДАЧИ, ПРИВОДЯЩИЕ К ТРАНСПОРТНЫМ СЕТЯМ Предр^им себе город, расположенный на пути к местам мас> соврго летнего отдыха. Улицы этого города обычно загружены сво¬ им «внутреннем» транспортом. В летний период город пропускает дополнительный непрерывный поток машин. Для избежания заторов этот поток распределяется по улицам с учетом их пропускной спо¬ собности. На рисунке 1 показаны возможныепути движения до¬ полнительного потока машин по улицам города в направлении от В0 к Вп. Здесь месту въезда'в город, месту выезда из него, перекре¬ сткам поставлены в соответствие вершины графа, а улицам, соеди¬ няющим их, — ребра. Для распределения дополнительного потока машин необходимо знать пропускную способность каждой улицы, т. е. число машин, которые одновременно могут проехать через «поперечноесечение улицы» за единицу времени. На рисунке 2 над ребрами поставлены числа, указывающие на пропускную способность.соответствующих улиц. Здесь пропускная способность ребра <В0; Bj> равна 3, а ребра <Bi, В8> равна4. Это означает, что наибольшее чис¬ ло машин, пропускаемых путем <В0; Bt> в единицу времени^ равно 3, а путем <Bt\ Вй> — равно 4. При заданных условиях требуется определить максимальное значение потока в сети улиц, а в конкретном случае — наибольшее Рис. I Рис. 2 5 Заказ 43 129
Рис. 3 чйсломашин, которое может пропустить сеть улиц в единицу вре¬ мени от В0 к Вп. Для решения этои задачи подсчитаем сначала, сколько машин (в единицу времени) могут пропустить улицы, «выходящие» из В0. Для этого условно «рассечем» их поперек (рис. 3) и найдем сумму их пропускных способностей: 3 + 4 + 5 — 12. , Все ли 12 машин (за единицу времени) сможет принять Вп? Нет, не все, так как только 2 + 6 + 1 = 9 машин могут пройти (за единицу времени) по улицам, ’«входящим» в Вл (рис. 4). Напо¬ минаем, что мы рассматриваем только непрерывные потоки. Но оказывается, что и 9 машин не могут быть пропущены (в единицу времени) через улицы города, поскольку пропускные спо¬ собности улиц города для этого недостаточны. Чтобы убедиться в этом, снова рассмотрим поперечное сёчение улиц города, проводя¬ щих транспорт от В0 к Вп, например, такой, который выделен на рисунке 4 двойной штриховой линией, и найдем сумму их*2 + + 1 + 3 + 1 + 1 = 8. Если рассмотреть все другие «поперечные сечения» улиц города и подсчитать сумму их пропуокных способностей, то окажется, что наименьшая из них равна 8. Интуитивно ясно, чтопоток значе¬ нием больше 8 этой сетью в единицу времени пропущен быть не мо¬ жет. Докажем* что 8 машин одновременно смогут пройти через любое «поперечное сечение» улиц города. Для этого распределим 8 машин по улицам, учитывая их пропускную способность. Один из вариан¬ тов распределения такого потока представлен на рисунке 5. Число машин, пропускаемых одновременно через поперечное сечение ули¬ цы, проставлено над соответствующими~ребрами правее их про¬ пускной способности. (Найдите для этой сети другое распределение по ребрам потока значением 8.) т Таким образом, для сети на рисунке 2 найдено значение мак¬ симального потока и показано, как этот поток можно распределить по ребрам. Заодно мы выяснили, что значение потока в сети не мо¬ жет быть больше, чем сумма пропускных способностей любого из «поперечных сечений» сети (к обсуждению этого факта мы еще вернемся). 130
Рис. 4 При рассмотрении этой задачи вы получили представление о таких важных понятиях, как «сеть», «транспортная сеть», «пропуск¬ ная способность ребра», «поток по ребру», «поток в сети», «разрез сети», «пропускная способность разреза». На конкретном примере убедились, что значение максимального потока определяется наи¬ меньшей пропускной способностью из множества всевозможных раз¬ резов сети. Конечно, рисунком 2 или подобным ему может быть изображена не только схема улиц города, но и какая-то другая транспортная сеть, содержащая пункт отпраЬления В0, пункт назначения Ва, "промежуточные уЗловые пункты Blt В2, В3, В4, Вь и пути, связы¬ вающие их между собой. f Если на рисунке 2 представлена сеть железных дорог, связы¬ вающих ряд пунктов, то из В0 в Вп может переправляться какой-то ^говар. Предполагаем только, что перемещаемый товар однородный йи его можно перераспределять в промежуточных пунктах. Если это ^сеть трубопроводов, то из В0 в В„ может переправляться, напри- умер, нефть. Эту же сеть можно рассматривать как систему каналов, ^пособных переносить поток информации. Это может быть телефон¬ ная или телеграфная сеть или электросеть и т. п. Промежуточные пункты можно рассматривать как места, в ко- горых происходит перераспределение потока с одних путей на дру¬ гие. Число, поставленное над ребром, указывает на пропускную ' % Рис. S а» i3i
Рис. 6 Рис. 7 способность ребра, т. е. на максимальное значение того потока (товаров, нефти, информации), который соответствующий путь может через себя пропустить в еди¬ ницу времени. Эти числа имеют естествен¬ ную природу, так как каждый участок трубопровода может пропустить только определенноеколичество нефти в час, а линия связи может обслужить только оп¬ ределенное число вызовов в день и т. п. Считаем, что некоторое число единиц (TQpapa, нефти или информаций) движется из Вь вдоль направленных путей; попадая в промежуточные узлобые пункты, оно пе¬ рераспределяется и движется дальше по новым пуЧгям до тех пор, пока все не по¬ падет в пункт Вл. Таким образом,на сеть накладывается условие, согласно которо¬ му все, что попадает в промежуточный пункт,должнои выйти из^ него. Ясно,что такогЬ типа ' задачи сводятся к решению следующей сетевой задачи. Задача о максимальном значении по¬ тока в сети. Определить максимальное значение потока, который может пропус¬ тить транспортная сеть из пункта отправ¬ ления В0 в пункт назначения Вп% если известна пропускная способность каждого ребра. Рассмотрим экономическую задачу, ко¬ торую можно описать и решить с помощью сетей. Пусть в какой-то области есть две мебельные фабрики Ф{ и Ф2, которые снабжают мебелью три магазина ^- Mi, Мг и М3. Нас будут интересовать только книжные шкафы. В начале месяца каж¬ дый из магазинов делаёт заявку на определенное число книжных шкафов, т. е. заказывает столько шкафов, сколько предполагает продать в течение месяца. Это позволяет-установить^бщее число книжных шкафов, которые требуется изготовить на этих двух фаб¬ риках. Пусть магазину Л^требуется 10 шкафов, магазину М2 — 8, магазину Мэ — 7, т. е. всего 25 книжных шкафов. Пусть на фаб- рике.0! изготовлено 11 шкафов, на фабрике Ф2 — 14 (рис. 6). Из¬ вестна стоимость Сц перевозки одного шкафа по каждому из путей от фабрики Ф1 до магазина Mj. Так, сп — 80 к., cl2 = 60 к., Ci3 = 1 p., Съ = 90 к., с88 = 50 K., с23 ^= 70 к. Поставим эти цены иад соотбетствуклцими ребрами (рис. 7). Требуется определить, сколько шкафов следует отправить с.каждои фабрики в каждый из Рис. 8
^магазинов, чтобы общая стоимость всех перевозок быламинималь- ной. ' - Для решения задачи, естественно, попытаться привезти най- ; большее число шкафов по путям с наименьшей стоимостью, В ма- газин Mi доставим всеЛО шкафов с фабрики Ф*, избежав транспор¬ тировки по пути Ф2М1л В магазин Мг доставим все 7 шкафов с фаб¬ рики Ф2, избежав пути Фх М3. Тогда в М2 остается^привезти 1 шкаф из Ф± и 7-шкафов из Ф2(рис. 8). При.такой планировке перевозок фабрика Ф± отправит 10 шкафов; фабрика Ф2 ~ 14. Общее число шкафов, полученных магазином Ми равно 10 + 0 = 10; магази¬ ном М2, равно 1 +ч7 = 8; магазином М3, равно 0 + 7 = 7. Под¬ считаем общую стоимость перевозки: (80 • 10 + 60 * 1 + 100-0) + + (9& • 0 + 50 - 7 + 70 • 7) = 1700 к. или 17 р. Можно составить и другие планы перевозок, но найденный план требует, действительно, минимальных затрат. Как искать план перевозки, требующей минимальных затрат, для аналогичныхи более общих задач? Существует ли такой план?. Методы линейного программирования (одного из разделов современной математики) позволяют утверждать, что такой план существуёт и его можно найти*. Врассмотренной частной зЗдаче считается, что пропускная способность путей достаточна. для любого варианта перевозки. •В более общем случае дополнительно задается пропускная способ- кость каждого пути. Задачи подобного содержания принято назы- Узать«транспортными задачами». В более общем случае вместофабрик рассматриваются пункты отправления, вместо магазинов — пункты &азначения**.Понятно,что математическая структура такого типа |адач позволяет применять к их решению сетевые методы и сводить kx к следующей сетевой задаче. Задача о потоке минимальной стоимости. В транспортнсйГ ce- Шн требуется так распределить заданное количество товаров по Йутям, чтобы общая стоимоеть транспортировки была бы минималь- ^ой.Предполагаются известными количествотовара, которыйне- £р0ходимо переправить по сети из В0 в ВпУ стоимость перевозки еди- ^йцы товара по каждому участку пути (ребру), пропускная способ¬ ность каждого пути. ОСНОбНЫЕ понятия Прежде чем говорить о методах решения приведенных- выше задач, уточним некоторые понятия. ’ Сетыо называется связный ориентированный граф без петель, ^ котором каждому ребру приписывается некоторое неотрицатель- йое число. „ г Число, которое приписывается ребру, называется nponycmgfi способностью ребра. * См.: Гасс С. Линейное программирование. М., 1961. ** Формулировка транспортной задачи в общем случае будет приведена на е. lf5; .133
в5 в6 Рис. 9 'BS В$ Рис. 10 Пропускную способность ребра <Bt; В} > будем обозначать а1} или а < Bt; Bf >. Пропускную способность ребра можно рас¬ сматривать как наибольшее число единиц, например товара, кото¬ рый можно переправить по ребру в единицу времени. На рисунке 9 изображена сеть, в которой вершины Bi и Вь явля¬ ются источниками, а вершины В3 и BR — стоками. Сеть, в которой только один источник и только один сток, назы¬ вается транспортной сетью. Транспортную сеть будем обозначать буквой Т. При решении экономических задач любая сеть может быть све¬ дена к сети, в которой только один источник и один сток. Например, сеть, изображенную на рисурке 9, можно преобра¬ зовать в транспортную сеть так, как показано на рисунке 10. Здесь добавлены вспомогательные вершины В0 и Вп и соответствующие ребра, причем a0j = a12 + су4, a05 ~ a64 ^^^ а5в» аЗп ~ а23 ~Ь ®43» авя ~ ®4в ^J^ ®6в* Потоком по ребру называется функция q>, заданная на множестве ребер сети, удовлетворяющая следующим условиям: 1) ф <B,; £у->.для каждого ребра целое и неотрицательное число; 2) q> < В,\ Bk > ^ alk для каждого ребра; п П' 3) 2 *P ^ 5i; ^k> = '2i4<Bb* Bj>WH каждой промежуточной t=0 /'=0 вершины Вк. В общем случае считают, что q> <Bt\ Bj > — 0f если Т не со¬ держит ребра < fl,; Bj>. Значение функции ф <£(.; Bj> можно рассматривать, напри- мер, как количество товаров, перевозимыхпо пути < B,; Bj > пря данном распределении потока. Условие 2) в определении потока по ребру вполне естественное* оно требует, чтобы количество пере¬ возимых по ребру товаров не превышало пропускной способности ребра. Условие 3) означает, что всякий промежуточный пункт, приняв какое-то количество товаров, должен этожеколичеството- 134
варов и выдать. В промежуточных пунктах товар как бы только перераспределяется по другим путям для продвижения дальше. Потоком в транспортной сети Т называется функция f (T), равная сумме потоков по ребрам, выходящим из источника В0, т. е. / (T) = S Ф <В°; Bi >• i=t Значение функции / (T) можно рассматривать как общее коли¬ чество товаров, передаваемых по сети в единицу времени из пунк¬ та В0 в пункт Вп. Для сети Т (на рисунке 10) f (T) =* 14. Из определения потока по ребру следует, что сумма потоков по ребрам, выходящим из В0, равна сумме потоков по ребрам, вхо¬ дящим в Вп, т. e., что f (T) = 2 Ф <Bo, Bt> = 2 Ф <B,', в„>. i=i i=i (Доказательство этого утверждения опускаем.) В дальнейшем ф <Bt\ В}> будем обозначать у1}. Естественно возникает задача отыскания максимального зна¬ чения потока в сети Т с заданной пропускной способностью ребер. Максимальное значение потока в сети Т будем обозначать Jma* (T)- Рассмотрим транспортную сеть на рисунке 11. Вершина В0 — источник, вершина Ва — сток, а0£ — 8, a0s = 5, ааа = 3, a23 = 2 и т. д. В этой сети ребра, входящие в вершину Ва, могут пропустить в единицу времени 15 единиц товара, а ребра, выходящие из В0, Только 13 единиц. Все да эти 13 единиц можно распределить по ребрам так, чтобы онй достигли вершины J3„? Попытаемся это сде¬ лать. Число единиц товара, переправляемого по ребру в единицу вре¬ мени, проставим над ребром в скобках правее числа ау. Один из вариантов распределения 13 единиц товара по ребрам сети показан ка рисунке 12. 3(3) 8? Рис.- 11 Рис. 12 135
Упражнения 1. По ребрам сети, изображенной на рисунке 11, распределите двумя разными способами поток, значение которого равно 10. 2. На рисунке 12 показан один из вариантов распределения потока в сети, значение которого равно 13. Найдитедругой вариант распределения по ребрам потока того же значения. 3. На рисунке 13 изображена транспортная сеть Т. а) Распределите значения потоков по ребрам в сети Г.так-, чтобы по сети можно было пропустить поток f (T), значение которого было бы равно~12. ' б) Почему число f (T) = 12 не является максимальным значе¬ нием потока в этой сети? в) Найдите максимальное значение потока в этой сети, т. е. f (T), и приведите одно из его распределений по ребрам. 4. Определите максимальное значение потока в транспортной сети Т (рис. 14). Найдитедва из возмоЖных вариантов распределе¬ ния значений нотока по ребрам, при котором по сети можнсипро- пустить поток f (T) с максимальным значением., Уточним еще два -понятия — «разрез в транспортной сети» и «пропускная способность разреза». Пусть множество вершин транспортной сети Т разбито на два непересекающихся подмножества Р и Р9 причем так^что Р содер¬ жит источник сети~В0, а Р — сток Вп (рис. 15). Здесь Р = {В0кВ1у В3}, Р = {5а, Вп}. Удалим из сети ребра, соединяющие Р и Р: <Вг; В2>, <В3;В2>, <В3; Вп>. При их удалении сеть разо¬ бьется на две, не связанные между собой сети (рис. 16). Подсчитаем, чему равна пропускная способность всех ребер, которые были направлены от вершин множества Р к вершинам мно¬ жества Р. «12 + «32-+. «3л = 12 + 1 +'2 = 15. Это означает, что по этим ребрам в направлении от B^ к Вп в единицу времени может пройти только 15 единиц товара. Разрезом транспортной сети называется множество ребер,' при удалении которых сеть распадается на две не связанные между Рис. 13 Рис. 14 136
собой части, в рдноб из которых находится источник, в другой — сток. Разрез можно задать одним из множеств 'P или Р. Для одной гЭ той же сети можно задать несколько разрезов, i. Лропускной способностью разреза называется сумма пропуск- $ых способностей тех ребер разреза, которые направлены от вер- ;Шин множества Р к-вершинам множества Р. Пропускная способность разреза обозначается а (P, .P); можно записать, что а tР, P) = 2ау ^ля таких значений i и /, где Bt € Р, Bj € Р. Для разреза сети Т (рис. 15) а (P, Р) = 15.Еслидлязтойжесетисделатьтакойразрез, рак показано на рисунке 17, то (P, P)- 14. Для разреза этой же сети, которьш определяется множеством ^= {£0, Bi}, пропускная спо¬ собность а (P, P) ~ аi2 + Sh аоз — 16.- Здесь пропускная £&особность ребра <В3; Ву > ^e учитывается, так как это сребро направлено от вершин ^ножества Р к вершинам мно* жества Р. Мы убедились, что разные разрезы одной и той же сети Т Йогут иметь разные пропускные Ёпесобности. Всегда можно вы,- брать разрез с ■ минимальной пропускной способностью.- (Та* хих разрезов может оказаться несколько.) Для сети Т на рисунке 17 раз¬ рез с минимальной пропускной способностью один:{<fi0; Bt>, <Вй\ В3>), сЦЯ, P) равна 14. ,137
Упражнения 5. На рисунке 18 изображена транспортная сеть Т. 1) Определите а (P, Р), если а) Р {B0f Ви В3}; б) P={B^ Ви BZf #5}; в) Р = {£0}* 2). Выделите какие-нибудь 3 разреза в сети Т,отличные от перечисленных выше, и определитеих пропускные способности, j3) Какой из возможных разрезов этой сети имеет минимальную пропускную способность? 4) Можно ли задать по¬ ток /(Г), значениекоторогобылобыбольше, чем минимальная про¬ пускная способность хотя бы одного разреза в этой сети? 5) Опре¬ делите максимальное значение потока в этой сети. 6) Сравните про¬ пускные способности разрезов и fmax (T). 6. Как вы считаете, существует ли сеть, в которой максимальное значение потока было бы больше,чем пропускная способность хотя бы одного разреза? 7. Для транспортной сети Т, изображенной на рисунке 14, вы¬ полните задания, перечисленные в упражнении 5. 8. Выделите какие-нибудь 4 разреза (задавая их множеством Р) в сети Т, изображенной на рисунке 10, и определите их пропускные способности. 9. Рассматривая транспортную сеть ^C как сеть газопроводов или сеть нефтепроводов, попробуйте объяснить тот факт, что зна¬ чение любого потока в сети не больше пропускной способности лю¬ бого разреза в этой сети. О ЗАВИСИМОСТИ ЗНАЧЕНИЯ ПОТОКА В ТРАНСПОРТНОЙ СЕТИ *ОТХ1РОПУСКНОЙ СПОСОБНОСТИ ЕГО РАЗРЕЗОВ ^ Сравнивай значения потоков в любой транспортной сети и про¬ пускные способности всех возможных разрезов этой сети, можно убедиться в том, что значение любого потока больше или равно пропускной способности любого из разрезов. Именно это утвержда¬ ется в нижеприведенной теореме. Интуитивно эта теорема должна быть понятной. Строгое обоснование ее здесь рассматриваться не будет. Т е o p е м а 1. Значение любого потока f (T) транспортной сети Т меньше или равно пропускной способности любого из разрезов*. Самой важной теоремой в теории сетей является следующая тео¬ рема, известная кактеорема Форда и Фалкерсона. Т е о р е м а 2. В любой транспортной сети максимальное, зна¬ чение потока равно минимальной пропускной способности из всех возможных разрезов. Для доказательства теоремы 2 достаточно показать, что суще¬ ствуют такие поток / (T) и разрез (P, Р), для которых значение по¬ тока и пропускная способность разреза равны. * С доказательством этой теоремы можно познакомиться, например, в кни¬ ге Ф о р д Л. Р.* Ф а л к е р с о н Д. Р. Потоки в сетях. М., 1966, 138
Доказательство этой теоремы мы опустим, но расскажем об одном из алгоритмов отысканий максимального значения потока. Для описания алгоритма нам потребуются новые понятия. Ребро <Bt; Bj> будем называть насыщенным, если Известно, что в сети От источника В0 к стоку Вп может вести несколько путейЛ, /2, ..., lk. Поток f (T) в сети называют полным, если каждый путь от В0 к Вп содержит по меньшей мере- одно на¬ сыщенное ребро. Из этого определения следует, что поток f (>Г) является непол¬ ным, если существует по меньшей мере один путь от В0 к Вп, не содержащий'ни одного насыщенного ребра. Для нахождения.максимального значения потока естественней сначала найти какой-то путь /j от В0 к Вп, который не содержит на¬ сыщенных ребер. Для всех ребер этого пути определяют разности &У — ф 1}. Находят наименьшую из них Д = min (ai} — ф,у) для веех ребер, принадлежащих 1г. > Далеек значениям %} по всем ребрам пути 1г прибавляют чис¬ ло А. После этого в пути появится по меньшей мере одно насыщен- вое ребро. Аналогично преобразовывают каждый путь, от В0 к Вп, не со¬ держащий ни одного насыщенного ребра. (Заметим, что различных путей от В0 к В„ в сети конечное число и их все можно перебрать.) После этого каждый путь, будет иметь хотя бы одно насыщенное ребро. Потокстановится полным: Но оказывается, что не всегда полный поток имеет максимальноезначение. Для некоторых сетей значение потока можно увеличить, даже если этот поток уже является полным. Рассмотрим два примера. П р имер 1. На рисунке 19 изображена транспортная сеть. По- Ток по всем ребрам равен 0. Сделаем поток полным. Выберем ка¬ кой-нибудь путь ot В0 до Вя. Пусть )j проходит через вершины В0, fi,, В3, Вп. По этому пути можно пропустить поток, значение ко- тЬрого равно 8. Ребро <В0; В2> в этом пути стало насыщенным. По пути /2, проходящему через В0, В1г В„, можно пропустить Поток, значение которого равно 2 фис. 20). Насыщенным стало рёбро <Bi, Вп >. <p <Вг; Bj> = аи. Во 40:n 8r 8? Ю;В В3 Рис. 19 Рис. 20 139
Рис. 21- Рис. 22 Поток значением 1 можно еще пропустить по пути /3, проходя- ,.,ow.. ..Л«^ в в- п tf^ (рИС 21). Насыщенными стали ребра Все возможные пути от В0 до Вп в этой сети можно представить в виде дерева, изображенного на рисунке*22. Здесь в каждом пути от В0 до Вп есть по меньшей мере хотя бы одно насыщенное ребро (они выделены более жирной линией). Итак, поток оказался пол- Пропускная способность разреза, заданного множеством Р = = {В0, Ви В2, В3}, равна 9 + 2 = 11. Этот разрез имёег мини-% мальную' пропускную способность. Пропускные способности всех* других разрезов, не меньше чем 11. (Проверьте это.) Следовательно, в этом случае полный поток имеет максимальное значение. П р и м е р 2. На рисунке 23 изображена транспортная сеть. В ^аждом пути от В0 дд В„ есть уже хотя бы одно насыщенное ребро, т. е. поток.являетсй полным, его значение равно 6. Ни по одному пути от В0 до В„ поток увеличить уже нельзя. (Проверьтеэто.) Если сравнить пропускные способности всех разрезов, то мень¬ шая из них равна 8. (Это тоже проверьте.) Как же в этом случае определить поток с максимальным значением? Оказывается,‘что увеличить значение потока можно другим способом. Заметим* что ребро <В0, Ва> еще не насыщено. Увеличим Фоа на 2; уменьшим Ф12 на 2 и увеличим ф5„ тоже на 2. 3ндчение потока в сети увеличи¬ лось на 2. При этом в промежуточных вершинах не нарушено усло¬ вие сохранения потока, т. е. значение «входящего» потока осталось равным значению «выходящего» потока (рис. 24). ным, его значение равно 11. Рис. 23 Рис. 24 140
Итак, значение потока в яранспортной сети можно уве¬ личивать не только ;за счет изме- до Вп, но и при изменениях зна¬ чений <ру в такой последователь- 'n ности ребер, соединяющих В6с ^Ft В„, в которой не все ребра нап- i Правлены от В0 к Вп. В таких слу- рис. 25 !чаях на ребрах, направленных е этой последовательности от 'Фя к fio, значение потока умень¬ шают. У пражнени я Ю. Опираясь на теорему 1, докажите, что если в сети Т значе- ?йиё потока оавнопоопускной способности какого-то из оазоезов j^(P,PJ— минимальная пропускная способность йз всех разре¬ зов сети. If. Нзрисуйте дерево всевозможных путей, ведущих от В0 0o В„, для сети на рисунке 25 и убедитесь в том, что вдоль путей ^рт В0 до Вп (при заданном распределении потока-по ребрам) зна¬ чение потока в сети увеличить нельзя. 12. На транспортной сети Т (рис. 25) задан полный поток, его рачение не является максимальным. Путем последовательных изме- ;яений. заданных значений фу увеличьте значение потока в сети. 13. В транспортной сети Т (рис. 26) задан поток. На сколько Зйдиниц и каким образом можно увеличить ёго значение? 14. В транспортной сети (рис. 27)_задан поток. На сколько еди- Йиц и каким образом можно увеличить его значение? Нами рассмотрены несколько простых сетейгмаксимальное зна¬ чение потока в которых нетрудно было определить по рисунку. рЭдача усложняется для сетей с большим числом вершин. Для оты- зС&ания максимального значения потока в транспортной сети с лю- Збым числом вершин разработаны различные алгоритмы. Приведем Йлгорйтм, являющийся эффективным, если в сети даже несколько Bi 5A fo fe г;г Ъ % Рис. 26 Рис. 27 141
тысяч вершин*. Используют его при решении сетевьж задач на ЭВМ. Если при чтении возникнут трудности, то с этим алгоритмом можно не знакомиться и переходить к йункту «Родственные задачи». ОБ АЛГОРИТМЕ НАХОЖДЕНИЯ МАКСИМАЛЬНОГО ЗНАЧЕНИЯ Алгоритм нахождения максимального значения потока в тран¬ спортной сети состоит в последовательном выполнении двух опе¬ раций: операции расстановки пометок вершин и операции измене^ ния потока. Сначала в сети задают некоторый поток, хотя бы нулевой. Операция меток вершин состоит в следующем. 1. Отмечают источник В0 индексом 0. 2. Вершины, смежные с В0, помечают индексом +0. 3. Если В, — уже помеченная вершина, а Bj— непомеченная и от Bt к В, ведет ребро, причем <p <В(', Bj> <ау, то Ву'помечают индексом +t. Те непомеченные вершины Bj, из которых ребра направлены к Bj и для' которых ф <Bj', Bt > большеО, помечают индексом —i. Процесс метки вершин продолжают до тех пор, пока не отме¬ тится сток либо пока ни одну из непомеченных вершин нельзя бу¬ дет пометить. При такой процедуре может быть помечена лишь часть вершин сети. Если сток Вп оказался помеченном, то это означает, что поток в сети можно увеличить. В этом случае переходят ic операции уве¬ личения потока в сети. Операция увеличения потока состоит в сле¬ дующем. 1. Выделяют последовательность 1 ребер, все вершины которых различны и помечены индексами предшествующих вершин, взйтыми со знаком «+» или со знаком «^-». Заметим, что не все ребра в этой последовательности могут быть одинаково направлены от В0 к В„ (рис. 28). 2. Определяют, насколько можно изменить поток по всем реб¬ рам последовательности /. Для этого находят неотрицательное чис- ПОТОКА ло Д = min {(oty— фу); ф„?) для таких значений i й /# при кото¬ ^+^ программирования, но алгоритм Фор- °п да и Фалкерсона (или итерационный процесс метки вершик) является бО' Рис. 28 лее эффективным.
рых <Bt; Bj> £ I и направлено от Во к Вп, и для таких значе¬ ний р и q, что <Вр\ Bti > С 1 и направлено от Вп к В0. 3. Увеличивают значение по¬ тока в сети на Д; а для этого: i а) увеличивают значение фу на .Д, если направление ребра <В4; Bj > совпадает с направ¬ лением движения в последова¬ тельности от В0 к Вп, и б) уменьшают значение фр1? на Д, если направление ребра <Вр\ Bq> в последовательности 1 противоположно направлению движения от В0 к Вп. '• v После этого все старые меткичстирают и начинают операцию ме¬ ток вершин сначала*. Заметим, что в процессметки вершина Bj не сможет быть по¬ мечена: 1) индексом +f предшествующей вершины, если только ребро - <B^Bj> уже насыщено, т. е. ф <Вг; Bj> = ау; 2) индексом —i, если только ф<Bf; В}> — 0. Процедуру пометок вершин следует повторять до тех пор, пока вершина В„ не сможет быть помечена с помощью указанной проце¬ дуры. Это' будет означать, что достигнуто максимальное значение потока, допустимое для-данной сети. ■ С доказательством этого факта можно познакомиться, напри- мер, в книге Форд JI., Фалкерсон Д. Потоки в сетях (М., Мир, 1966). Покажем работу этого алгоритма на примере уже рассмотрен¬ ной здесь простой сети, для которой известно значение максималь¬ ного потока. Убедимся, что алгоритм приведет к тому же резуль¬ тату. Пусть в сети Т задан нулевой поток (рис. 29). I. Индексом 0 пометим В0. Индексом +0 можно пометить только Вг и В2. Помечаем их. Индексом +1 можно пометить только Вп. Помечаем. .Процесс пометки останавливается, так как сток Вп уже помечен. II. Начнем операцию увеличения потока. Выделим последова¬ тельность вершин от стока до «сточника, помеченных индексами предшествующих вершин: В„, Ви В0, тогда /i — {<Bo'> Вг>\ <Bi, В„>} и Д = min {(3 — 0); (5 — 0)} = 3. Так как все ребра в 1\ направлены от В0 к Вп, то увеличиваемзначение потока по ним на 3. Сотрем пометки (рис. $0). III. Снова начнем процесс пометок вершин. Индексом 0 поме¬ тим В0. * Число Д можно определять постепенно, в процессе меток вершин. Дл$ это- то используют вторую пометку у -вершины. Рис. 29 143
8г Рис. 30 1+Z Рис. 31 Индексом +0 можно пометить теперь только В2. Помечаем. Индексом +2 можно пометить Вп и Вг. Процесс пометок оста¬ навливается, так как сток уже помечен (рис. 31). IV. Выделяем последовательность вершин от стока до источ¬ ника, помеченных индексамипредшествующих вершин: {В„, ба, В0}, тогда /а = {'<В0; Bz>, <В2; Вп>). А — min {(6 ^0); (3 — 0)} = 3. Так как все ребра в 72 направлены от В0 к:Вп , то навсех этих ребрах значение потока увеличимна 3 (рис. 32). После этого стираем метки. V. Снова помечаем вершины (рис. 32)Л%дексом0 помечаем В0. Индексом +0 можно пометить только В2. Помечаем. Индексом +2 можно нометить только B{. Помечаем. Индексом 4*1 можно пометить только Вп. Помечаем. Процесс пометок останавливается, так как сток. Вп помечен. VI. В этом случае l^ = {<В0; Ва>, <В2; Bt>, <Bt; Вп >}. Снова все ребра направлены от В0 к Вп. Д = min {(6 — 3); (2 — 0); (5 — 3)} = 2. Увеличиваем $начение потока на 2 (рис. 33). Сти¬ раем пометки. VII. Снова метим вершины (рис. 34). Индексом 0 отметим В0. Индексом +0 можно отметить только В2. Помечаем. Процесс поме¬ ток на этом останавливается. (Индексом +2 уже нельзя пометить ни Ви ни В , так как ребра <В2; Вп > и <В2; Bt> насыщены. Индексом —2 тожё нельзя пометить Вь так как ф12 = 0.) Согласно алгоритму это означает, что найденный поток нмеет максимальное значение, т. е. фтах (Г) = 8. Заметим, что множество вершин сети разбилось на два подмножества Р и Р, одному из которых прш?ад- *V *1 Рис. 32 Рис. 33 144
Рис. 34 Рис. 35 лежит источник, другому — сток (рис. 34). Именно ребра, направ¬ ленные от помеченных веригин к непомеченным, образуют разрез ;е минимальной пропускной способностью. На рисунке 34 оНи вы- щелены более жирными линиями а (P, Щ *= 3 + 2 + 3 == 8. Итак, *£ледуя описанному алгоритму, мы нашли фтах (T) и разрез с мини¬ мальной пропусКной способностью. -5 Работу алгоритма проиллюстрируем еще на одНом примере. Пусть в той же сети каким-тообразом уже задан другой полный дюток (рис. 35).xp (Г) = 6. Ни по одному пути от В0 к Вп поток уве- личить нельзя. Воспользуемся алгоритмом. ,- I-. Б0 получает пометку О. Индексом +0 можно отметить только Зй. Индексом +2 нельзя пометить ни одну из вершин. Индексом —2 можно пометить Bi, так как ребро <Вг; В2> направлено к В% >и ф1а>0. Вершина Вп получает пометку +1. w;' IL Последовательностьсоответствующих ребер I — {<S0; В2>, |<:Br; Вй>, <By, Вп>} на рисунке 35 выделена более жирными ^линиями. Tf Д = шin {(6.— 3); 2; (5 — 3)} = 2. На ребрах, принадлежа- 4Bfix 1 и направ.^нных от В0 к Bnt увеличиваем значение потока на Д. Нат>ебрах, направленных от Вп к В0, уменьшаем значение потока ;на А. Это приводит к увеличению значения потока в сети на 2. По¬ дучаем максимальное вначение потока. Если продолжить процесс меток, то он оборвется, не дойдя до B^ s Мы проследили за работой алгоритма на примерах простых се¬ тей. При решении более сложных сетевых задач на ЭВМ составля¬ ется специальная программа. У п р а ж н е н и я - 15. Используя Ъписанный алгоритм, определите максимальное значение потока в сети, изображенной: а) на рисунке 19; б) на рисунке 25;. -в) на рисунке 26. 16. Используя описанный алгоритм, определите максимальное значение потока в сети, изображенной на рисунке 36. 115
РОДСТВЕННЫЕ ЗАДАЧИ Сетевые задачи о потоках имеют большое практическое значе¬ ние. Приведем три задачи, сводящиеся к задачам о потоках. 1. Задача о кратчайшем пути. Эта задача является частным случаем задачи о потоке минимальной стоимости. Пусть сеть (рис. 37) представляет собой карту дорог; В0 — юрод, из которого начина¬ ется маршрут, Вп — город, в котором заканчивается маршрут, а Сц — расстояние между городами Bt и Bj. Будем считать, что f (T) — 1 и для каждого ребра <B,; Bj> аи = 1..Мы как бы от¬ правляем одну единицу товара из пункта В0 в пункт Вп так, чтобы общее расстояние, пройденное этой единицей, было минимальным*. 2. Транспортная задача. Пусть имеется т пунктов отправ¬ ления Аи А2, -.., Ат и k пунктов назначения Du &2> -*м Dk. В каж¬ дом из пунктов отправления находится некоторый однородный про¬ дукт в количестве ait 02* •••» ат единиц соответственно. Весь имею¬ щийся продукт необходимо доставить в пункты назначения в коли¬ чествах du d2, ...» dk единиц соответственно. Стоимость перевозки единицы продукта из At в Dj равна clJa Количество продукта, пере¬ возимого из At в Dj, обозначим Хц* * Задача состоит в отыскании плана перевозок из пунктов Аг в пункты Dj с минимальной стоимостью. Другими словами, в опре¬ делении количества продукта xtJ , которое нужно отправить по каждому пути <Лг; Dy>TaK, чтобы суммарная стоимость перевозок была бы минимальной. Такую сеть всегда можно преобра¬ зовать в транспортную с одним источником и одним стоком (рис. 38). Эта задача сводится к задаче нахождения максимального зна¬ чения потока, который при этом имеет минимальную стоимость. Данная модель имеет широкий круг приложений. Пункты от¬ правления и назначения рассматриваются, напшщер, как склады и магазины или системы портов, а коэффициенты ^j— как расстоя¬ ния* время, стоимость перевозки. * С одним из алгоритмов решения такой задачи можно познакомиться в кни¬ ге Гасс С. Путешествие в страну линейного программирования. М., 1978, с. 102. 146
—3^5ВП Рис. 38 3. Задача об оптимальном назначении. Эта же модель применима для решения задачи а_наилучшем назначении. Пусть m = k, вершины Ai, Аг,..., Ат соответствуют рабочим, вершины DUD2, .... Dm видам машин. Предполагается, что каждый ра¬ бочий может обслужить любую машину. Производительность труда рабочего At на машине Dj обозначим через с1}. Считаем, что каждый рабочий должен быть назначен только на одну машину и каждая машина долж¬ на обслужИваться только одним рабочим. Оптимальное назначение состоит в таком распределении рабочих no машинам, при котором суммарная производительность будет наибольшей. Рассмотренные задачи отличаются друг от друга по адатематиче*- ской формулировке. При помощи простых приемов их условиямож- но свести к единой форме. ч Отметим еще, что задачу о минимизаций одной функции можно свести к задаче максимизации другой функции. Подробнее об этих задачах и их решениях можно прочитать, например, в книгах [1], C6], [10] изсписка, приведенногона странице 110 данного пособия.
ЗАДАЧИ АЛГЕБРА 1. Крайняя слева цифра шестизначного числа единица. Если эту цифру перенести на последнее место справа, то полученное чис¬ ло окажется втрое больше первоначального. Найти первоначаль¬ ное чнсло. 2. Доказать, что, каково бы ни было натуральное число ti, число n8 — п делится на 6. 3.' Найти все такие простые числа р, чтобы 8ра + I -было про¬ стым. 4. Доказать, что существует натуральное число, десятичная запись квадрата которого начинается с 99 девяток. 5. Какое из чисел больше: 10»»« + i ю1,м + i ^ : ИЛИ ? 10»»»* 4-1 ю1ем + i 6. Доказать, что при любых значениях х сумма х10 — х9 4г 4- дг4 — Xs Ц- 1 всегда принимает положительное значёние. 7. Пусть х и у катеты прямоугольного треугольника, г — его. гипотенуза. Тогда х% + у* = г*. А чтр можно сказать о х3 +'y3? Эта сумма больше, чем z*, или меньше? Что можно сказать о'о?мме — + — ? Больше она, чем —, или меньше? х* у* ** 23 3s 8. Какое из чисел больше: 23 или 3s ? В. Произведение, Нескольких последовательных нечетных чисел оканчивается -цифрой 9. Сколько в этом произведении может быть сомножителей? -10. Сделать рациональными знаменатели дробей В) ут+ yr+ /Г; б) ^r+ Yb + 3/Г- If. Автомобиль идет со скоростью 60 км7ч. На сколько надо увеличить ero скорость, чтобы в минуту выиграть километр рас¬ стояния? ^ 148
12. Найти значения х, при которых 2(x* + 4**-12) + ** + H** + 3Q 10 ** + 6 13. Иванов шел с постоянной скоростью и по дороге встретил Петрова, который задержал его на время Т = kv, где k ~г- поетоян- ная величина. Затем Иванов продолжал путь с той же скоростью. Расстояние от дома Иванова до пункта назначения равно е. С ка¬ кой скоростью Иванову надо было выйти, чтобы быстрей достичь цели? 14. В шахматном турнире участвовало п шахматистов — грос¬ смейстеры и мастера. После окончания турнира оказалось, что каждый участник набрал ровно половину своих очков в партиях против мастеров. Доказать, что У~п — целое чйсло. ГЕОМЕТРИЯ 15. Даны три параллельные прямые. Построить равносторон¬ ний треугольник, вершины которого лежали бы на этих прямых. 16. ABC — прямоугольный треугольник. На его гипотенузе АС и катетах АВ и ВС построены квадраты соответственно ACFE, ABHD, ВСМК• Затем дополнительно проведены отрезки НК, FM и DE. Доказать, что площади треугольников HBK, FCM и DAE равны. 17. Сформулировать и доказать теорему, обратную теореме Пифагора, считая теорему Пифагора доказанной. . 18. Доказать, что в любом треугольнике c9 сторонами а, Ъ и с a* + Ь2 — kc2, где k > —. 2 19: В равнобедренномтреугольнике ABC (| АВ | = | ВС |) угол ВАС равен 40°. На биссектрисе угла А взята точка 0 так, что про¬ веденный из нее отрезок ОС составляет со стороной ВС.треугольни- ка угол, равный 10 . Точки 0 и В соединены отрезком прямой, Най- ти угол CBO. 20. Около прямоугольного треугольника АВС описана окруж¬ ность, а на его катетах АВ и .BC, как на диаметрах, построены полукруги, при этом внешним образом. Окружность отсекает от полукругов луночки (серпы). Доказать, что чэбе луночки имеют одинаковую ширину1, т. е. равны диаметры наибольших вписан¬ ных в луночки окружностей. 21. Земной шар опоясан по экватору шнуром. Этот шнур уд¬ линили на один метр и равномерно растянули. Пролезет ли в об¬ разовавшийся зазор мышь? 1 22. Дан угол и внутри него точка. Провести через эту точку прямую так, чтобы отрезок прямой, заключенный внутрй угла, делился этой точкой пополам. ' ' 23. Пусть О — внутренняя точка произвольного треугольника АВС. Прямая АО пересекает отрезок ВС в точке Alf ВО пересекает Ы9
АС в точке Ви СО пересекает АВ в точке Сг. Доказать, что верно следующее равенство: - - 1^1 и \СА^ щ |ВСг\ = j \Bfil \А,В\ | CtA 1 24. В произвольнойтрапеции ABCD проведена прямая EF, параллельная основаниям AD и ВС (Е £ [ЛВ], а F £,[CD]). Эта прямая чгересекает^иагональ трапеции ЛС в точке К* а ВЛ — в точке AJ. Доказать, что отрезок ЕК равец отрезку MF. 25. Дан произвольный четырехугольник (выпуклый). Доказать, что четырехугольник, вершинами которого являются середины сторон данного четырехугольника, есть параллелограмм, площадь которого вдвое меньше площади данного четырехугольника. 26. В произвольиом,выпуклом четырехугольникеЛВСО сере¬ дина N стороны АВ и середина L стороны DC соединены отрезками прямых с серединой К диагонали АС и серединой М диагонали BD, Доказать, что четырехугольник KLM'N параллелограмм. 27. Дана фигура (выпуклая), ограниченная дугой ВС окруж¬ ности и*двумя пересекающимися в точке D прямыми, проходящи¬ ми через концы дуги окружности. Провести прямую так, чтобы она разбила данную фигуру на две фигуры равной площади. 28. Доказать, что на плоскости нельзя расположить k точек так, чтобы каждая из них^была серединой отрезка, соединяющего какие-то две другие точки данного множества. 29. В данный квадрат со стороной а вписать квадрат c6 сторо¬ ной fc, где ^L <^ b <^ а. V *• 30. Дан произвольный треугольник ABC. Выбираем на стороне ВС произвольную точку Рх так, чтобы проведенная дуга окруж¬ ности с центром в вершине В и радиусом, равным отрезку ВР1л пересекла сторону АВ. Точку пересечения этой дуги окружности с АВ обозначим,Р2/ Далее проводим аналогичную дугу окружно¬ сти с центром’в точке А и радиусом, равным отрезку АР2 до пере¬ сечения с АС. в точке Р3 и т. д. (при этом вершины треугольника АВС обходятся против часовой стрелки). Доказать, что: а) точка Р7 совпадет с точкой Рг\ б) точки Ply Pz, P^P^ Р&* Р6 лежат на одной окружности. 31. Доказать, что сумма,всевозможных векторов, начала кото¬ рых помещены в центр правильного многоугольника, а концы — в его вершины, равна нулю. РАЗНЫЕ ЗАДАЧИ 32. Как с помощью одной лишь линейки с делениями определить полную емкость обыкновенной бутылки с плоским круглым дном, если площадь дна известна? 33. *Самолеглетит по прямой при отсутствии ветра с постоянжЭй скоростью из пункта А в пункт В, а затем обратно. Будет ли за¬ 150
трачено на тот же путь меиьше, больше или то же самое время, если ветер.дует с постоянной силой и направлением? 34. Пусть х и у положительные числа, х ф у, х ф —у, какая j^2 I _ п2 j^2 _ _ ц2 из дробей больше: —^- или —? х+у х—У 35. На развилке двух дорог, одна из~которых ведет в город лжецов, а другая в город правдистов, путник встретил жителя одно¬ го из этих городов. Задав ему единственный вопрос, он узнал, ка¬ кая дорога куда ведет. Какой это был вопрос? Указания: 1) вопрос таков, что ответом на него служит слово да или слово шт\- 2) ответ на вопрос не дает возможности узнать, из какого города встречный. ' 36. Петр и Павел не виделись с детства; случай свел их в одном из переулков родного города, Петр радостно сообщил Павлу, что у него уже трое сыновей. «Сколько же им л£т?» — снросил Павел. «А вот сколько, ■— ответил Петр, — произведение чисел их пол¬ ных лет равно 36, а сумма — числу окон в доме напротив». Павел подумал и сказал: «Эгого недостаточно, чтобы узнать, сколько лет каждому». Тогда Петр ‘^добавил: «Старший — рыжий». Сколько лет каждому сыну Петра и как об этом догадался Павел? 37. Доказать, что наибольшего простого числа несуществует. 38. Расстояние по воздуху между городами А и С равно 57 км, между городами А и D — 105 км, между городами С и D — 48 км. Город В удаленот города С на 120 км, а от города D на 72 км. Ка¬ ково расстояние по воздуху между городами А и В?. 39. Можно ли из шести спйчек сложить четыре равносторонних треугольника? 40. Найти множество точек земного, шара, обладающих следую¬ щим свойством: если, выйдя из любой такой точки, пройти 10 км по меридиану на юг, затем 10 км по параллели на восток и затем снова по меридиану 10 км на север, то вернешься в исходную точку. 41. Пусть а, b и с — стороны треугольника.Доказать, что если а2 + Ьг + с3 = ab + bc + ca, то треугоЛьник — равносторонний. 42. Если Петя идет на стадион пешком, а возвращается оттуда на троллейбусе, то дорога туда и обратно занимает у него полтора часа. Если же в оба конца QH едет на троллейбусе, то весь путь длится 30 мин. Сколько времени затратит Петя на путь туда и об¬ ратно пешком? 43. Придумать утверждение, справедливое для рюбого числа, за исключением чисел 7 и 53. 44. У меня есть друг Платонов Егор. Интересно, что, переставив буквы, входящие в его имя и фамилию, можно получить назвацие его профессии. Какова профессия'моего друга? te 45. Из вершины квадрата, длина стороны которого равна 3 см, проведены два отрезка, делящих квадрат на три фигуры с одина¬ ковыми Шющадями. Чему равна длина каждого из проведенных отрезков? 151
Ответы, решения, указания fc задачам 1. 100000 + х — первоначальное число, тогда 10.v + 1 — чис¬ ло, полученное после перенесения 1 на последнее место. По усло¬ вию 10jc + 1 = 3 (100000 + *), откуда х = 42857, а искомбе число равно 142 857. . 2. л3 — п — h (tt2 — 1) = п (п — 1) (n + 1). Из трех после¬ довательных натуральных чисел одно делится на 3 и по крайней мере одно четно, т. е. п3 — п делится на 2 и на 3, а значит и на 6. 3. Любое простое число рл не равное 3, можно представить в виде 3п + 1 лйбо в виде 3n — 1, где п натуральное. В первом слу¬ чае 8p2 + 1 — 72пг + 48n + 9, во втором 8p2 + 1 = 72п2 — — 48ft + 9. Значит, если р Ф 3, то 8p2 + 1 кратно З и, очевидно, больше 3. В этом случае 8p* + 1 — число составное. Если же р = 3, то 8р2 + 1 = 73 — простое число. Итак, р = 3 — единст¬ венное простое число, удовлетворяющее условию. 4.Условию удовлетворяет, например, число п — 10101—1. В самом деле, n* = Ю202 ^ 2 • 10101 + 1 - 10ш (10‘01 — 2) + 1. Из Последнего равенства видно, что первые 99 цифр, числа п2 — дев'ятки. N - п Ю«-»+1 ^ 10" н-1 5. Покажем, что ^— 1 .10"+l 10л+1 + 1 при любом натуральном -n. Действительно, (10*^+l){l0"+* + + 1) > (10я + 1)2, в чем нетрудно убедиться, раскрыв скобки. Поделив обе части неравенства на (10“ -|- 1) (10"+1 + 1) и под¬ ставив п. — 1982, получим требуемое.. 6. Разберем три случая. 1) х <0. Тогда каждое из слагаемых выражения х10 + х* + 1 + (—х° — х3) положительно. 2) 0 ^ х ^ ^ 1. Тогда каждое из слагаемых выражения я4 (1 — Xs) + л10 4= + (1 —^) положительно. 3) х > 1: Й в этом случае каждое из слагаемых выражения х® (jc — 1) + дс3 (х — 1) + 1 положительно. - 7. Пусть х <^ у ^ 'z, тогда х2у + у® = г2у и я3 + у8 ^ z2y^z®. Итак, jr + у* ^ 2®. Из условия задачи следует, что • *M^l+l = _fL>l>^, т. e:l + l>I. х 2у2 х2 у2 х2у2 ха z2 х2 у2 i2 8. Имеем, что 2*8 = 28?‘8, что больше 2зТ* * = 43?, что больше 3*7, ио 37 > 3е > 29, поэтому 3*?> 32*. Следовательно, 2®23 > 3^2. 9. 2 или 4 (примеры 9 • llj 7 • 9 • 11 • 13). Произведение трех последовательных нечетных чисел не оканчивается; цифрой 9. В произведении больше четырех сомножителей- один из них, а, следовательно, и все произведение оканчиЬается цифрой 5. 10. а) ^ !_ _-V^+Vr-Vr ут+уГ-УГ Ya + Yb+Yc (yt + yTf — c а + Ь — с + 2УаЬ “ (V~iT+ Vt^- yt)(a + Ь — с — 2Vd>) tS2 (а + Ь — cf — 4ab
б) Следует воспользоваться тождеством (х + у + z) (л2 + уг + 4- гs — ху — хг — уг) =■ х3 + у8 + z3 — 3xyz. Примем х ~ ^a, у — yfb, г = Y.c> тогда знаменатель дроби приведется к виду d — Уl- Теперь числитель и знаменатель остается умножить на d? + dl'1' + Г/а, и иррациональности в знаменателе не останется.' 11. На один километр пути затрачивается одна минута.Требуется, чтобы в минуту автомобиль проезжал 2 км. Это достигается, если он идет со скоростью 120 км/ч. 12. Раскрыв скобки и приведя подобные члены в числителе, по¬ лучим выражение Зх1 + 19ла + 6 x*+-6 Оно преобразуется к виду 1-+ Зх2. Решая неравенство 1 + Зх2 < < 10,. получаем |х| < yr3. , 13. Время, которое тратит Иванов в пути до пункта назначения, вычисляется по формуле t = — + kv. Преобразуем это выражение, вынося за скобки постоянный множитель V"sft. Получим '"V^(/H+/H- Известно, что если г > 0, то z + — ^> 2, причем наименьшее зна¬ чение, равное 2, это выражение принимает при z = — =. 1. Полу- 2 . _ чаем, что ^—* v = 1, и искомое значение v = ^f ^. . 14. Нетрудно убедиться, что в турнире с k участниками играет¬ ся JiSk n^ - партий. Пусть в турнире участвовало т мастеров и g 2 .. ^ ' гроссмейстеров,, т. е. п = m + g. Прй любом исходе каждой пар¬ тии между двумя игроками суммарное число очков, набранных обоими, равно 1. Следовательно, сумма очков, Набранных мастера¬ ми в партиях против мастеров, равна числу партий, сыгранных между мастерами; т. е. ™(т~~*\ Так как каждый мастер половину 2 ’СВОИХ.ОЧКОВ набрал в партиях с гроссмейстерами, то общее число очков, набранных мастерами в партиях против гроссмейстеров, также равно 171 ^т~—. Из аналогичных рассуждений следует, что сумма очков.набранных гроссмейстерами в партиях против мастеров, равна f^g ~—. Число партий, в которых гроссмейстер нграл против мастера, равно mg. Отсюда: кз
~2— + ~^ = т8'т* — 2тё+ё2 — т + g',m*-2tng + g® = п. т. е. п = (m —g)2 и Vп — | т —g|. Следовательно, V^n — целое число. 15. Предположим, что задача решена, т. е. вершины равносто¬ роннего треугольника АВС лежат на трех данных параллельных прямых а, b, с, причем А 6 о, В 6 bt С 6 с. Пусть для определен¬ ности прямая b находится между прямыми а и с и вершины треуголь¬ ника АВС обходятся против движения часовой стрелки. Заметим, что если повернуть треугольник АВС относительно вершины А на 60ч против часовой стрелки, то вершина В займет положение вер- шины- С. Зададим произвольно положение вершины А искомого треуголь¬ ника АВС на прямой а и повернем плоскость относительно верши¬ ны А на 60° против часовой стрелки. При этом прямая b в новом положении пересечется с прямой с (в старом положении) в точке С. Аналогично поворотом на 60° по часовой стрелке относительно точки А найдем третью вершину В. 16. Покажем, например, что SMCF = $ивк — I ?н I' I в^ ^ 2 Действительно, SMCF = -^ | CAf |. C7^l - sin MCF= ^lCM | • |CF|sin^0°'+ BAC) = «=-|ВК| • \АС\ • совВЛС = — \BK\-\BH\, 2 2 что и требовалось. Равенство SDAB = $нвк доказывается аналогично. 17. Сформулируем теорему: если а, b и с — стороны треуголь¬ ника и а2 4- b2 = с2, то треугольник прямоугольный. Проведем до¬ казательство. Рассмотрим прямоугольныйтреугольник с катетами а и b. Тогда гипотенуза его находится из равенства а® + Ьг = с* по прямой теореме Пифагора. Но этот треугольник будет равен данному по трем сторонам, поэтому и его угол С будет прямой. 18. Возьмем произвольный треугольник. Дополним его до па¬ раллелограмма. Одной диагональю его будет с, другбй d. Дока¬ жем сначала, что для параллелограмма, стороны которого равны а и b, а диагонали — d и с, верна формула с? + еР — 2 (а®. + b2). Действительно, по теореме косинусов с® = а2 + b2 — 2 ab cos £, d2 = oa + b2 + 2 ab cos Cl Складывая почленно эти равенства, получаем искомую формулу, из которой следует, что а* + &»=£±*>*. • 2 2 154
Итак, действительно, а2 4- b2 = kc2, где k > ~. 19. Выполним дополнительные построения так, чтобы данная фигура явилась частью другой, свойства которой были бы хорошо известны. При рассмотрении новой фигуры получим новые сведения о данной, которые и приведут к ответу на поставленный в условии вопрос. Этой новой фигурой может быть равносторонний треугольник ADC, основанием которого служит сторона АС данного треуголь¬ ника ABC, & вершина последнего В находится внутри треугольника ADC. Заметим, что точки D н В лежат на высоте (она же биссект¬ риса), проведенной из вершины D равностороннего треугольника, а точки 0 и С на высоте, проведенной из вершины С того же тре- ’угольника (докажите это). Если теперь провести биссектрису угла А треугольника ADC, то нетрудно доказать, что треуголвни- ки ABD и АОС симметричны относительно этой биссектрисы. Следовательно, треугольник ABO — равнобедренный. ABO = = (180° — 20°) : 2 = 80°. Но ABC= 180° — 2 • 40° = 100°. От¬ сюда и получаем искомое: OBC = АВС — ABO = 100°- — 80° = 20°. 20. Пусть R — радиус окружности, описанной околотреуголь- ника АВС, а гАВ и гвс — радиусы полукругов, построенных со¬ ответственно на катетах АВ и ВС. Тогда ширина луночки, отсечен¬ ной окружностью радиуса R от полукруга, построенного на катете АВ, равна rBC + тдв — R. Аналогично находим ширину второй луночки и убеждаемся, что она оказывается равной ширине первой луночки. 21. Пролезет не только мышь, но и кошка. Пусть r — радиус земного шара, а гА — радиус окружности, образованной удлинен¬ ным шнуром. Тогда 2яг, — 2nr = 1, откуда гг — r »= — г» — « 2it b,3 « 0,16 (м), т. е. примерно 16 см. Следует заметить, что найденная разность радиусов не зависит от r. 22. Пус|ь даны угол MON и внутри него точка А. Восполь¬ зуемся тем, что в параллелограмме диагонали точкой пересечения делятся пополам. Проведем прямую через вершину 0 угла MON и точку А. Отложим на ней отрезок | АВ\ = |ОА|. Через точку В Проведем прямую ВС параллельно [ON) и прямую BD параллель¬ но [OM). Фигура OCBD — параллелограмм, его диагонали CD и ОВ в точке А делятся пополам. Следовательно, отрезок CD иско¬ мый. Так как прямая СВ симметрична прямой ON относительно точки А, то любая другая прямая, проходящая через^ точку А, условию не удовлетворяет. iss
23. Рассмотрим пары треугольников АОС и AOB', ВОА и BOC, СОВ и СОА. Каждая пара этих треугольников имеет общую сторо¬ ну. Для них верны следующие равенсТва: S&AOC I C^l I SfrcQB I BCi I ^AВОА I ^i 1 /#\ ®ДАОВ ! ^l^l- $AСОА I ^iA I * 5дBoc ' I В\С | Докажем справедливостьодного из этих равенств, например пер¬ вого. Ёыполним вспомогательные построения: проведем (CfQ j. ± (AA^ и (BM) J. (AAJ, М и К 6 Й^г)- Треугольник Ai_M,B подобен треугольнику ALKC. Тогда I С\ 1 I СК) _ S\Aoc WB I I ^M | ЯдАОВ Аналогично доказываются остальные два равенства. Перемножая соответственно левые и правые части равенств (*), получим тре¬ буемое. 24. Треугольники BDC и АВС имеют равные высоты, проведенные соответственно из вершин D и Л; обозначймдлину этих высотчерез hi- Треугольники FDM и ЕАК. тоже имеют одинаковые высоты, проведенные соответственно из вершин D и А; длину этих высот обоз¬ начим через ft2. Из подобия треугольников MDFn £ЮСследует, что |^Ё^. = ^£? а из подобия-треугольников АЕК и ВАС следует, i ВС | Аг - ( что ^^ = —. Следовательно, L^J_-'L^1 и jMfi- | ЕК\- \ВС\ ht \ВС\ I BC[ л 1 1 1 25» Меньший четырехугольник является параллелограммбм, так как его стороны попарно параллельны диагоналям данного че¬ тырехугольника (по теореме о средней Линии треугольника). Диагоналк данногочетырехугольника делят его на четыре тре¬ угольника, а параллелограмм — на четыре параллелограмма. Каж¬ дый из этих параллелограммов имеет площадь, вдвое меньшую, чем площадь соответствующего треугольника (проведя диагональ в параллелограмме, получаем четыре 'равновеликих треугольни¬ ка). Следовательно, площадь всего параллелограмма вдвое меньше площади данного четырехугольника. 26. Указание. Воспользоватьсясвойством средней линии тре¬ угольника. 27. Пусть точка А является серединой данной дуги. Проведем отрезки ВС, AF ± ВС (F 6 [BC]) и FD. Тогда ломаная AFD де¬ лит заданную фигуру BDCA на две фигуры равной площади Дока¬ жите это). Заменим сечение данной фигуры ломаной AFD сечением прямой. Для эТого проведем Ътрезки DA, FE \\DA {E 6[OC]) и AE. Заметим, что треугольники DAF и DAE иМеют равные пло¬ щади. (Они «меют общее основание DA и равные высоты, проведен¬ ные из вершин Е и F k -этому основанию, так как Е и F лежат на прямой, параллельной основанию ЛО.) Поэтому если в фигуре 156
BDFA, состоящей из криволинейного треугольника ABD и тре¬ угольника AFD, заменить треугольник AFD на равновеликий ему треугольник ADE, то полученная фигура BDEA будет иметь ту же площадь, т. е. прямая АЕ — искомая. 28. Число отрезков, соединяющих две из k точек, конечно. Значит, среди, них есть хотя бы один наибольший отрезок; назовем ^ero АВ, Пусть точка А есть середина отрезка CD. Один из двух "у^глов ВАС и BAD не меньше 90°. Пусть это угол ВАС. Тогда в треугольнике ВАС сторона ВС больше АВ, что противоречит вы¬ бору отрезка АВ. Следовательно, точка А не может бытьсереди- ной никакого из рассматриваемых отрезков. 29. Проведем в данном квадрате ABCD диагонали. Построим квадрат i4jBiQDj, равный искомому, т.; е. со стороной b, так, чтобы его диагонали лежали на диагоналях данного квадрата. (Предварительно построим отрезок, длина которого равна длине .Диагонали квадрата со стороной 6, т. е. Ь^2.) После этого постро¬ енный квадрат i416vQD1 повернем относительно центра О-так, Этобы точка Ах заняла положение Л2 на стороне АВ заданного рЬадрата, точка Bt положение Ва на стороне ВС и т; д. Тогда ■Квадрат со стороной b окажется, как и требовалось, вписанным ^ данный квадрат. - Из построения следует, что решений может рыть два. Одно получается при вращении по часовой стрелке, дру- Тое — против часовой стрелки. 30. Покажем сначала, что все точки Рг, Рг, Ра', Р4, ... лежат Эна окружности, концентрической по отношению к окружности, ^писанной в треугольник АВС. Впишем в треугольник АВС окруж¬ ность. Пусть О — ее центр, а Q„ Q2, Q3 — точки касания этой рсружности со сторонами треугольника АВС, причем Qr 6 [BC]l Bs е iABy, Q3 € lACl Тогда lBQjl = lBQ,l и _\ВР1\ = \ВРг\. роэтому l^iQxl= | P2Qi I и | бРг | — 10Р2|. Точно так зке доказы- увается, что |ОР3| = [0Р4| = |ОР2| = ... Итак.всеточки Pv Рг, р®?, ... лежат на окружности, концентрической с окружностью, «писанной в треугольник АВС. Но точёк пересечения этой окруж¬ ности с прямой ВС всего две; одна из них — Рл, а другой может фыть только Рх. "г 31. Полученный вектор-сумма является нулевым вектором. Действительно, если повернуть правильный n-угольник на угол. POjr r^r-fe, где k = 0, 1, 2,..., то вектор-сумма не должен менять свое -*л ЗДачение. Это возможно только в том случае, если вектор-сумюа ;йвляется нулевым вектором. - 32. Наливаем примерно полбутылки воды и держим бутылку вертикально горлышком вверх. Измеряем линейкой высоту столба Воды и умножаем ее на площадь основания; получаем некоторое число Vi. Переворачиваем теперь бутылку горлышком вниз, не давая воде пролиться, и измеряем лннейкой высоту столба воздуха вбутылке; умножая это число на площадь дна, получаем число V't, сумма которого с Vx и есть емкость бутылки. 157
33. Больше. На первый взгляд может показаться, что время будет то же самое, так как опереженйе графика при попутном ветре компенсируется отставанием при встречном. На самом деле этс не так. Пусть v — скорость самолета, х — скорость ветра, tt — время полета при отсутствии ветра, t% — время полета при ветре; путь от А до В примем за t. Тогда v + х v — х v2 — х2 v __ £_ v v— — V ->- v2 ^ • V 34. *±£а (х + ^-2,у =;с *JL;*=£e* + j. х + У‘ Х+У JC + y х — у Следовательно, х2—уа ^ x*+y* х — у x + y' 35. Путник, указав на одну из дорог, задал вопрос: «Ведет ли этадорога в твой родной город?». Кто бы ни был встречный — лжец или правдивец, он ответит «да», если дорога ведет в город правдивцев, и «нет», если дорога ведет в город лжецов. 36. Обозначим возрасты сыновей Петра через х, у и z; тогда хуг =* 36; 36 « 2 -18 • 1 = 3 • 12 • 1 = 4 • 9 • 1 = 6 • 6 • 1 = =« 2 • 2 • 9 =з 2 ■ 3 • 6 =5 3 • 3 • 4 =з 36 • 1 • 1. Сообщение Пет¬ ра, что сумма возрастов его сыновей равна числу окон в доме на¬ против, не снимает неопределенности; значит, выбирать нужно из тех троек сомножителей, суммы которых равны. Таких троек ровно две: 6.+ 6 + 1 = 13, 2 + 2 + 9 = 13. Дополнительная информация «старший — рыжий» позволяет исключить тройку 6, 6, 1. Значит, сыновьям.Петра 2 года, 2 года и 9 лет. - 37. Допустим, что существует самое большое простое число ря. Рассмотрим число q, равное рх • р2 • ... • рп_х • рп + 1, где рх, рг, ..., рп_х — простые числа, меньшие рп. Число q не делится ни на одно из чисел plt p^,..., р„ (в остатке всегда будет1); значит, оно либо само простое, либо делится на простое число, большее, чем рп. Следовательно', наибольшего простого числа не существует. 38. Города А, С и D расположены на одной прямой, причем город С лежит между А и D, так как \АС\ + jCD| = \AD \. Города С, D и В расположены на одной прямой и D лежит между С и В, так как |CD|+ \DB\ «= |СВ|; следовательно, все четыре города расположены на одной прямой в следующем порядке: А, С, D, В. Расстояние между А и В равно |АС\ + \CD\ + \DB\ *= 177 км. 39. Можно. Надо построить- из данных спичек треугольную пирамиду. 158
40. Одной из таких точек является Северный полюс. На первый .взгляд можеу показаться, ^fo этим решение исчерпывается: два меридиана пересекаются только в полюсах; от Южного полюса пути на юг нет; следовательно, единственное решение — Северный полюс. Однако это решение получено в предположении, что первый й последний участки пути проходят по различным меридианам, тогда как условие задачи не исключает возможности совпадения этих участков. Допустив, что первыйитретий участки пути проходят по одному и тому же меридиану, получаем, что в искомое множе¬ ство, кроме Северного полюса^' входят еще и все точки параллели, расстояние которой по меридиану от параллели длиной в 10 км (вблизи Южного полюса) равно 10 км. 41. Равенство а2 + b2 + с2 = ab + bc + са равносильно ра¬ венству (а — b)2 + (а — с)2 -+ (b — с)2 = 0, которое справедливо только в том случае, если все три слагаемых равны 0. Отсюда a=b, b = с, а = с, т. е. а = b = с. 42. Если Петя дважды проделает путь на стадион и обратно первым способом, то он при этом дважды пройдет расстояние меж¬ ду домом и стадионом пешком и дважды проедет его на троллейбусе, затратив на все три часа. Путь на троллейбусе в оба конца зани¬ мает полчаса. Следовательно, весь путь решком Петя может пройти за два с половиной часа. 43. (х — 7) (х — 53) Ф 0. 44. Невропатолог. 45. 13 см. Площадь квадрата равна 9 см2; следовательно, каж¬ дый из прямоугольных треугольников, отсеченных проведенными отрезками, имеет площадь, равную 3 см2. Площадь прямоугольного треугольника равна половине произведения его катетов. Посколь¬ ку один из катетов равен 3 см, то другой равен 2 см, а гипотенуза ^13 см. ДОПОЛНИТЕЛЬНАЯ ЛИТЕРАТУРА Бартенев Ф. А. Нестандартные задачи по алгебре. М., 1976. Г а р д н е р М. Математические головоломки и развлечения. М., 1971. Г а р д н е р М. Математические досуги. М., 1972. Д' ь ю д е н н и Г. ^ентерберииские головоломки. М., 1979. Игнатьев Е. И. В царстве смекалки. М., 1978. К э р о л л Л. История с узелками. М., 1973. Т р и гг Ч. Задачи с иаюминкоб. М., 1975. Фетисов А. И. Геометрия 6 задачах. М., 1977. Ш т е й н г а у з Г. Сто задач. М., 1976.
ОГЛАВЛЕНИЕ П р е д и с л о в и е . 3 Системы счисления иарифметические основы электронных вычислитель¬ ных машин . . * 4 Симметрия .^ 15 Элементыматематическойлогики '. . .^ 40 Множества на координатной плоскости * * . * ~1 \ . .. , . 61 Бесконечные множества ' 86 Пряложения теории графов . . * * . ,. . . . . 98 Алгоритмы и программирование — . 111 Транспортные сети _. '. 1-29 Задачи . . . . . ■. ^. ; . 148 МЕТОДИКА ФАКУЛЬТАТИВНЫХ ЗАНЯТИЙ В 7 ~ 8 КЛАССАХ Избранные вопросы магематики Редакторы Л. В. Туркестанская, С. В. Пазельский Художник обл. Б. Л. Николаев Художественный редактор Е. Н. Карасик Технический редактор В. В. Новоселов* Корректор Р. П. Евдокимова И&№ 5834 Сдаяо в набор 19.01.81. Подписано к печати 31.07.81. 60x90*/it. Бум. тип. №3. Гарн. Лит. Печать высокая. Усл. п. л. 10,0. Усл. кр. отт. 10,25. Уч.-иэд. л. 9,75. Тираж* 147 000 экз. Зак. 43. Цена 25 коп. Ордена Трудового Краевого Знамени издательство «Просвеще¬ ние» Государственного комитета РСФСР по делам издательств, полнграфни н книжной торговли. Москва, 3-й проезд Марьяной, рощи, 41. Саратовский ордена Трудового Красного Знамени полиграфиче¬ ский комбинат Росглавполиграфпрома Государственного комитета РСФСР по делам издательств* полиграфии и киижной торговли. CapaTOB> ул, Чернышевского, 59.